Sei sulla pagina 1di 310

SUBJECT CODE BOOQET CODE

2014 (I)

PPI'BISECALSCIENCES
TEST BOOmET
Time :3:QO Hours

Maximum d4arkr: 200

KNSTRUCTH8NS
1.

"a

3
4.

5.
6.

9.

I.
9.
I .

111.

,I,

,,.-;..

You have opted for EngPish as medium oPQmestion Paper. This Test Booklet contains
sevcnty five (20 P&'Aq+25 Part 'B' + 30 Parlt 'C') Multiple Choice Questions (MCQs).
You axe required to ariswer a maximum of 15.20 and 20 questions fiom p a t 'A' 'B' and
'C'respectively. If more &an required number of questions are answered, only first 15,
20'28 questions in Papts 'A' 'B' and 'C' respectively, will be taken up for evaluation.
Answer skeet has been prnvided separately. Before you s t a t filling up your particulars,
please msure that the bookla contains requisite numbcr of pages and that these are not
torn or mutilated. If it is SO,yoi! may request the Invigilator to change the booklet of the
same code. Likewise, check t!e answer sheet also. Sheets for rough work have been
appended to the a s t booklet.
Write your Roll No.,Name and Serial Number of this Test Booklet on the Answer sheet
in the space provided. Also put your signatures in the space earmarked.
You musf darken the a~pmpriatecircles with a black bael oen related to Roll
Number. Suba'ect Code, Booklet Code and Centre Code on the OMR answer sheet.
It bthe sole responsibilitv of the candidate to aneticmlorsslv follow the instructions
given on the Answer $beet, fiiing which. the eom~aatershall not be able to decipher
the 60kkect details which mav uitirnatelv result in loss, inciodinp reiection of the
OMR answer sheet.
Each qlaestiszi in Part 'A9 carries 2 marks, Part 'R' 3.5 marks and Fatt 'C' 5 marks
respectively. There will be negative marking @ 25% for each wiolrrg answer.
Below each question in Part 'A', 'B' and '13' four alternatives or responses are given.
Only one of these alternatives is the "correct" option to the question. Ygu have to find,
for each question, the correct or the best answer.
Candidates found copying or resorting to my unfair means are liable to be disqualified
from this and future examinations.
Candidate should not write anything anywhere except on a~iswersheet or sheets for

Use of calculator is not


After the Best is over, at the perforation point, tear the 8MW answer sheet, hand
m e r the original QMW answer sheet to the invigilator and retain the carbonless
c o w for your record.
Candidates who sit for the entire duration of the exam will o9ly be permitted to cany
their Test bboklet.

-5
!. 2..?.o.Y.. ........_..
..

Sf

h
Name ...................................

N~..GSA.?
.CI.?

O k A I I ~ Sheet
~ P ~

I have verified all the information filled


in by the candidate.

F
K

Bi
P
V

1,

5acm

WTO
cm

4.

-loam

pfji

K
L

4.

3. J

2.

5. Andrcher climbs to the top of a 10 m


high building and aims at a bird atop a
trre 17 rn away. The line of sight h m
angle of
the archer to the b i d makes
45' to the horizontal. What is the heighp
of the tree?
1. 117m
27m

1. P

1. 5 Jcrn

'b

2. Consider a right-angled triangle _-ABC


where AB = AC 3. A m l e APOQ
is drawn inside it, as shorn. such that the
height o f the rectangle is twice its width.
--r--The rectangle is moved horizontally by a
distance 0.2 2s shown schematically in
thFdiagram (not to scale).

3. 37 rn

4.

\7 4

pa

47m

6 . Consider P t h e
set
of 1
(17', 17', .--,1 7 ~ ~HOW
~ 3 many
.
of h s
rlumbers end with the digit 3?
1. 60
2. 75
3. 100
w 150

s \7

Find the missing number in the triangle.

Wkai is the value of the ratio


h

1. 625

Area of AABC
Area ofPOST

4.

80

8. The time- gap. between the t~voin:;tants,,


: o n L ~ e f o r eand one after 12.00 noon,
when the angle between s h o u r hand
7

2. 400
-3. . i25

3L--2Z

1. 16
3. 50

'

3. 8 0 gsm paper is cut into sheets of 200


r
n
x x 300 mrn size and assernblz in
pickets of 500 sheets. What will be the
weight of iTj5iEket? (gSm = g/rn')
;

2.

4.

2.4 kg
4.8 kg

4. Three identical flat equilateral-triangular


p l a t e i d e 5 c m each arejlaced
together such that t h z form a trapezium.
+

'

1. 1.2 kg
3. 3.6 kg

and the minute hand is 6 6 O , is


1. 12 min
2. 16 min
: 3. 18 min
4. 24 min

9. A m-ant-s
eqyal numbe~sof shirts
and t r o u s e r r Z Z p a y s - Rs. PmO.If the
3 poa1
c o s t w h i r t s is &f 805'ana that
trouser d R s . 1 0 0 p h e n how many sh~rts
were bqught?
3-3-8w
1. 60
(&me
3. 1 5 .
4. 10

b50 a r

years of a child, the height


it~crcasesas the square root of thezge
vlltlile the weikht increases in direct

60

Ij
2

&.0.0-(

.-_-.-

........T~-.-_.-I.-.

.
. . . . . . .

-.

..

.. . .
. .. . ... . .... .

..

... .:.... ;:.


.:....

. ,
. .
. ..::i*!.,.c
. . .. .. . . ..,... ?
.
%
*
A
:

2. reduces with age.


increases with age.
4. is constant only if the
height at birth are both zero.

L3/

11. In 450 g of pure coffee powder 50 g of


chicory is added. A person buys 100 g of
this mixture andadds 5 g of chicory to
that. What would. be the rounded-off
percentage of chicory in this final

shorn (units at arbitrary)?

1. 10

drawn, wit&out replacement, so that you


blue and 7 white balls?
1. 140

13. Suppose

+,

and x have their usual mea~liags..


What is the v a w f
-

/---

((197

315) - (197 A 315)) . (197 * 315)!

/
.

17.Suppose n is a positive integer. -then


(n2 n)(Zn 1)

-+

I. may not be-divisible by 2.


2. is-always divisible by 2 but- may not

18. There is a train of length 500 m, in which


a m.m is standing at the rear end. At the

1. 41.5 km.

3. 40.0km.

1. 3
3. 12

2.
4.

4.

-E

):(

tdnh

/-

s = c ~314~114
is a constant. The ~ i b b ~
C = U - TS pV f o s this

system is
cu

2. 3
,

US

4. -

11
15

4v

28. If all horses are donkeys, some donkeys

are monkeys, and some monkeys are


men, then which
y1. Alldonkeysaremen.

t2/ Some donkeys may

23. For a particular +hemodynamic system


thezntropy S is related to the internal
energy U and Glume V by

19. A rwtangular area of sides_9


is to be covered by s G r e tiles
2, and 5 units. The minimum
..
files needed for this is

3. zero

40.5 km.
41.Okm.

2.
4.

degenerate, while the others are nondegenerate. The average., energy at


inverse tem~eratuie13 is

instant the rear end crosses a stationary


observer on a platform, the man starts
walking from the rear to the front and the
front tor the rear of the train at a constant
speed of 3 km/hr. The speed of the train
is 80 lun/hr. The distance of the man
from the observer at the end of 30
minutes is

--.

7.3.-. Some horses are men.


4. All horses
'-1

24. An op amp-based voltage follower


1. is useful for converting a low impedance
source into a high impedance source
$.-is useful for converting a high
impedance source into a low il~lpedance
source
3 has infinitely high closed loop output
impedance
4. has infinitely high closed loop gain
25. A particle of mass m in three dimensions
is in the potential

21. One gram of salt is dissolved -ier


that is filled to a height of 5 cm in a
beaker of diametei 16 cm. T b c c u r a c y
of le,"gth w s u r e m e n t is 0.01 cm while
that of mass measurement is 0.01' mg.
; W h ~------T
m e a s u r i n gth
: the fractional error bC/C is
0.8%
2. 0.14%
3. 0.5%

- .

1,

r-

....... .. ..

..

..

. %

_.

ii2
2ma2

...I-.... ,.

I...

~.

._.&_

__..r_._l._,~_

.,.

7T2

1.

. ~.

3~

Its ground state energyis

22. A system can have t


E = "0,+ E . The level

7
m
7
7
7
.
.
,
-

~ ( r ) = ( r~< a
:cc r 2 a'

..~--.
..

@to]
.

..-n2fi2

2. ma2

43 6;"

.,.

37. Consider the matrix


...

33. A particle of mass m and coordinate


-. q
has the ~ a ~ r a n i i

The eigenvalues of M are


1. - 5 , - 2 , 7
kd--7,0,7
3. 4i, 2i, 21
4. 2 , 3 , 6

38. Consider the differential equation

tvhere R is a constant. The Haniltonian


for the system is given by
with the initial conditions x(O) := 0 anii
~ ( 0 )= 1. The solution x ( t ) attains its
maximum value when t is
.

#--1/2

34.

IF^ = yz i + zxj + xyk

and C is the
circle of unit radius in the plane defined
by z= 1, with the centre on the z-axis,
++
then the value of the.integra1 $c A. d t is

2. 1

39. A light source is switched on and off=


.=c-coCGGquency J
~p c ~ ~ s q r v c ~
movin
-f?-with a velocityc----u with respect to
t u g h t source will obszr-ve the
frequency of the switching to be

35. Given

EL,P,(X)P= (I - 2xt + t2)-'I2,

3. f ( 1 - $ )

f/Z

%2

4..f(l---4

C .

f o r I t J < f , t h e v a l u e o f P5(-l)is

40. If C is the contour defined by iz( =


value of the integral
dz

:, the

-z--r

w
36. A charged particleis& a d i s t a n c e s o m
an infinite wnducting plane maintained
at zero potential. When released from

is

distance d / 2 from the plane. At- what


41. The iime period of a simple penduluni
under the influence of the acceleration

>

..

.
.

;>:..*.*.*.+,?i?fp@:~..~
,I; 1.

. ,i.;Tp'?'.i--::
.. . ..:-"f&s.&5:..
,
;
,

~*$p>.%%??..:
.
,

magnitude f i g in the ho-mn,,


. .,,.<$.,* + .
direction. Assuming small osiiU&ons.:
a ,?-2,
.-'
,the mean position -and time period -of
oseiIlation, respectively, of
bob::hll
be
1. 0 to the vertical md *T
2. 30" to the vertical and T/2
3. 60" ta the vertical and ~ / f i
4. 0" to the vertical .md ~ f f i
.:

<,.

'

Which of the following graphs reprnsents


the c w e n t is in the sewn ary coil?

4%. Consider an electromagnetic wave at the


interface between two G n e o w
dielec&ic media sf .=---da-consmts
s s m i n e2 > el md- no
chsarges on the SUP ace, the electric fieId
vector E-and the displZ&iment vector
Bin tEie two media satisfy the following
inequalities
C

9 1

__C1

I141 > llill

q E 2 1 > 1z11

and-p21
<

E < 1$1
P/I&I < IEII
4. E l
(Ell
2.

and
and

lEll

45. A time-dependent emen: (r) = K$


(where K is a constant) is*
on at
t = O in an infinite cment-carrying
wire. The magnetic vector potential a ,
pe~pndi=
distance a from the wire is
given (for tmse t > a / c ) by

141> lql
161< lEll

43. If the electrostatic potential in spherical


d

polar c75tZdEnates is
-,-----

.t/

q ( r ) = tpOe--r/ra

d c T G 2

charge density at a distance r = r o s


/
be "
/
-

ova
1. 7

:.
.

. .. ..
L

. .. .
. : .....
.

.'. . . . . . . .. .. .. . . .
. :.i.;.,.
:. .
.,

. . ..

2.

era
3 . - 7 Eovo
.

.. . . . . .. . .. . .
.
.
....

.
.

e~ovo
2 r,Z

4.---

era

2.

2eeov0

ct-427-2

dz (a2 +.2)1/2
-@F=i?

/ where cpO and r,, are constmts, then the


'

zG
,POK
*poK

z4n

jl

dz

-st

.f

-(a2+ 22)1/2

d2 c i ~ 3. 2-poK
(a2 9)
4nc -ct

--

44..
A current iipflows through the primary
. . . . .
coil-ofa G s f o m e r . Ihe gi-ap=i,(t)
a s a functionsf time t & shown in the
figr;;r^ebelow;
. .

-::

'.

czT--

.. .

.
--

. .. ..

......

.:

.>,.,,

. . . . .
.............

.;1.:-

:.: >' :

..

..

--...

wu
PART

46.

3. Decreases by 4
4. Does not change-.,

The pressure of a nonrel&vistic free


Femi gas . in three-dimensions
depends, at T = 0, on the density of
femi&n as
C

$1Tn5/3

k3- n2i3
47.

'c'

-.

figure below
I------------a

2. n'I3
4- n4i3

A double slit interference experiment

weaa laser emitting IW.of two


adjacent frequencies vl and v2
(v;: 15). The minimum path
difference between the interf&ing
beams for which the intergreence
pattern disapp'ears is

49.

Far the logic circuit shown in the

a simplified equivalent circuit is

The recently-discovered Higgs boson


sat the LHC experiment h m e c a y
mzde infOa photon and a Z boson7 If
the rest masses of the Hig@-and Z
boson xe 125 @e'89/cLand 90 ~ e ~ l c ~
respectively, and the decaying Higgs
52. A spectral line duk a transition from .
particle is at rest, the energy of the
an electronic state p to an s state splits
photon will approximately be
into three Zeernan lines in the presince
of
a 3trong lnlagnetic field. At
2. 35-GeV
1. 3 5 f i ~ e v
intermediate field strengths the num6er
3. 30 GeV
4. 1 5 d . e ~
o9bectral lines is
A permanently deformed even-even
10
2. 3
v
3.
6
4.
9
nucc&w with I P= 2+ has totsional
energy 93 keV. The energy of the next
53. A particle in the infinite square well
excited state is
1. 372 keV
2. 310keV
O<x<a
3. 273 keQ
4. 186 keV
.=
otherwise

-.- 1. _ Increases by 2

:I

The dispersion relation for electrons in


an fc.c. crystal is given, in t h ~ h t
%&ing approximati&, by -

The expectation
the particle is

/
-

-k 665

54. The average local internal magnetic

field acting on an
&@ni s a =
af-re
M is the magnetization and
a 'is a positive constmt. At a
tempewtu~eT sufficiently close to (and
abovz) the a5tical temperat~lre'Ic,
the
rnzg,s~eticsmce~tEb1Ei~y
at zero external
field is proportional to (kB is the
BsHtzmann cernstgnt)

2. (kBT 4- a);'
4. tanh(k,T.$ a) .

I. k,T-a
@k,T
-- a)--"

- .

3./4%

Csnsicier an eleciro-rr In a b.c.c. laitice


with Initice - constact a, i%>inglc
partlie v~avef~inctisn
that saisfies Lbe
Bloch theorem wiQI have the f01-m
f(?) erp(i6. f ) ; with f(i) k i n g ,

I. l + c o s ~ ~ ( ~ + ~ - z ) ] + c o r [ ~ ( - x +
+zj]
-.

cos

+ cos[:(r

[?(z + .)I

- +z)]

where a is the lattice coilstant and q,is


a constant vvi& the dimension; of
energy. $h.e w-component of the
velocity of the electrons a?
0.0) Is .
'1, -%soar~fi.
2. 2&,a/A
3. - 4 ~ , a / &
4. 4roa/h

(,:

58.

59. En o m d i r n ~ a ~ i o n8, rmdozn w&er


t&es a step with equal pr&asiify to
the left 0r.right. W"i.lat-is%fiep~obability
that the walker re"brmrxs to the s:.ziijrag
point aAcr 4 steps?
,,/3/8
2. 5/16
3,
4. 1/16
56.

""

+ css 2 cos ,
"a]

'

kya k,a:
COS 2
2

The f i ~ i o x ~ i ndzta
g is obtained in ail
experima.ra. CBhdt measure~tht:visc~sity
q as a G,'sn.crisn of moiecular weight M
fix a S ~ ~f
I p~lyiners.

fid fDzj

q (Wa-s)

990

0.28 f 0.03

5032

30 f.2

10191

259 -t- 10

The relation that best describes the


dependence of q on M is
1. n lW4j9
2. r~ iV3I2

---

59. The integral 1


: fi d x is to be & w e d
3. ~ + c o s [ ~ ( x + y ) ] + c o s [ ~ ~ + z ) ] + up to 3 decirnal places osing siFPsoo's
3-pcGnt mle. Hf the in<<mal [O,i] is
cos [f ( z f I[)
divided into 4 eaual Darts.
. the correct
4. ~ + ~ o sa [ ~ ( ~ + ~ - - z ) ] + c o s [ ~ ( - x result
+
is
1: 0.483
Y + z)] + cos (x - + z ) ]
-3.
3.057
4

-1

[E

In a classical model, a scalar (spin-8)


meson consists of a
.-antlquru k bound by a p

where a = 200 MeV f m - ' b ~ b~=d 100


MeV fm. If the masses of the quxk
and antiquark are negligible, the mass
of the meson can be estimated as
approximatcIy

,+

Let y -.- (xl x2) - -/A, where xl and


2
.-.
x2 are independent and identically
d l s t r i b u t e ~ ~ s s irz:dorn
m
variables
of ff&n ,u md starxiarc? deviation o-.
~ h E - - { ~ " ) jis~ 4
.

9. [A, li] = B

2. [A, 31 =

3.

;~,a]

41 [A, I?]

?'he gaph of a real perit.Bic frariction


f (16) for th.1: razgc: 1-33, cc+jis shown
below

satisfy ihlc curmuiztiisn relibk3~1as

+ C,

[B,C]

0, [C, A] -= 8

i- C

C, [B, &.I = A, [C,A] = B


B, ~ B , C I-/

a, Ec.Aa
/

Cs [a,C] = 0, LC,A]

L=

-+

64. The fur~ction@(x, y,z, t ) ==


cos(z vt) ~e(sin(xt ly))

= B

-=

-the

equdibn

65. The csordicates a d rnctmenta X j l P i


(i = 1,2,3) of a particle satisfy-the
canonical Poisson bracket relations

- .

1.
2.
3.
4.

{C2, C3) = 61
(C2,C3I = -Cl
{Cz1C3) = -el
(C2,C3)=C1

interface at normal incidence. The


refractive index of the dielectric
medium is n and that of the metal is
n2 = n(l 4- ip). If the beam is
polarisedl parallel to the interface, then
the phase change e x p x c e d by th;;
light upon r e f l e c t i s
1. tan(2/p)
2. t a n - ' ( u p )

and 6631 Cl) = C2


aqd (C3,C1) = -62
and CC3,C1)= C2
and {C3rC1)=-C2

A canonical transformation relates _(he


old coordigates (q, p) to-the new ones
( n j by the relations Q = q2 apad
P = g/2q. The corresponding timeA
independent generating fiuacti~rr
is
2. q'"
1. P / q 2
3. q 2 / P
4. q P 2

'

scattering grnplitude f (8) for the


potential V(s) = p e-Pr ,where P and
p are positive constants, is given, in the
Born approximation,
by
--

7 1 . y

'

67. The time evolution of a onedimensional dynkical. system is


described by

1.

If this 6& one stable and two unstable


f&ed points, then thi parameter b
satisfies
I. o < b & 1
2. b >
3. b < -31
4. b - 2
--A

92.

68. A charge (- e ) is placed in vacwm at


the point (dl 8,O), where d > 0. T&
regiozx _< 0 is filled lrnifmly with a
metal. The electric field at the point

70. A beam of light of frequency w is


reflected .. from a dielectric-rn&l

-~

Ir2k2

4mPP
~ ( b ~ i $ ) 2~

4mP~

- fi2b2 ( b2L

The ground state eigenfunction for the


potential V(x) = -6(x), where 6 ( x ) is
the delta hctiog, is given by q ( x ) =
~ e - ~ l where
~ l , A and a > 0 are
eonstants, If a perturbation H' = bxZ
correctiouo
1.

69. An electron is in the ground state -.of a


hydrogen atom. The probability that rt
is within t h e ~ o ~ a d i ---uiss
azoximately equal to

(in the following b = 2ksin; and E = =)

aaz

2.

a2

"1. A thin, infinitely long solenoid p W


a e t h e z-axis c o n i i a n n e t k f l u x4. W c h of the ' following vector
potentials corresponds to the mag~etic
fiel- an arbitrary point (x, y, z) ?
1- ( ~ X ' A Y ' A .=
~ (-%
4

,.

;rY;;; @

o )

-gas is given by

pressure, volume and temperature


respectively, m d a and b are constant
parmeters. At the critical point, where
all the roots of thz ibove-cubic equation
are degenerate, the volume is given by

is designed so that at a fieqrtncy


w = lo7 rads the intemity of the
external radiation that penetrates the
room is I% of the incident radiation.

its minimum thickness should be (given


that 1x1 10 = 2.3)
1.- 4.60 mm

..

-.

..

2. - 2.30 mm

PHYSICAL SCIENCE NET - DECEMBER 2013 (BOOKLET-C)


Part-A
1.
A cylinder of radius 1 cm and height 1 cm is broken into three pieces. Which of the following MUST be true?
(1) At least one pieces has volume equal to 1 cm3.
(2) At least two pieces have equal volumes.
(3) At least one piece has volume less than 1 cm3.
(4) At least one piece has volume greater than 1 cm3.
Soln. Radius of cylinder = 1 cm
height = 1 cm
Hence, volume = 12 1 3.14 cm3
3.24
1.046 cm3
3
Then, it is clear that at least one piece has volume greater than 1 cm3.
Correct answer is (4)

We can divide it 3 equal part then volume of one part

2.

For real numbers x and y, x 2 y 4 0 . Then the vlaue of x + y is


(1) 0

(3) 2

(2) 2

(4) 4

2
2
Soln. We know that if a b 0 then both term should equal to zero.

x2 y 4 0

x 0 and y 4 0

x 2 0 and y 4 0
y4

then x y 0 4 4
Correct answer is (4)
3.

Every time a ball falls to ground, it bounces back to half the height it fell from. A ball is dropped from a height
of 1024 cm. The maximum height from the ground to which it can rise after the tenth bounce is
(1) 102.4 cm
(2) 1.24 cm
(3) 1 cm
(4) 2 cm
Soln. The question is related to G.P.
1
512 , on second jump
2
1
1
1
512 256 , on third 256 128 ............ and on 10th bounce = 2 1 cm
2
2
2
Correct answer is (3)

If ball is dropped from height of 1024 cm, then after bounce it will jumped 1024

4.

A farmer gives 7 full, 7 half-full and 7 empty bottles of honey to his three sons and asks them to share these
among themselves such that each of them gets the same amount of honey and the same number of bottles. In
how many ways can this be done? (bottles cannot be distinghuished otherwise, they are sealed and cannot be
broken)
(1) 0
(2) 1
(3) 2
(4) 3
H.O.: 28-A/11, Jia Sarai, Near-IIT, New Delhi-16 , Ph : 011-26851008, 26861009 www.careerendeavour.com
B.O.: 48, First Floor, Mall Road, G.T.B. Nagar (Metro Gate No. 3), Delhi-09, Ph: 011-65462244, 9540292991

2
Soln. Every son get 7 bottle and 3.5 unit honey.
I
II
III
1/2
1/2
1/2
1
1
1
1
1
1
1
1/2
1/2
0
1/2
1/2
0
0
0
0
0
0
Since II and III are same. So, distribution is possible only 3 different ways.
Correct answer is (4)
A car is moving aloing a straight track. Its speed is changing with time as shown above.
Which of the following statement is correct?
(1) The speed is never zero
(2) The acceleration is zero once on the path
(3) The distance covered initially increases and then decerases.
O
time
O
(4) The car comes back to its initial position once.
Soln. Since graph firstly increasing and then decreased. Then it is clear that the acceleration is zero once on the path.
Correct answer is (4)
speed

5.

6.

If a + b + c + d + e = 10 (all positive numbers), then the maximum value of abcde is


(1) 12
(2) 32
(3) 48
(4) 72
Soln. If we take different values of a, b, c, d, e according to condition. We see that the value is maximum when value
of all are equal as 22222 = 32
Correct answer is (2)
7.

How many nine-digit positive integers are there, the sum of squares of whose digits are 2?
(1) 8
(2) 9
(3) 10
(4) 11
Soln. Since sum is equal to 2. Then it is clear that number contain digit less than 2, i.e., either 1 or 0. Hence, number
be
1 1 0000000
We can rotate the blocked digit in 8 different places. Hence, numbers are 8.
Correct answer is (1)
8.

A circle of radius 7 units lying in the fourth quadrant touches the x-axis at (10, 0). The centre of the circle has
coordinates.
(1) (7, 7)
(2) (10, 7)
(3) (10, 7)
(4) (7, 7)
Soln. Since, circle is located at 4th quardrant. Hence value of x is negative and y is positive. Therefore, only (c)
option satisfies the condition.
Correct answer is (3)
9.

One of the four A, B, C and D committed a crime. A said, I did it, B said, I didnt. C said, B did it. D said,
A did it. Who is lying?
(1) A
(2) B
(3) C
(4) D
Soln. The statement of A is satisfied by B and D but not satisfied by C. So, C is lying.
Correct answer is (3)

H.O.: 28-A/11, Jia Sarai, Near-IIT, New Delhi-16 , Ph : 011-26851008, 26861009 www.careerendeavour.com
B.O.: 48, First Floor, Mall Road, G.T.B. Nagar (Metro Gate No. 3), Delhi-09, Ph: 011-65462244, 9540292991

3
10.

A cylinder of radius 1 cm and height 1 cm is broken into three pieces. Which of the following MUST be true?
(1) At least one pieces has volume equal to 1 cm3.
(2) At least two pieces have equal volumes.
(3) At least one piece has volume less than 1 cm3.
(4) At least one piece has volume greater than 1 cm3.
Soln. Radius of cylinder = 1 cm
height = 1 cm
Hence, volume = 12 1 3.14 cm3
3.24
1.046 cm3
3
Then, it is clear that at least one piece has volume greater than 1 cm3.
Correct answer is (4)

We can divide it 3 equal part then volume of one part

11.

There are 2 hills, A and B, in a region. If hill A is located N30E of hill B, what will be the direction of hill B when
observed from hill A? (N 30E means 30 from month towards east).
(1) S 30W
(2) S 60W
(3) S 30E
(4) S 60E
N

A N30E

30

Soln.

30

B
530W

12.

Correct answer is (1)


What is the next number in the following sequence?
39, 42, 46, 50, ....................
(1) 52
(2) 53
(3) 54
+7

+8

(4) 55

+9

Soln.
36,

42,

46,

50,

55

given series is alternative increasing.


Correct answer is (4)
What is the perimeter of the given figure as above, where adjacent sides are at right angles to each other?
5cm

1cm

4cm

13.

(1) 20 cm

(2) 18 cm

(3) 21 cm

(4) cannot be determined.

H.O.: 28-A/11, Jia Sarai, Near-IIT, New Delhi-16 , Ph : 011-26851008, 26861009 www.careerendeavour.com
B.O.: 48, First Floor, Mall Road, G.T.B. Nagar (Metro Gate No. 3), Delhi-09, Ph: 011-65462244, 9540292991

4
5 cm

B
D

AB = IF + FE + EJ = IJ (since, CD = EJ)
AH + HI = AI = BC + CJ = BJ (since, DE = CJ

4 cm

AB = IJ = AI = BJ = 5cm (since, HG = IF)

Soln.

Hence, perimeter = 20

1 cm
I

from given figure.


Correct answer is (1)
14.

Three fishermen caught fishes and went to sleep. One of them woke up, took away one fish and 1/3rd of the
remainder as his share, without others knowledge. Later, the three of them divided the remainder equally. How
many fishes were caught?
(1) 58
(2) 19
(3) 76
(4) 88
Soln. We will solve it by hit and trail method. According to condition after subtracting 1, remaining number should be
2
exactly divisible by 3 and also rd part of number again divisible by 3. From option (2)
3
18
12
6 and 18 6
4
3
3
Correct answer is (2)
19 1

15.

What is the arithmetic mean of

1
1
1
1
1
,
,
,
,.....,
?
1 2 2 3 3 4 4 5
100 101

1
(2)
101

(1) 0.01

(3) 0.00111...

1
1

(4) 49 50 50 51
2

Soln. The given number can be written as


1
1 1 1 1 1
1

1 , , ,..........

2 2 3 3 4
100 101

1 1 1 1 1
1
1
1 ........

2 2 3 3 4
100 101
Hence, Average =
100
1
100
1

1
100
101 100 101
101
Correct answer is (2)

16.

25 5 3 2 4 16 4 3

(1) 61
Soln. By BODMAS

(2) 22

(3) 41/24

(4) 16

25 5 3 2 4 16 4 3
5 3 8 64 3
0 61 61

Correct answer is (1)


H.O.: 28-A/11, Jia Sarai, Near-IIT, New Delhi-16 , Ph : 011-26851008, 26861009 www.careerendeavour.com
B.O.: 48, First Floor, Mall Road, G.T.B. Nagar (Metro Gate No. 3), Delhi-09, Ph: 011-65462244, 9540292991

5
17.

Consider the sequence of ordered sets of natural numbers:


{1}, {2, 3}, {4, 5, 6}, .........
What is the last numbers in the 10th set?
(1) 10
(2) 19
(3) 55
(4) 67
Soln. Since first term contain one digit, second term contain two digit, 3rd three.
Hence, it is A.P. i.e. 1, 2, 3, .......10th term
10
10
2 1 10 11 11 55
2
2
th
Hence, last digit of 10 term = 55
Correct answer is (3)

then S10

18.

A student buys a book from an online shop at 20% discount. His friend buys another copy of the same book
in a book fair for Rs. 192 paying 20% less than his friend. What is the full price of the book?
(1) Rs. 275
(2) Rs. 300
(3) Rs. 320
(4) Rs. 392
Soln. Let the price of book = x rs.
then price for student = x

and price for his friend =

100 20 x 80
100

100

4x
rs
5

4 x 100 20

1920
5
100

192 5 100
300 rs
80 4
Correct answer is (2)

19.

366 players participate in a knock-out tournament. In each round all competing players pair together and play
a match, the winner of each match moving to the next round. If at the end of a round there is an odd number of
winners, the unpaired one moves to the next round without playing a match. What is the total number of
matches played?
(1) 366
(2) 282
(3) 365
(4) 418
Soln. Whenever game is based on knockout system the number of matches for deciding the winner is less than one
of number of players. Hence, 366-1 = 365
Correct answer is (3)
20.

What does the diagram above establish?

Note: The diagram is a circle inside a square.


(1) 3

(2) 2 2

Soln. Let side of square = a unit

(4) is closer to 3 than to 4.

(3) 4
A

then radius of circle = 9/2 unit


Hence, area of square = a2
2

a2
a
and area of circle =
4
2

Since, circle is incircle. Hence, area of square > area of circle.


H.O.: 28-A/11, Jia Sarai, Near-IIT, New Delhi-16 , Ph : 011-26851008, 26861009 www.careerendeavour.com
B.O.: 48, First Floor, Mall Road, G.T.B. Nagar (Metro Gate No. 3), Delhi-09, Ph: 011-65462244, 9540292991

a2
a2 a2
4
Correct answer is (3)
a2

21.

Part-B
A horizontal metal disc rotates about the vertical axis in a uniform magnetic field pointing up as shown in
the figure. A circuit is made by connecting one end A of a resistor to the centre of the disc and the other
end B to its edge through a sliding contact S. The current that flows through the resistor is
B
B

(1) zero
(3) DC from B to A

(2) DC from A to B
(4) AC


Soln. Due to rotation force on electrons (negative charge) = f v B towards axis of rotation

Hence, centre is negatively charged and edge is positvely charged.


So, current will flow from B to A.
Correct answer is (3)
22.

A spin

1
1 1 i
particle is in the state

in the eigenbasis of S2 and Sz. If we measure Sz the


2
11 3

probabilities of getting
(1)

and , respectively, are


2
2

1
1
and
2
2

Soln. Given : A spin

(2)

2
9
and
11
11

(3) 0 and 1

(4)

1
3
and
11
11

1
1 1 i
particle is in the state

2
11 3

Measurement of Sz gives

1
0

and with respective eigenbasis 1/2 and 1/2


2
2
0
1

Method-1:
Representing

1
0
1 1 i
3 C1 0 C2 1
11

Probability of finding Sz as

C1

1 i
3
, C2
11
11

2
9
2
2
and are C1
and C2
respectively..
2
2
11
11

H.O.: 28-A/11, Jia Sarai, Near-IIT, New Delhi-16 , Ph : 011-26851008, 26861009 www.careerendeavour.com
B.O.: 48, First Floor, Mall Road, G.T.B. Nagar (Metro Gate No. 3), Delhi-09, Ph: 011-65462244, 9540292991

7
Method-2:
Initial state of particle = i

1 1 i

11 3

1
0
Final state of particle = f1 and f2
0
1
Probability of finding Sz as

and are f1 | i
2
2

2
and f 2 | i
11

9
respectively..
11

Correct answer is (2)


23.

Which of the following functions cannot be the real part of a complex analytic function of z x iy ?
(1) x 2 y

(2) x 2 y 2

(3) x 3 3xy 2

(4) 3x 2 y y y 3

Soln. The real part and imaginary part of a complex analytic function of z x iy , will be harmonic in nature
i.e. both will be satisfy Laplaces equation. Suppose, u be the real part, then
2u
x 2

2u
y 2

2 x2 y 2 y 0

2 x2 y
Since,

24.

x 2

y 2

Correct answer is (1)


The motion of a particle of mass m in one dimension is described by the Hamiltonian

p2 1
H
m2 x 2 x . What is the difference between the (quantized) energies of the first two levels?
2m 2
(In the following, x is the expectation value of x in the ground state)
(1) x

2
(3)
2m2

(2) x

(4)

2
Soln. Given : Hamiltonian of the system : H p 1 m 2 x 2

x
2m 2

Perturbed
Unperturbed
Hamiltonian

Hamiltonian

First order correction to energy of the nth state.


1
En n x n

2m

n a a n 0

Second order correction to energy of the nth state.

2
En

m n

n x m
0
0 E
En m

2m mn

n a a m

0
0
En Em

H.O.: 28-A/11, Jia Sarai, Near-IIT, New Delhi-16 , Ph : 011-26851008, 26861009 www.careerendeavour.com
B.O.: 48, First Floor, Mall Road, G.T.B. Nagar (Metro Gate No. 3), Delhi-09, Ph: 011-65462244, 9540292991

8
The non-zero contribution comes only from m n 1 and n 1
2

En
2m

So,

1
2

0
1
2
En En En En n
2

2m 2

n 1


2m 2

Energy difference between two levels is


3
2 1
2
E

1 0


2 2
2
2
2
m

2
m

Correct answer is (4)


25.

Let nlm denote the eigenfunctions of a Hamiltonian for a spherically symmetric potential V(r). The expectation
value of Lz in the state

1
200 5 210 10 211 20 211 is

(1)

18

Soln. Given:

(2)

1
200 5 2100 10
6

(3)
211

(4)

18

20 211

Since, Lz nm m nm
1
5
10
20
Therefore, Lz Pi Lz i 36 0 36 0 36 36
i
5

16
Correct answer is (4)

Lz

1
fermions are to be distributed in two non-degenerate distinct energy levels. The
2
number of ways this can be done is
(1) 8
(2) 4
(3) 3
(4) 2
Soln. The maximum number of particles with spins can be filled in a non-degenerate energy level = 2s + 1.
Fermians are quantum particles and quantum identical particles are in general indistinguishable. If we include
the concept of indistinguishability then three identical spin- particles (fermians) can be distributed in two
non-degenerate distinct energy levels as

26.

Three identical spin

2-level

2-level

1-level

1-level

The number of ways = 2


H.O.: 28-A/11, Jia Sarai, Near-IIT, New Delhi-16 , Ph : 011-26851008, 26861009 www.careerendeavour.com
B.O.: 48, First Floor, Mall Road, G.T.B. Nagar (Metro Gate No. 3), Delhi-09, Ph: 011-65462244, 9540292991

9
On changing the direction of spin or orientation of spin, all the distribution will be identical. If we
include the concept of distinguishability due to spin or we distinguish the particles due to spin the 3identical particles with spin- can be distributed in two non-degenerate energy levels as

27.

Soln.

2-level

2-level

1-level

1-level

2-level

2-level

1-level

1-level

2-level

2-level

1-level

1-level

2-level

2-level

1-level

1-level

The number of ways = 8


But quantum identical particles are in general indistinguishable and there is no hint to consider particles
distinguishable.
Correct answer is (4)
Let A, B and C be functions of phase space variables (coordinates and momenta of a mechanical system).
If {,} represents the Poisson bracket, the value of {A,{B,C}} {{A,B},C} is given by
(1) 0
(2) {B,{C,A}}
(3) {A,{C,B}}
(4) {{C,A},B}

y A, B, C A, B , C

y A, B, C C , A, B

Jacobi identity

A, B, C B, C , A C , A, B 0

A, B, C C , A, B B, C , A

y B, C , A C , A , B

Correct answer is (4)

28.

If A, B and C are non-zero Hermitian operators, which of the following relations must be false?
(1) [A, B] = C
(2) AB + BA = C
(3) ABA = C
(4) A+ B = C

Soln. Given : A A, B B, C C

C A, B AB BA C AB BA B A A B BA AB C
So, C = [A, B] is not hermitian.
Correct answer is (1)
H.O.: 28-A/11, Jia Sarai, Near-IIT, New Delhi-16 , Ph : 011-26851008, 26861009 www.careerendeavour.com
B.O.: 48, First Floor, Mall Road, G.T.B. Nagar (Metro Gate No. 3), Delhi-09, Ph: 011-65462244, 9540292991

10
29.

The expression

2
2
2
2
1

2
is proportional to
2
2
2
2
2
2
2
x1 x2 x3 x4 ( x1 x2 x3 x4 )

Soln.

(1) ( x1 x2 x3 x4 )

(2) ( x1 )( x2 )( x3 )( x4 )

(3) ( x12 x22 x32 x42 ) 3/2

(4) ( x12 x22 x32 x42 ) 2


2
1
2 x1

x1 x1 x22 x32 x42 x1


x12 x22 x32 x42

2
2
2
2
2
x

x
x1.2 x12 x22 x32 x42
2
3
4
1
2
4

x12 x22 x32 x42

2x1

8 x12 x12 x22 x32 x42 2 x12 x22 x32 x42


x12

x22

x32

4
x42

6 x12 2 x22 2 x32 2 x42


3

x12 x32 x22 x42

6 x22 2 x12 2 x32 2 x42


2
1

3
Similary, x22 x12 x32 x22 x42
x12 x22 x32 x42

and for others we can also calculate.


2

2
2
2
1

0 for x , x , x , x 0
2
2
2 2
2
2
2
x
1 2 3 4

x
x

x
1
2
3
4 1
2
4
3
Correct answer is (2)

30.

Given that the integral

(1)

Soln.

dx

2
2
y x
2 y , the value of

y3

(2)

dx
2 2

y2 x

So, f z

(y

(3)

dx
x 2 ) 2 is

8y 3

(d)

dx

2y 3
Imz

2 2

y2 x

z = iy

1
z2 y2

4y 3

z = iy

has poles of order 2 at z iy


R

Rez

Since, both poles are not on real axis, then given integral
I

1
2 i Residue of f z at z iy
2
H.O.: 28-A/11, Jia Sarai, Near-IIT, New Delhi-16 , Ph : 011-26851008, 26861009 www.careerendeavour.com
B.O.: 48, First Floor, Mall Road, G.T.B. Nagar (Metro Gate No. 3), Delhi-09, Ph: 011-65462244, 9540292991

11


i d
1
2

z iy
2 1! dz
z2 y2

31.

z iy

i
3
3
z iy
4y
z iy

Correct answer is (2)


The force between two long and parallel wires carrying currents I1 and I2 and separated by a distance D
is proportional to
(1) I1I2 / D
(2) (I1 + I2) / D
(3) (I1 I2 / D)2
(4) I1 I2 / D2
f

Soln. Force per unit length on each wire is given by

0 I1I 2
4 D

D
I1

I2

Correct answer is (1)


32.

1 2 3 4 5
6
, , , ,
and
of turning up 1, 2, 3, 4, 5 and 6,
21 21 21 21 21
21
respectively. If it is thrown twice, what is the probability that the sum of the numbers that turn up is even?

A loaded dice has the probabilities

(1)

144
441

(2)

225
441

(3)

221
441

(4)

220
441

Soln. If the loaded dice is thrown twice, the sum of the numbers that turnup will be even if both number will be
even or both will be odd.
Required probability = P(Even Even) + P(Odd Odd)
12 12 9 9 225
=
21 21 21 21 441

Correct answer is (2)


33.

2
2
A particle moves in a potential V x y

constant (s) of motion?


(1) none

z2
. Which component (s) of the angular momentum is / are
2

(2) Lx, Ly and Lz

(3) only Lx and Ly (4) only Lz

z2
Soln. Given V x, y, z x y
2
In polar co-ordinate (spherical polar)
2

r 2 cos 2
V r sin
2
2

x r cos sin
y r sin sin

z r cos

H.O.: 28-A/11, Jia Sarai, Near-IIT, New Delhi-16 , Ph : 011-26851008, 26861009 www.careerendeavour.com
B.O.: 48, First Floor, Mall Road, G.T.B. Nagar (Metro Gate No. 3), Delhi-09, Ph: 011-65462244, 9540292991

12
Therefore, Lagrangian of system is
L=TV

1
r 2 cos2
2
2 2
2
2 2
2
2

L m r r r sin r sin
2
2

is cyclic, therefore p is constant of motion.


p is equal to Lz. Therefore, Lz is constant of motion.
Correct answer is (4)
34.

The Hamiltonian of a relativistic particle of rest mass m and momentum p is given by H

p 2 m2 V ( x) ,

in units in which the speed of light c = 1. The corresponding Lagrangian is

Soln.

(1) L m 1 x 2 V ( x)

(2) L m 1 x 2 V ( x )

(3) L 1 mx 2 V ( x)

(4) L

1 2
mx V ( x )
2

p 2 m2 V x

This is Hamiltonian of a relationistic particle with c = 1. Its very well known that Lagrangian of a relativistic
particle is
L mc 2 1 v 2 /c 2 V x

Taking v x , and c 1 , we get


L m 1 x 2 V x

35.

Correct answer is (2)


A ring of mass m and radius R rolls (without slipping) down an inclined plane starting from rest. If the centre
of the ring is initially at a height h, the angular velocity when the ring reaches the base is
R

(1)

g / (h R) tan

(2)

g / (h R)

(3)

g (h R) / R 2

(4)

2 g / (h R)

Soln. Let v = speed of the ring when it reaches the base.


2
1 2 1 2 1
1 2
2 v
2
Therefore, total K.E. of the ring I mv .mR . 2 mv mv
2
2
2
2
R

Height descended by the centre of the ring = hR


Therefore, loss in P.E. = gain in K.E.
mg h R mv 2
H.O.: 28-A/11, Jia Sarai, Near-IIT, New Delhi-16 , Ph : 011-26851008, 26861009 www.careerendeavour.com
B.O.: 48, First Floor, Mall Road, G.T.B. Nagar (Metro Gate No. 3), Delhi-09, Ph: 011-65462244, 9540292991

13

v g h R

Angular velocity

36.

g h R
R2

Correct answer is (3)


Consider the op-amp circuit shown in the figure.
1F

1K
1K

Vi

V0

If the input is a sinusoidal wave Vi 5sin(1000t ) , then the amplitude of the output V0 is
(1)

Soln.

5
2

XC

(2) 5

(3)

5 2
2

(4) 5 2

1
1

Jk
3
J C J 10 106

Z F X C ||1k

J 1 J

1 J 1 J

V0 Z F
1 J

Vi
R
1

V0
J

Vi 1 J

V0
1

Vi
11

Vi 5sin 100t ,

V0
1

Vi
2

103

Vi 5

V0

1
Vi
2

V0

5
2

V0

5 2
2 2

V0

5 2
2

Correct answer is (3)


37.

If one of the inputs of a J-K flip flop is high and the other is low, then the outputs Q and Q

(1) oscillate between low and high in race-around condition


(2) toggle and the circuit acts like a T flip flop
(3) are opposite to the inputs
(4) follow the inputs and the circuit acts like an R-S flip flop
Soln. J-K flip-flop follow the I/P and the circuit acts like an R-S flip flop
H.O.: 28-A/11, Jia Sarai, Near-IIT, New Delhi-16 , Ph : 011-26851008, 26861009 www.careerendeavour.com
B.O.: 48, First Floor, Mall Road, G.T.B. Nagar (Metro Gate No. 3), Delhi-09, Ph: 011-65462244, 9540292991

14
J
0
0
Through table of JK = 1
1

Truth table of RS =

K
0
1
0
1

Qn1
Qn
0
1
Qn

S
0

R
0

Qn1
Qn

Invalid

Correct answer is (4)


38.

Two monochromatic sources, L1, and L2, emit light at 600 and 700 nm, respectively. If their frequency
bandwidths are 101 and 103 GHz, respectively, then the ratio of linewidth of L1 and L2 is approximately
(1) 100 : 1
(2) 1 : 85
(3) 75 : 1
(4) 1 : 75

Soln.

1 600 nm,

2 700 nm

v1 101 GHz ,

v2 103 GHz

v , where is line width.

1 v1 1
v 2


hence,

2 v2 2
c

1 101 600 3600

75
2 103 700
49

Correct answer is (3)


39.

Let (V, A) and (V , A) denote two sets of scalar and vector potentials, and a scalar function. Which
of the following transformations leave the electric and magnetic fields (and hence Maxwells equations)
unchanged?
(1) A A and V V

(2) A A and V V 2

(3) A A and V V

(4) A A 2 and V V

Soln. We can choose different sets of v, A so that fields remains uncahnged after transformation.


If A ' A then V ' V d
dt
Correct answer is (1)
H.O.: 28-A/11, Jia Sarai, Near-IIT, New Delhi-16 , Ph : 011-26851008, 26861009 www.careerendeavour.com
B.O.: 48, First Floor, Mall Road, G.T.B. Nagar (Metro Gate No. 3), Delhi-09, Ph: 011-65462244, 9540292991

15
40.

Consider the melting transition of ice into water at constant pressure. Which of the following thermodynamic
quantities does not exhibit a discontinuous change across the phase transition?
(1) internal energy
(2) Helmholtz free energy
(3) Gibbs free energy
(4) entropy

41.

Two different thermodynamic systems are described by the following equations of state:

1
3RN (1)
1
5 RN (2)

and (2)
where T (1,2) , N (1,2) and U (1,2) are respectively, the temperatures, the
T (1)
2U (1)
T
2U (2)
mole numbers and the internal energies of the two systems, and R is the gas constant. Let U tot denote the
U (1)
total energy when these two systems are put in contact and attain thermal equilibrium. The ratio
is
U tot
(1)

5 N (2)
3N (1) 5 N (2)

(2)

3 N (1)
3N (1) 5 N (2)

(3)

N (1)
N (1) N (2)

(4)

N (2)
N (1) N (2)

Soln. Let T 1 & T 2 and U 1 & U 2 be the temperature and internal energies of first and second system
respectively. When both systems are in thermal equilibrium.
When both of the systems are in thermal contact then energy exchange takes place as
1
2
1
2
U U U U U total constant

When both of the systems attain thermal equilibrium their temperatures become equal.
(But T 1 T 1 & T 2 T 2 )

1
2
T T
Then,

3 1 1
1
N RT
1 1
1 1
U
2
3 N T
3 N T

U tot 3 N 1 RT 1 5 N 2 RT 2
1
1
2
2
1
1
2
2
3 N RT SN T 3 N T 5 N T
2
2
1 1
3N T
= T 1 3N 1 5 N 2

(as T 1 T 2 )

1
1
1

U T
3N

U tot T 1 3 N 1 5 N 2 this must be the correct answer..

If U 1 is the energy of first system in thermal equilibrium then


1
1
1

U T
3N

U tot T 1 3 N 1 5 N 2

1
1
U
3N

U
1
2
tot
3N 5 N

Correct answer is (2)

H.O.: 28-A/11, Jia Sarai, Near-IIT, New Delhi-16 , Ph : 011-26851008, 26861009 www.careerendeavour.com
B.O.: 48, First Floor, Mall Road, G.T.B. Nagar (Metro Gate No. 3), Delhi-09, Ph: 011-65462244, 9540292991

16
42.

The speed v of the molecules of mass m of an ideal gas obeys Maxwells velocity distribution law at an
equilibrium temperature T. Let (vx , v y , vz ) denote the components of the velocity and kB the Boltzmann
constant. The average value of (v x v y ) 2 , where and are constants, is
(1) ( 2 2 )kT / m

Soln.

vx v y

(2) ( 2 2 )kT / m

(3) ( )2 kT / m (4) ( )2 kT / m

2 vx2 2 v y2 2 vx v y
2

kT
kT
kT
2
2 2
m
m
m

kT
2

2
vx v y m , vx v y 0

43.

Correct answer is (2)


The entropy S of a thermodynamic system as a function of energy E is given by the following graph
S
C
B
A
E

The temperatures of the phases A, B and C, denoted by TA, TB and TC, respectively, satisfy the following
inequalities:
(1) TC > TB > TA
(2) TA > TC > TB
(3) TB > TC > TA (4) TB > TA > TC
Soln.

1 S

T E

S
S
S
E E E

A
E
B
TB TC TA
Correct answer is (3)
44.
The physical phenomenon that cannot be used for memory storage applications is
(1) large variation in magnetoresistance as a function of applied magnetic field
(2) variation in magnetization of a ferromagnet as a function of applied magnetic field
(3) variation in polarization of a ferroelectric as a function of applied electric field
(4) variation in resistance of a metal as a function of applied electric field
Soln. Variation in resistance of metal as a function of electric field cannot store the memory.
Correct answer is (4)

H.O.: 28-A/11, Jia Sarai, Near-IIT, New Delhi-16 , Ph : 011-26851008, 26861009 www.careerendeavour.com
B.O.: 48, First Floor, Mall Road, G.T.B. Nagar (Metro Gate No. 3), Delhi-09, Ph: 011-65462244, 9540292991

17
45.

Two identical Zener diodes are placed back to back in series and are connected to a variable DC power
supply. The best representation of the I-V characteristics of the circuit is
I

(1)

(2)

(3)

(4)

Soln. The best representation of the I-V characters of the circuit is (with DC supply)
I

Vz

Vz

Correct answer is (3)


46.

Part-C
A pendulum consists of a ring of mass M and radius R suspended by a massless rigid rod of length l
attached to its rim. When the pendulum oscillates in the plane of the ring, the time period of oscillation is
lR
g

(2)

2 2
(l R 2 )1/4
g

2 R 2 2 Rl l 2
(3) 2
g (R l )

(4)

2
(2 R 2 2 Rl l 2 )1/4
g

(1) 2

Soln. This is a compound pendulum

Here,

I
T 2
mg 0
I = M.I. about point of suspension
0 = distance of point of suspension from centre of gravity..

Here,

I I c md 2

I mR 2 m R

H.O.: 28-A/11, Jia Sarai, Near-IIT, New Delhi-16 , Ph : 011-26851008, 26861009 www.careerendeavour.com
B.O.: 48, First Floor, Mall Road, G.T.B. Nagar (Metro Gate No. 3), Delhi-09, Ph: 011-65462244, 9540292991

18

I mR 2 m R
mR 2 m R
T 2
mg R

47.

= 2

2 R 2 2 2 R
g R

Correct answer is (3)


Spherical particles of a given material of density p are released from rest inside a liquid medium of lower
density. The viscous drag force may be approximated by the Stokes law, i.e, Fd 6Rv , where is

the viscosity of the medium, R the radius of a particle and v its instantaneous velocity. If ( m) is the time
taken by a particle of mass m to reach half its terminal velocity, then the ratio (8m) / ( m) is
(1) 8
(2) 1/8
(3) 4
(4) 1/4
Soln. Each particle has same density but different radii and masses. We have to calculate time of fall in terms of
mass of particle therefore we will write our equations explicitly in terms of mass and we will remove radius
from our equations wherever it appears.
Drag force on particles
Fd
B
4 3
Fd 6Rv , Mass of a particle m . R
3
mg
1/3

1/3

1/3

3m
3m
3m
1/3
R
Fd 6

v Km v, where K 6

4
4
4
If B is buoyancy force, then equation of motion of a particle is
dv
4
4

m mg Fd B where B = R 3g R 3g . mg , = density of medium


dt
3
3

or


dv

dv
mg Km1/3v mg , m mg 1 Km1/3v
dt

dt


dv
g 1 Km 2/3v
dt

when terminal velocity is reached

... (a)
dv
0
dt


g 1

0 g 1 Km2/3vt vt 2/3
...(b)
Km

Now, if be the time to reach half the terminal velocity then from (a)
vt

Km2/3 vt

g 1
dv

2
1
dt ,
ln
2/3

Km

g 1 Km 2/3 v 0
g 1

using value of vt from (b) we get,

m 2/3
ln 2
K

or

(m)

m2/3 ln 2
K

(8m) / (m) (8m)2/3 / m 2/3 4


Correct answer is (3)

H.O.: 28-A/11, Jia Sarai, Near-IIT, New Delhi-16 , Ph : 011-26851008, 26861009 www.careerendeavour.com
B.O.: 48, First Floor, Mall Road, G.T.B. Nagar (Metro Gate No. 3), Delhi-09, Ph: 011-65462244, 9540292991

19
48.

A system of N classical non-interacting particles, each of mass m, is at a temperature T and is confined


by the external potential V (r )

1 2
Ar (where A is a constant) in three dimensions. The internal energy of
2

the system is
3
Nk BT
2

(1) 3 Nk BT

(2)

(3) N (2mA)3/ 2 k BT

(4) N

A k BT
ln

m m

Soln. The contribution of thermal energy with each quadratic term in hamiltonian is

The hamiltonian of a classical particle

49.

1
kT .
2

p2 1 2
Ar
2m 2

1
IN 3-D, hamiltonian has 6 quadratic term. So, the internal energy of a particle is 6 kT 3kT
2
The internal energy of a system of N particles = 3NkT.
Correct answer is
Consider a particle of mass m attached to two identical springs each of length l and spring constant k (see
the figure below). The equilibrium configuration is the one where the springs are unstretched. There are no
other external forces on the system. If the particle is given a small displacement along the x-axis, which of
the following describes the equation of motion for small oscillations?
y

(1) mx

kx 3
0
l2

(2) mx kx 0

(3) mx 2kx 0

(4) mx

kx 2
0
l

Soln.

F
2 x2

H.O.: 28-A/11, Jia Sarai, Near-IIT, New Delhi-16 , Ph : 011-26851008, 26861009 www.careerendeavour.com
B.O.: 48, First Floor, Mall Road, G.T.B. Nagar (Metro Gate No. 3), Delhi-09, Ph: 011-65462244, 9540292991

20
Let x be the displacement of particle along x-axis.
Therefore, net force on the particle along x axis is

2F cos
2 2

Where, F k elongation=k x

Therefore, using Newtons law we get Net force mx

2F cos mx

x
2k 2 x 2 .
mx

2 x 2

2kx 1 2 x 2

1/2

x2

mx
2kx 1 1 2

1/2

mx

Using binomial expansion {keep only upto x 2 term}


We get,

2kx.

mx

x2
22

kx3
2

mx

Correct answer is (a).


50.

If ( x ) A exp( x 4 ) is the eigenfunction of a one dimensional Hamiltonian with eigenvalue E = 0, the


potential V(x) (in units where 2m 1 ) is
(1) 12x 2
(2) 16x 6

(3) 16x6 + 12x2

(4) 16x6 12x2

4
Soln. Given : x A exp x is an eigenfunction of a 1-D hamiltonian with eigenvalue E = 0

From 1-D Schrodinger equation,

2 d 2
V 0
2m dx 2

2
Aexp x 4 16 x 6 12 x 2 V 0

2m

Taking 1, 2m 1

16 x6 12 x2 V 0

V x 16 x 6 12 x 2

Correct answer is (4)

H.O.: 28-A/11, Jia Sarai, Near-IIT, New Delhi-16 , Ph : 011-26851008, 26861009 www.careerendeavour.com
B.O.: 48, First Floor, Mall Road, G.T.B. Nagar (Metro Gate No. 3), Delhi-09, Ph: 011-65462244, 9540292991

21
51.

The electric field of an electromagnetic wave is given by E E0 cos[(0.3 x 0.4 y 1000t )]k . The

associated magnetic field B is


(1) 103 E0 cos[(0.3 x 0.4 y 1000t )]k

(2) 104 E0 cos[(0.3 x 0.4 y 1000t )](4i 3 j )

(3) E0 cos[(0.3 x 0.4 y 1000t )](0.3i 0.4 j )

(4) 102 E0 cos[(0.3 x 0.4 y 1000t )](3i 4 j )

Soln. Electric field of an e.m. wave is given by

E E0 cos 0.3 x 0.4 y 1000t k

Wave vector k is given by k 0.3i 0.4 j

k E 0.3i 0.4 j k E0 cos 0.3 x 0.4 y 1000t k


Hence, B

1000
3
= 10 0.3i 0.4 j E0 cos 0.3x 0.4 y 1000t k

52.

Correct answer is (2)


The energy of an electron in a band as a function of its wave vector k is given by
E ( k ) E0 B (cos k x a cos k y a cos k z a ) , where E0 , B and a are constants. The effective mass of the

electron near the bottom of the band is

2 2
(1)
3Ba 2
Soln.

2
(2)
3Ba 2

E k E0 B cos k x a cos k y a cos k z a

2
(3)
2Ba 2

2
(4)
Ba 2

Near the bottom of the band


k 2 a 2 k 2 a 2 k 2 a 2
2

y
1 z E0 B 3 a k x2 k y2 k z2
E k E0 B 1 x 1
2
2
2 =

a2 2
E k E0 3B B k
2

d 2E
dk

Ba 2

Effective mass, m

dE
Ba 2 k
dk

Ba 2

Correct answer is (4)


53.

A DC voltage V is applied across a Josephson junction between two superconductors with a phase
difference 0 . If I0 and k are constants that depend on the properties of the junction, the current flowing
through it has the form
2eVt

0
(1) I 0 sin

2eVt

0 (3) kV sin 0
(2) kV sin

(4) I 0 sin 0 kV

H.O.: 28-A/11, Jia Sarai, Near-IIT, New Delhi-16 , Ph : 011-26851008, 26861009 www.careerendeavour.com
B.O.: 48, First Floor, Mall Road, G.T.B. Nagar (Metro Gate No. 3), Delhi-09, Ph: 011-65462244, 9540292991

22
Soln. On the application of DC voltage. AC current flows across the junction and that is given by
2evt

I I 0 sin
0

Correct answer is (1)


54.

( 0 )
Consider the following ratios of the partial decay widths R1
and
( 0 )
( p )
R2
. If the effects of electromagnetic and weak interactions are neglected, then R1 and
( n)
R2 are, respectively,
(1) 1 and

(2) 1 and 2

(3) 2 and 1

(4) 1 and 1

55.

The intrinsic electric dipole moment of a nucleus ZA X

56.

(1) increases with Z, but independent of A


(2) decreases with Z, but independent of A
(3) is always zero
(4) increases with Z and A
According to the shell model, the total angular momentum (in units of ) and the parity of the ground state
of the 37 Li nucleus is
(1)

3
with negative parity
2

(2)

3
with positive parity
2

1
7
with positive parity
(4) with negative parity
2
2
A point charge q is placed symmetrically at a distance d from two perpendicularly placed grounded

(3)
57.

conducting infinite plates as shown in the figure. The net force on the charge (in units of 1/ 4 0 ) is
q

d
d

q2
(2 2 1) away from the corner
(1)
8d 2
(3)

q2

towards the corner

2 2d 2

q2
(2 2 1) towards the corner
(2)
8d 2
3q 2
(4)
away from the corner
8d 2

Soln. Force on charge +q will be attractive type

F1

F2

4 0 4d

q2

4 0 4d 2

F3
d
+q

F1
F2

d
x

y
+q

H.O.: 28-A/11, Jia Sarai, Near-IIT, New Delhi-16 , Ph : 011-26851008, 26861009 www.careerendeavour.com
B.O.: 48, First Floor, Mall Road, G.T.B. Nagar (Metro Gate No. 3), Delhi-09, Ph: 011-65462244, 9540292991

23

F3

q2

4 0 8d

x y

Net force, F F1 F2 F3

q2

4 0 8d 2

2 1

q2

4 0 8d 2
q2

4 0 8d 2

x
y

2 x 2 y 2 2

2 2 1
2 2 1

x
y
2
2

Towards the corner.

Correct answer is (2)


58.

Let four point charges q, q/2, q and q/2 be placed at the vertices of a square of side a. Let another point
charge q be placed at the centre of the square (see the figure).
q

q/2

q
q

q/2

Let V ( r ) be the electrostatic potential at a point P at a distance r >> a from the centre of the square. Then
V(2r) / V(r) is
1
1
1
(3)
(4)
2
4
8
Soln. Monopole and dipole moment of the charge configuration is zero. Hence, potential at large distances

(1) 1

(2)

r a various as V r

1
r

r3 1


Hence,
V r 8r 3 1 8
Correct answer is (d).
V 2r

59.

q/2

+q

3
q

+q

q/2

Let A and B be two vectors in three-dimensional Euclidean space. Under rotation, the tensor product
Tij Ai B j

(1)
(2)
(3)
(4)

reduces to a direct sum of three 3-dimensional representations


is an irreducible 9-dimensional representation
reduces to a direct sum of a 1-dimensional, a 3-dimensional and a 5-dimensional irreducible representations
reduces to a direct sum of a 1-dimensional and an 8-dimensional irreducible representation

H.O.: 28-A/11, Jia Sarai, Near-IIT, New Delhi-16 , Ph : 011-26851008, 26861009 www.careerendeavour.com
B.O.: 48, First Floor, Mall Road, G.T.B. Nagar (Metro Gate No. 3), Delhi-09, Ph: 011-65462244, 9540292991

24
60.

Fourier transform of the derivative of the Dirac -function, namely ( x ) , is proportional to


(1) 0
(2) 1
(3) sink
(4) ik

1
Soln. Fourier transform of ' x
2

ikx

' x e

dx

Using the property of Dirac delta function

f x ' x a dx f ' a

F ' x

So,

61.

1
ik
2

F ' x ik

Correct answer is (4)


A particle is in the ground state of an infinite square well potential given by,
0 for a x a
V ( x)
otherwise

The probability to find the particle in the interval between


(1)

1
2

(2)

1 1

(3)

a
a
and is
2
2

1 1

(4)

0 for a x a
Soln. Given : V ( x)
otherwise

Symmetrical infinite potential well of width 2a


Probability of finding the particle in the interval between
a /2

'1 1dx

a /2

1
a

a /2

a /2

x
2x
cos 2a dx = 2a 1 cos a dx
a /2
a /2
a /2

1
a
x

x sin

2a
a a /2
62.

a
a
to
is
2
2

1
2a 1 1
a

2a
2

Correct answer is (2)


The expectation value of the x-component of the orbital angular momentum Lx in the state
1
3 2,1, 1 5 2,1,0 11 2,1, 1 (where nlm are the eigenfunctions in usual notation), is
5

(1)

(3)

10
( 11 3)
25

10
( 11 3)
25

(2) 0

(4) 2

H.O.: 28-A/11, Jia Sarai, Near-IIT, New Delhi-16 , Ph : 011-26851008, 26861009 www.careerendeavour.com
B.O.: 48, First Floor, Mall Road, G.T.B. Nagar (Metro Gate No. 3), Delhi-09, Ph: 011-65462244, 9540292991

25
1
Soln. Given : 3 2,1,1 5 2,1,0 11 2,1,1
5

Lx Lx

1
3 2, 1, 1 5 2, 1, 0 11 2, 1, 1
5

1 L L

5
11
2,1,0
2,1,1
2, 1, 1
5
5
5

Now, we know,

L nm

m m 1 nm1

L nm

m m 1 nm1

5
11
Lx 2,1,1
2,1,0
2,1,1
5
5
5

1 3
5
5
11
2 2,1,1
2 2, 1, 1
2 2,1,0
2 2,1,0
2 5
5
5
5

Lx

3
1 5
11 1 3 5
11 1 5
. 2
2
. .
2
. .
2 5
5
5
2
5
5
5
2
5

3 5
11 5
10
2
2
25
25
25

11 3

Correct answer is (1)


63.

A particle is prepared in a simultaneous eigenstate of L2 and Lz. If ( 1) 2 and m are respectively the
eigenvalues of L2 and Lz, then the expectation value L2x of the particle in this state satisfies
(1)

2
(3) 0 Lx

Soln.

(2) 0 L2x 2 2

L2x 0

( 1) 2
3

(4)

2
( 1) 2
L2x
2
2

L2x L2y L2 L2z 1 2 m 2 2


1
2
2
2 2
then L x 1 m
2
For a given l, m varies from l to l.
2
2
Since, L x L y

L2x

L2x

max

min

occurs when m 0

L2x

occurs when m

L2x

max

min

1
1 2
2

1 2

Correct answer is (4)


H.O.: 28-A/11, Jia Sarai, Near-IIT, New Delhi-16 , Ph : 011-26851008, 26861009 www.careerendeavour.com
B.O.: 48, First Floor, Mall Road, G.T.B. Nagar (Metro Gate No. 3), Delhi-09, Ph: 011-65462244, 9540292991

26
64.

If the electrostatic potential V (r , , ) in a charge free region has the form


V ( r , , ) f ( r ) cos , then the functional form of f ( r ) (in the following a and b are constants) is
2
(1) ar

b
r

(2) ar

b
r2

(3) ar

b
r

r
(4) a ln
b

Soln. Potential is given by V r , , f r cos


Laplace equation is given by
2V r , , 0

1 v
1

v
r
2
sin
0
2 r r

r sin r
r

2
1
r f ' r cos
sin f r sin 0
r
sin

cos r 2 f " r 2rf ' r

2sin cos f r
0
sin

r 2 f " r 2rf ' r 2 f r 0

... (i)

The value of f(r) in option (b) satisfy the equation (i).


Correct answer is (2)
65.

, then the integral


jxz kxy
If A = iyz

A.dl

(where C is along the perimeter of a rectangular area

bounded by x = 0, x = a and y = 0, y = b) is
(1)
Soln.

1 3 3
(a b )
2

A yzi xzj xyk

(2) (ab 2 a 2b)

(c) (a 3 b3 )

(d) 0


A 0 i.e. field is conservative.

Since, line integral of a conservative field along a closed path is zero, then

A.d 0
C

Correct answer is (4)


66.

Consider an n n(n 1) matrix A, in which Aij is the product of the indices i and j (namely Aij = ij). The
matrix A
(1) has one degenerate eigenvalue with degeneracy (n 1)
(2) has two degenerate eigenvalues with degeneracies 2 and (n 2)
(3) has one degenerate eigenvalue with degeneracy n
(4) does not have any degenerate eigenvalue

1 2 3
Soln. Given : Aij ij . So, for a 33 matrix A 2 4 6
3 6 9

33

Since, the second row and third row is multiple of first row and the same is true for columns, then eigenvalue
of the matrix will be Tr(A), 0, 0 i.e. 14, 0, 0
0 is a degenerate eigenvalue with degeneracy 2
In general, matrix A (order n) has a degenerate eigenvalue with degeneracy n1.
Correct answer is (1)
H.O.: 28-A/11, Jia Sarai, Near-IIT, New Delhi-16 , Ph : 011-26851008, 26861009 www.careerendeavour.com
B.O.: 48, First Floor, Mall Road, G.T.B. Nagar (Metro Gate No. 3), Delhi-09, Ph: 011-65462244, 9540292991

27
67.

A child makes a random walk on a square lattice of lattice constant a taking a step in the north, east, south,
or west directions with probabilities 0.255, 0.255, 0.245, and 0.245, respectively. After a large number of
steps, N, the expected position of the child with respect to the starting point is at a distance
(1)

2 102 Na in the north-east direction

(2)

2 N 102 a in the north-east direction

(3) 2 2 10 2 Na in the south-east direction


(4) 0
Soln. Single step length = a
After a single step, the expected value of position of the child.

x 1 Pi xi
a

x 1 0.255 ai 0.255 aj 0.245 ai 0.245 aj

0.01a i j

After n steps, x

68.

P=0.225
N

i^ ^j

P=0.245
W

P=0.255
E

P=0.245
S

0.01 Na i j

Expected position x N 2 102 Na (NORTH-EAST DIRECTION)


Correct answer is (1)
A Carnot cycle operates as a heat engine between two bodies of equal heat capacity until their temperatures
become equal. If the initial temperatures of the bodies are T1 and T2, respectively, and T1 > T2 then their
common final temperature is
(1) T12 / T2

(2) T22 / T1

(3) T1T2

(4)

1
(T1 T2 )
2

Soln. Carnot cycle is reversible cycle so the entropy change must be zero.

S 0
Let the equilibrium temperature be T then
T dT
dT
C
0
T1 T
T2 T

S C

T
T
ln ln 0
T1
T2
T2
ln
0
T1T2

2
T T1T2 T T1T2

Correct answer is (3)

H.O.: 28-A/11, Jia Sarai, Near-IIT, New Delhi-16 , Ph : 011-26851008, 26861009 www.careerendeavour.com
B.O.: 48, First Floor, Mall Road, G.T.B. Nagar (Metro Gate No. 3), Delhi-09, Ph: 011-65462244, 9540292991

28
69.

Three sets of data A, B and C from an experiment, represented by , and


log scale. Each of these are fitted with straight lines as shown in the figure.

, are plotted on a log-

1000
100

10
1

A
0.1
0.1

10
x

100 1000

The functional dependence y(x) for the sets A, B and C are, respectively
(1)

x , x and x2

(3)

1
, x and x2
x2

Soln. Plot of Data A :

x
(2) , x and 2x
2

(4)

3

4

Plot of Data B :

1
, x and x2
x

m tan is between 0 and 1.

m tan 1
4


m tan is >1
4
2
Since, datas are plotted on log-log scale and all passes through (1, 1) then the straight lines will be of the
nature, log y m log x
Plot of Data C :

Correct answer is (4).


70.

A sample of Si has electron and hole mobilities of 0.13 and 0.05 m2/V-s respectively at 300K. It is doped
with P and Al with doping densities of 1.5 1021/m3 and 2.5 1021/m3 respectively. The conductivity of
the doped Si sample at 300K is

(1) 8 1m 1
(2) 32 1m 1
(3) 20.8 1m1
Soln. Si doped with P and Al.
As doping of Al > doping of P.
Hence, semiconductor will behave as p-type semiconductor.

(4) 83.2 1m 1

Majority carrier concentration 2.5 1021 1.5 1021 11021 /m3


The conductivity pe p 11021 1.6 1019 8 1m 1
Correct answer is (1)
71.

A 4-variable switching function is given by f (5, 7,8,10,13,15) d (0,1, 2) , where d is the do-not-carecondition. The minimized form of f in sum of products (SOP) form is

(1) AC BD
(2) AB CD
Soln. 4-variable switching function is given by

(3) AD BC

(4) BD BD

H.O.: 28-A/11, Jia Sarai, Near-IIT, New Delhi-16 , Ph : 011-26851008, 26861009 www.careerendeavour.com
B.O.: 48, First Floor, Mall Road, G.T.B. Nagar (Metro Gate No. 3), Delhi-09, Ph: 011-65462244, 9540292991

29

f A, B, C , D 5, 7,8,10,13,15 d 0,1, 2
CD
AB
00 01 11
00 X 0 X1 3
01 4
15 1 7
11

113

12

10 1 8

115
11

10
X2
6
14

110

There are two Quad.

72.

DB DB
Correct answer is (4)
A perturbation Vpert = aL2 is added to the Hydrogen atom potential. The shift in the energy level of the
2P state, when the effects of spin are neglected up to second order in a, is

(2) 2a 2 a 2 4

(1) 0

3 2 4
2
(4) a a
2

(3) 2a 2

Soln. Method-1:
The energy corrected to second order
0
1
2
E E E E

The shift in energy E E 0 E 1 E 2


where, E 0 = unperturbed energy
1
E = first order correction in energy
2
E = second order correction in energy

Perturbation,

V pert aL2

2p state of H-atom is 3-fold degenerate. Secular determinant to find first order correction in energy.
1
1, 1 aL2 1, 1 E

1, 0 aL2 1, 0

1, 1 aL2 1, 1

1, 0 aL2 1, 1

1
1, 0 aL2 1, 0 E

1, 0 aL2 1, 1

1, 1 aL2 1, 1

1, 1 aL2 1, 0

1
1, 1 aL2 1, 1 E

0
2 2 E

2 2

22

2 2

1
2 2 E

22

2 2

2 2

1
22 E

1
E 22

The second order correction in energy for degenerate perturbation theory includes matrix element (that has
summation over non-deg states). There is no non-deg states, so E 2 0
The shift in energy = 2 2
Correct answer is (3)

H.O.: 28-A/11, Jia Sarai, Near-IIT, New Delhi-16 , Ph : 011-26851008, 26861009 www.careerendeavour.com
B.O.: 48, First Floor, Mall Road, G.T.B. Nagar (Metro Gate No. 3), Delhi-09, Ph: 011-65462244, 9540292991

30
Method-2:
Since the enery correction to the all degenerate states are equal. Therefore, we can use the non-degenerate
perturbation theory to solve the problem as follows.
Given : V pert aL2 is added to a hydrogen atom potential.
First order correction to energy of the 2p state n 2, 1
1
En 2 P aL2 2 P a 1 2 2a 2

Second order correction to energy of the 2P state n 2, 1

2
En

n aL2 m

mn

73.

0
0
En Em

2 P aL m

0
0
E2 P Em

m 2 P

Therefore, the shift in the energy level En1 En 2 2a2


Correct answer is (3)
A gas laser cavity has been designed to operate at 0.5m with a cavity length of 1m. With this setup, the frequency is found to be larger than the desired frequency by 100 Hz. The change in the effective
length of the cavity required to retune the laser is
(1) 0.334 1012 m
(2) 0.334 1012 m
(3) 0.167 1012 m
(4) 0.167 1012 m

Soln. The separation between cavity mirros, d

m
where m is mode number..
2

0.5 m, d 1m

m 0.5 10 6
1
2

20
106
0.5

m 4 106

Charge in length of cavity, d m

But,

2
c

0.5
1012
3
Correct answer is (4)

m 2
d
2 c
d 0.167 1012 m

H.O.: 28-A/11, Jia Sarai, Near-IIT, New Delhi-16 , Ph : 011-26851008, 26861009 www.careerendeavour.com
B.O.: 48, First Floor, Mall Road, G.T.B. Nagar (Metro Gate No. 3), Delhi-09, Ph: 011-65462244, 9540292991

31
74.

The spectroscopic symbol for the ground state of 13Al is 2 P1/ 2 . Under the action of a strong magnetic field
(when L-S coupling can be neglected) the ground state energy level will split into
(1) 3 levels
(2) 4 levels
(3) 5 levels
(4) 6 levels
state

Soln.

P1/2

1, s

75.

ms

m 2ms

2, 0

1, 1

1 ,

0, 2

Out of the given 6 states, 2 states have same energy shift due to strong magnetic field. Therefore, the ground
state energy level will split into five levels.
Correct answer is (4)
A uniform linear monoatomic chain is modeled by a spring-mass system of masses m separated by nearest
neighbor distance a and spring constant m02 . The dispersion relation for this system is

ka
(1) (k ) 20 1 cos
2

2 ka
(2) (k ) 20 sin
2

ka
(3) (k ) 20 sin
2

ka
(3) (k ) 20 tan
2

Soln. Frequency of monoatomic one dimensional lattice is given by

4c
ka
sin
m
2

where c is the force constant (spring constant)


But c m02

4m02
ka
sin
m
2

ka
k 20 sin 2

Correct answer is (3).

H.O.: 28-A/11, Jia Sarai, Near-IIT, New Delhi-16 , Ph : 011-26851008, 26861009 www.careerendeavour.com
B.O.: 48, First Floor, Mall Road, G.T.B. Nagar (Metro Gate No. 3), Delhi-09, Ph: 011-65462244, 9540292991

UV PHYSICS ACADEMY
Shivam Road, New Nallakunta, Hyderabad, Ph: 04032458147, 09885072826
www.uvphysics.com
CSIR JUNE-2013
PART-A ANSWER ANY 15 QUESTIONS, EACH QUESTION CARRIES 2 MARKS
Q1. There is an equilateral triangle in the XY plane with its centre at the origin. The distance of its sides from
the origin is 3.5cm. The area of its circumcircle in cm2 is
a) 38.5
b) 49
c) 63.65
d) 154
Ans: a
Q2. A sphere of iron of radius R/2 fixed to one end of a string was lowered into water in a cylindrical container
of base radius R to keep exactly half the sphere dipped. The rise in the level of water in the container will be

a) R/3
b) R/4
c) R8
d) R/12
Ans: d
Q3. A crystal grows by stacking of unit cells of 10 x 20 x 5nm size as shown in the diagram given below. How
many unit cells will make a crystal of 1cm3 volume?

a) 106
Ans: d

b) 109

Q4. What is the value of

c) 1012

d) 1018

1
1
1

......to ?
1 2 2 3 3 4
b) 1
d) 2

a) 2/3
d)
Ans: b
Q5. A solid cylinder of basal area A was held dipped in water in a cylindrical vessel of basal area 2A vertically
such that a length h of the cylinder is immersed. The lower tip of the cylinder is at a height h from the base
of the vessel. What will be the height of water in the vessel when the cylinder is taken out?

a) 2h

b)

3
h
2

4
c) h
3

5
d) h
4

Ans: b
Q6. Of all the triangles that can be inscribed in a semicircle of radius R with the diameter as one side, the
biggest one has the area
a) R2
b) R 2 2
c) R 2 3
d) 2R2
Ans: a
Q7. Choose the largest number:
a) 2500
b) 3400
Ans: b

c) 4300

d) 5200

UV PHYSICS ACADEMY
Shivam Road, New Nallakunta, Hyderabad, Ph: 04032458147, 09885072826
www.uvphysics.com
Q8. A daily sheet calendar of the year 2013 contains sheets of 10 x 10 cm size. All the sheets of the calendar are
spread over the floor of a room of 5m 7.3m size. What percentage of the floor will be covered by these
sheets?
a) 0.1
b) 1
c) 10
d) 100
Ans: c
Q9. How many rectangles (which are not squares) are there in the following figure?

a) 56
b) 70
c) 86
d) 100
Ans: b
Q10. Define a b 1cm(a, b) gcd( a, b) and a b a b b a . What is the value of 1 2 3 4? Here LCM
= least common multiple and gcd = greatest common divisor.
a) 145
b) 286
c) 436
d) 572
Ans: c
Q11. A square pyramid is to be made using a wire such that only one strand of wire is used for each edge. What
is the minimum number of times that the wire has to be cut in order to make the pyramid?
a) 3
b) 7
c) 2
d) 1
Ans: d
Q12. A crow is flying along a horizontal circle of radius R at a height R above the horizontal ground. Each of a
number of men on the ground found that the angular height of the crow was a fixed angle 450 when it
was closest to him. Then all these men must be on a circle on the ground with a radius
a) R R sin
b) R R cos
c) R R tan
d) R R cot
Ans: d
Q13. How many pairs of positive integers have gcd 20 and 1cm 600? (gcd = greatest common divisor; 1cm =
least common multiple)
a) 4
b) 0
c) 1
d) 7
Ans: d
Q14. During an evening party, when Ms. Black, Ms. Brown and Ms. White met, Ms. Brown remarked, It is
interesting that our dresses are white, black or brown, but for each of us the name does not match the colour
of the dress!. Ms. White replied, But your white dress does not suit you!. Pick the correct answer.
a) Ms Whites dress was brown.
b) Ms. Blacks dress was white.
c) Ms. Whites dress was black
d) Ms. Blacks dress was black
Ans: c
Q15. Two integers are picked at random from the first 15 positive integers without replacement. What is the
probability that the sum of the two numbers is 20?
a)
b) 1/21
c) 1/105
d) 1/20
Ans: b

UV PHYSICS ACADEMY
Shivam Road, New Nallakunta, Hyderabad, Ph: 04032458147, 09885072826
www.uvphysics.com
Q16. Identify the next figure in the sequence?

a)
b)
c)
d)
Ans: a
Q17. In a customer survey conducted during Monday to Friday, of the customers who asked for child care
facilites in super markets, 23% were men and the rest, women. Among them, 19.9% of the women and 8.8%
of the men were willing to pay for the facilities.
A. What is the ratio of the men to women customers who wanted child care facilities?
B. If the survey had been conducted during the weekend instead, how will the result change?
With the above data,
a) Only A can be answered
b) Only B can be answered
c) Both A and B can be answered
d) Neither A nor B can be answered
Ans: a
Q18. The map given below shows contour lines which connect points of equal ground surface elevation in a
region. Inverted V shaped portions of contour lines represent a valley along which a river flows. What is
the downstream direction of the river?

a) North
b) South
c) East
d) West
Ans: b
Q19. During a summer vacation, of 20 friends from a hostel, each wrote a letter to each of all others. The total
number of letters written was
a) 20
b) 400
c) 200
d) 380
Ans: d
Q20. A person has to cross a square field by going from A to C. The person is only allowed to move towards
the east or towards the north or use a combination of these movements. The total distance travelled by the
person

a) depends on the length of each step


c) is different for different paths
Ans: d

b) depends on the total number of steps


d) is the same for all paths

UV PHYSICS ACADEMY
Shivam Road, New Nallakunta, Hyderabad, Ph: 04032458147, 09885072826
www.uvphysics.com
1
m 2 x 2 . The bosons
2
interact via a weak potential, V12 V0 exp m( x1 x2 ) 2 / 4 where x1 and x2 denote coordinates of the

Q21. Two identical bosons of mass m are placed in a one-dimensional potential V ( x)

particles. Given that the ground state wave function of harmonic oscillator is 0 ( x)

1/ 4

mx 2
2

The ground state energy of the two-boson system, to the first order in V0, is:
a) 2V0

b)

V0

c) V0 1

1 / 2


d) V0 1

Ans: c
mx 2

mx 2

1
2
m
Exp: Perturbed potential V12 V0 exp m( x1 x2 ) / 4 ; 0 ( x1 , x2 )
e 2 e 2 ;

m( x1 x2 ) 2 1 m 2 2 ( x1 x2 ) 4

...
E (1) H 1 V12 Taylor series of V12 V0 1
2
4
2
16

1/ 2

m x12 x22 2 x1
m( x1 2 x2 2 2 x1 x2 )

V0 1
.... ; V12 V0 1
4

x2

... ;

1 / 2
m 2m 2m 0

... V 1 ; V V 1
V0 1
12

0
0
4

4
2

Q22. Ten grams of ice at 0oC is added to a beaker containing 30grams of water at 25oC. What is the final
temperature of the system when it comes to thermal equilibrium? (The specific heat of water is 1cal/gm/oC
and latent heat of melting of ice is 80 cal/gm.)
a) 0oC
b) 7.5oC
c) 12.5oC
d) -1.25oC
Ans: a
Exp: The amount of Heat required to melt the ice of mass 10gm at 0oC is Q = mL = 800cal.
L 80cal / gm The amount of Heat available in water of mass 30gm at 25oC is Q = mST = 30 x 1 x 25 =
750cal. Since the heat available is less than the heat required to melt the ice therefore ice will not melt as a
result, the temperature of the system will be at 0oC only.

UV PHYSICS ACADEMY
Shivam Road, New Nallakunta, Hyderabad, Ph: 04032458147, 09885072826
www.uvphysics.com
Q23. A vessel has two compartments of volume V1 and V2, containing an ideal gas at pressures p1 and p2, and
temperatures T1 and T2 respectively. If the wall separating the compartments is removed, the resulting
equilibrium temperature will be
p T p 2T2
V T V2T2
p1V1 p 2V2
1/ 2
a) 1 1
b) 1 1
c)
d) T1T2
p1 p 2
V1 V2
( p1V1 / T1 ) ( p 2V2 / T2 )
Ans: c
pV
pV
Exp: n n1 n2 1 1 2 2 ;U U 1 U 2 n1cV T1 n2 cV T2 ncvT
T1
T2
p V p 2V2
p V p 2V2
nT n1T1 n2T2 T 1 1
1 1
p1V1 p 2V2
n

T1
T2
Q24. For temperature T1 > T2, the qualitative temperature dependence of the probability distribution F(v) of the
speed v of a molecule in three dimensions is correctly represented by the following figure:

a)
Ans: a

b)
3/ 2

c)

d)

mv 2

m
2
Exp: F (v)dv 4N
e 2 kT v dv . As temperature Increases, mean velocity shift towards right.
2

kT

Q25. A system of non-interacting spin-1/2 charged particles are placed in an external magnetic field. At low
temperature T, the leading behavior of the excess energy above the ground state energy, depends on T as: (c
is a constant)
a) cT
b) cT3
c) e-c/T d) c ( is independent of T)
Ans: c
x
H
x

H
Exp: U ln z B H tanh B B H e x e x , where x B ; excess energy from the ground level
kT

kT
e e
e x ex
B H x
x
e e

2e x
2e kT

( B H ) B H x
B H B H
x
H
B

e e
e kT e kT

ve c / T , where c B H .

Q26. The acceleration due to gravity g is determined by measuring the time period T and the length L of a
simple pendulum. If the uncertainties in the measurements of T and L are T and L respectively the,
fractional error g / g in measuring g is best approximated by
| L | | T |

L
T
Ans: d

a)

b)

| L | | 2T |

c)
L
T

| L | 2 | T | 2

d)
L
T

L
4 2 L g
L 2T
g
;

Exp: T 2

2
g
g
T
L T
2

| L | | 2T |

L T

UV PHYSICS ACADEMY
Shivam Road, New Nallakunta, Hyderabad, Ph: 04032458147, 09885072826
www.uvphysics.com
Q27. A silicon transistor with built-in voltage 0.7V is used in the circuit shown, with VBB=9.7V, RB = 300k,
VCC = 12V and RC = 2k. Which of the following figures correctly represents the load line and the
quiescent Q point?

a)
Ans: b

b)

c)

d)

VCC VBE
V
9.7 0.7
12

30A; I C , sat CC
6mA
3
RB
RC
300 10
2 10 3
Q28. If the analog input to an 8-bit successive approximation ADC is increased from 1.0 V to 2.0V, then the
conversion time will
a) remain unchanged b) double c) decrease to half its original value d) increase four times
Ans: a
Q29. The insulation resistance R of an insulated cable is measured by connecting it in parallel with a capacitor
C, a voltmeter, and battery B as shown. The voltage across the cable dropped from 150V to 15V, 1000
seconds after the switch S is closed. If the capacitance of the cable is 5F then its insulation resistance is
approximately
Exp: I B

a) 10 9
Ans: b

b) 108

c) 10 7

d) 10 6

q 75c
V
15

75nA; R
2 108 108
T 1000
I 75nA
Q30. The approximation cos 1is valid up to 3 decimal places as long as is less

Exp: q cV 75c; I

than: (take 180 o / 57.29 o )


a) 1.28o
b) 1.81o
Ans: b
Exp: cos 1

2
2

2
4

..... 1

c) 3.28o

2
2

; If 1.810 0.0314 cos 1

d) 4.01o

2
2

1 (0.00049) 1

UV PHYSICS ACADEMY
Shivam Road, New Nallakunta, Hyderabad, Ph: 04032458147, 09885072826
www.uvphysics.com
Q31. The area of a disc in its rest frame S is equal to 1 (in some units). The disc will appear distorted to an
observer O moving with a speed u with respect to S along the plane of the disc. The area of the disc
measured in the rest frame of the observer O is (c is the speed of light in vacuum)
u2
a) 1 2
c
Ans: a

1/ 2

u2
b) 1 2
c

1 / 2

u2
c) 1 2
c

u2
d) 1 2
c

u2
Exp: Area of the disc wrt s frame A0 a ; Area of the disc wrt s A aa a ; A A 1 2 Since

c
2

u2
c2
Q32. A particle of charge e and mass m is located at the midpoint of the line joining two mixed collinear dipoles
with unit charges as shown in the figure. (The particle is constrained to move only along the line joining the
dipoles.) Assuming that the length of the dipoles is much shorter than their separation, the natural frequency
of oscillation of the particle is
A = 1 A1 1

a)

6e 2 R 2
0 md 5

b)

6e 2 R
0 md 4

c)

6e 2 d 2
0 mR 5

d)

6e 2 d
0 mR 4

Ans: d
Exp: E 2 p

1
1
6d
6e 2 d

;
E

x
(

2
ed
);
F

eE

x;

4 0 ( R x) 3 ( R x) 3
0 R 4
0 R 4

6e 2 d
6e 2 d
6e 2 d
2
x
;

m 0 R 4
m 0 R 4
m 0 R 4

Q33. A current I is created by a narrow beam of protons moving in vacuum with constant velocity u . The
direction and magnitude, respectively, of the Poynting vector S outside the beam at a radial distance r (much
larger than the width of the beam) from the axis, are

I2
I2
a) S u and | S |
b) S || (u ) and | S |

4 2 0 | u | r 2
4 2 0 | u | r 4

I2
I2
c) S || u and | S |
d) S || u and | S |

4 2 0 | u | r 2
4 2 0 | u | r 4
Ans: c

I
Exp: I u, where is the charge per unit length. B (at a distance r)= 0 ;
2r

2

I
EB
I
E
r
r; S

z
2
2 0 r
2 0 ur
0
4 0 ur 2
a

UV PHYSICS ACADEMY
Shivam Road, New Nallakunta, Hyderabad, Ph: 04032458147, 09885072826
www.uvphysics.com

Q34. If the electric and magnetic fields are unchanged when the vector potential A changes (in suitable units)

according to A A r, where r r (r )r, then the scalar potential must simultaneously change to
a) r
b) r
c) r / t
d) r / t
Ans: c

Exp: Under Gauge transformation A1 A and 1


where is Gauge function Given,
t

r
A1 A r r r (t ) 1
t
2
r0 r / r0
Q35. Consider an axially symmetric static charge distribution of the form, 0 e
cos 2 . The radial
r
component of the dipole moment due to this charge distribution is
a) 2 0 r04
b) 0 r04
c) 0 r04
d) 0 r04 / 2
Ans: a
2
2
1
2

r0 r / r0
2
1
1
1
1 r0
Exp: 0 e
cos . ;Dipolemoment p r (r )d 0 r 1 e r / r0 cos 2 r 1 dr 1 sin dd
r
r

2
1

1!
1
2 (2 ); p 2 0 r04 r
0 r02 r 1e r / r0 dr 1 sin d cos 2 d 0 r02
2
2
(1 / r0 )
0
0
0
Q36. In a basis in which the z-component Sz of the spin is diagonal, an electron is in a spin state
(1 i ) / 6
. The probabilities that a measurement of Sz will yield the values / 2 and / 2 are,

2/3

respectively.
a) and
b) 2/3 and 1/3
c) and
d) 1/3 and 2/3
Ans: d
1 i
6
Exp: (1 i) / 6 1 i 1 2 0 ; 1 i z 2 z ; P
( / 2)

2
2/3

3 1
3
6 0
6
1 i

6
2

2 / 6; P( / 2)

2
3
1 i
6

2
3

2
3

2
3

UV PHYSICS ACADEMY
Shivam Road, New Nallakunta, Hyderabad, Ph: 04032458147, 09885072826
www.uvphysics.com
Q37. Consider the normalized state of a particle in a one-dimensional harmonic oscillator:

b1 0 b2 1 where 0 and 1 denote the ground and first excited states respectively, and b1 and b2 are
real constants. The expectation value of the displacement x in the state will be a minimum when
a) b2 0, (b1 1)

b) b2

1
2

c) b2

b1

1
b1
2

d) b2 = b1

Ans: d
Exp: b1 0 b2 1 ; x x b12 0 x 0 b22 1 x 1 2b1b2 0 x 1 ; 0 x 0 1 x 1 0;

x 2b1b2 0 x 1 ; x 1 (b1 b2 ) 2

0 x 1 ( b12 b22 1); x

min

b1 b2

Q38. A muon from cosmic rays is trapped by a proton to form a hydrogen-like atom. Given that a muon is
approximately 200 times heavier than an electron, the longest wavelength of the spectral line (in the
analogue of the Lyman series) of such an atom will be
a) 5.62Ao
b) 6.67Ao
c) 3.75Ao
d) 13.3Ao
Ans: b
m m p
E

Exp: Muonic Atom E n1 E n where


180me E n1 21 180 ; for n = 1 E11 180E1 ;
m m p
n
m
hc
hc
for n = 2 E21 45E1 E hv

6.67 Ao

E
Q39. The un-normalized wave function of a particle in a spherically symmetric potential is given by

(r ) zf (r ) where f (r ) is a function of the radial variable r. The eigen value of the operator L2 (namely the
square of the orbital angular momentum) is
a) 2 / 4
b) 2 / 2
c) 2
d) 2 2
Ans: d
Exp: (r ) zf ( f ) r cos f (r ) ;here l=1 (l= value is given by the power of cos ) ; Eigen value of
L2 2 l (l 1) 2 2

Q40. Given that H n ( x)


n 0

a) 12
Ans: a

2
tn
e t 2tx the value of H4(0) is
n!
b) 6
c) 24

d) -6

tn
tn
t4 t6
t4 t4
4!
t 2 2tx
t 2
2
H n (0) e 1 t .... H 4 (0) H 4 (0) 12
Exp: H n ( x) e
n!
n!
2! 3!
4! 2!
2!
n 0
n 0

UV PHYSICS ACADEMY
Shivam Road, New Nallakunta, Hyderabad, Ph: 04032458147, 09885072826
www.uvphysics.com
Q41. A unit vector n on the xy-plane is at an angle of 120o with respect to i. The angle between the

vectors u ai bn and v an bi will be 60o if


a) b 3a / 2
b) b 2a / 3
c) b a / 2
d) b = a
Ans: c

Exp: The angle between u and v is 60o i.e u .v | v || v}cos 0 0 ; a 2 i.n b 2 n.i ab ba | u || v | cos 60 o
| u || v |
;
a 2 cos 120 o 2ab b 2 cos 120 o
2
a2 b2
2
1
2ab a 2 b 2 2ab cos 120 ;(a 2 b 2 ) 4ab a 2 b 2 ab

2
2

a 2 b 2 5ab b a / 2
Q42. With z x iy , which of the following functions f ( x, y) is NOT a (complex) analytic function of z?

a) f ( x, y) x iy 8 4 x 2 y 2 2ixy
3

b) f ( x, y) x iy 1 x iy

c) f x, y x 2 y 2 2ixy 3
d) f x, y 1 x iy 2 x iy
Ans: d
4
6
Exp: From option (c) f ( x, y) (1 x iy ) 4 (2 x iy ) 6 ; f ( x, y) 1 ( x iy ) 2 ( x iy ) ;
5

f ( x, y) (1 z ) 4 (2 z ) 6 due to presence of z , f ( x, y) is not analytic.


Q43. A planet of mass m and an angular momentum L moves in a circular orbit in a potential, V(r) = - k/r,
where k is a constant. If it is slightly perturbed radially, the angular frequency of radial oscillations is
a) mk 2 / 2L3
b) mk 2 / L3
c) 2mk 2 / L3
d) 3mk 2 / L3
Ans: b
Veff
L2
k
L2 k
L2

0
Exp: Veff
for
circular
orbit
(where k is constant)

0
r
mk
2mr 2 r
mr 3 r 2

2V
r 2
m

2V
r 2

r r0

3L2 2k

mr 4 r 3

L2
mk

m3k 4
mk 2

L6
L3

r r0

1 2
mx bx where b is a
2
positive constant. The coordinate of the particle x(t) at time t is given by: (in the following c1 and c2 are
constants)
b 2
bt
bt
bt
bt
a)
t c1t c2 b) c1t c2 c) c1 cos c2 sin d) c1 cosh c2 sinh
2m
m
m
m
m
Ans: a
1
d L L
d
Exp: L mx 2 bx Equation of motion
0 (mx ) b 0; mx b

2
dt x x
dt
Q44. The Lagrangian of a particle of mass m moving in one dimension is given by L

d 2x
b
dx
b
b 2

t c1 ; x(t )
t c1t c2
2
m
dt
m
2m
dt

UV PHYSICS ACADEMY
Shivam Road, New Nallakunta, Hyderabad, Ph: 04032458147, 09885072826
www.uvphysics.com
Q45. A uniform cylinder of radius r and length , and a uniform sphere of radius R are released on an inclined
plane when their centres of mass are at the same height. If they roll down without slipping, and if the sphere
reaches the bottom of the plane with a speed V, then the speed of the cylinder when it reaches the bottom is:
14r
r
14
4V
a) V
b) 4V
c)
d) V
2
15R
15
15R
15
Ans: d
g sin
mr 2
2
Exp: Acceleration a
for solid cylinder I
, for solid sphere I mr 2 ;
I
5
2
1
2
mr
v
2 g sin
5 g sin vc ac
14
14
ac
, as
;
( v 2 2as) c
vc v
3
7
vs
15
15
vs as
Q46. The components of a vector potential A A0 , A1 , A2 , A3 are given by A k ( xyz , yzt , zxt , xyt ) where k
is a constant. The three components of the electric field are
a) k yz , zx, xy
b) k x, y, z
c) (0, 0, 0)
d) k xt , yt , zt
Ans: c

Exp: A k ( xyz , yzt , zxt , xyt ) ( , Ax , Ay , Az ); k ( yzx xzy xyz);

A
A
k ( yzx zxy xyz); E
0
t
t
Q47. In the Born approximation, the scattering amplitude f ( ) for the Yukawa potential is given by:(in the
following b 2k sin / 2, E 2 k 2 / 2m)
2m
2m
2m
2m
a)
b) 2 2
c)
d)
3
2
2
2
2 2
2
2
2
2
b
b
2 b2
b

Ans: b

2m
2m b
where
k
=
b
=
2
k
sin
;
r
sin
krV
(
r
)
dr
f
(

r sin brV (r )dr


2
2k 0
2b 0
b
m b
1
2m
m 1
2 e ibr r dr e ibr r dr 2

2 2

2
i b 0
0
i b ib ib ( b )
Q48. If nlm denotes the eigen function of the Hamiltonian with a potential V = V(r) then the expectation value
1
of the operator L2x L2y in the state 3 211 210 15 211 is
5
2
2
a) 39 / 25
b) 13 / 25
c) 2 2
d) 26 2 / 25
Ans: d
1
Exp: 3 211 210 15 211 ; L2x L2y L2z L2 L2x L2y L2 L2z ;
5
2
Lx L2y L2 L2z ; L2 L2z s p s , where 2 l (l 1) m 2
Exp: f ( )

2 (2 1)

9
1
15 2
26 2
2 (2) 2 (2 1)

(9 2 15)
25
25
25 25
25

UV PHYSICS ACADEMY
Shivam Road, New Nallakunta, Hyderabad, Ph: 04032458147, 09885072826
www.uvphysics.com
Q49. An oscillating current I (t ) I 0 exp it flows in the direction of the y-axis through a thin metal sheet of
area 1.0cm2 kept in the xy-plane. The rate of total energy radiated per unit area from the surfaces of the
metal sheet at a distance of 100m is
a) I 0 / 12 0 c 3
b) I 02 2 / 12 0 c 3
c) I 02 / 12 0 c 3
d) I 0 2 / 24 0 c 3
Ans: b

Exp: I (t ) I 0 e

it

power radiated per unit area

0 p 2
6c

where p is the dipole moment per unit length

0 I 02 2 e 2it
0 I 02 2
I 02 2
dp
it
it

p
I 0e
p I 0 (i )e
Power radiated
6c
12c
dt
12 0 c 3
0 0 x a, 0 y a
Q50. Consider a two-dimensional infinite square well V ( x, y )
. Its normalized eigen
otherwise
function are nx ,n y x, y

2 n xx n y y
where n x , n y 1,2,3,...... If a
sin
sin
a a a

a
a

V0 0 x , 0 y
perturbation H
2
2 is applied, then the correction to the energy of the first excited

0 otherwise
state to order V0 is
V
V
V
V
64
16
32
a) 0
b) 0 1 2
c) 0 1 2
d) 0 1 2
4
4 9
4 9
4 9
Ans: b
2 x 2y
Exp: First excited state is doubly degenerate. 1 (1,2) sin sin
;
a a a

V
2 2x y
2 a / 2 2 x a / 2 2 2x
dx sin
dy 0
2 (2,1) sin
sin H 11 1 H 1 V0 sin
a
a
4
a a a
a 0
0
2

V
2x a / 2
2y
y
2 a / 2 x
H 22 2 H 2 0 ; H 12 1 H 2 V0 sin sin
dx sin
sin dy
4
a
a
a
a
a 0
0
V0
16V0

H 11 H 12
16
4
9 2 0

;
Diagonalisation
of
Eigen
values
H 12 V0
H

16V0 V0
9 2
21 H 22

4
9 2
2

V
64
V
16V0
0
0 0 1 2
2
4 9
4
9

UV PHYSICS ACADEMY
Shivam Road, New Nallakunta, Hyderabad, Ph: 04032458147, 09885072826
www.uvphysics.com
Q51. Consider a system of two Ising spins S1 and S2 taking values 1 with interaction energy given
by JS1 S 2 , when it is in thermal equilibrium at temperature T. For large T, the average energy of the
system varies as C/kBT, with C given by
a) 2J 2
b) J 2
c) J 2
d) 4 J
Ans: b
s1 s 2
1 1 J
e J e J

Exp: Ising model 1 1 J z 2 e J e J ; U ln z J J


;
p
e e J

1 1 J

1 1 J
1 J (1 J )
2
J
;U J ( J ) J / kT
1

Q52. Consider three particles A, B and C, each with an attribute S that can take two values 1.
Let S A 1, S B 1and S C 1 at a given instant. In the next instant, each S value can change to S with
probability 1/3. The probability that SA + SB + SC remains unchanged is
a) 2/3
b) 1/3
c) 2/9
d) 4/9
Ans: d
Exp:
SA SB
SC
SA + SB + SC
1
1
-1
1
-1
1
1
1
1
-1
1
1
2 2 2
1 2 1
2 1 1 4
At a given instant SA = 1 SB= 1, SC= - 1 p ( ) ( ) (
3 3 3
3 3 3
3 3 3) 9
Q53. The bound on the ground state energy of the Hamiltonian with an attractive delta-function potential,
2 d 2
( x) using the variational principle with the trial wave
namely H
2m dx 2

function ( x) A exp bx 2 is [Note: e t t a dt a 1]


0

a) m / 4
Ans: c
2

b) m / 2
2

2
d2
b 2

; ( x)
2m
2m
dx 2
E
2m 2 2
m 2
0b
;
E

minimization,
min
b
2
2

Exp: 1 A 2

2b

d) m 2 / 5 2

c) m 2 / 2

2b

; H

b 2
2b

for
2m

UV PHYSICS ACADEMY
Shivam Road, New Nallakunta, Hyderabad, Ph: 04032458147, 09885072826
www.uvphysics.com
Q54. Consider two different systems each with three identical non-interacting particles. Both have single
particle states with energies 0 ,3 0 and 5 0 , ( 0 0). One system is populated by spin-1/2 fermions and the
other by bosons. What is the value of E F E B where EF and EB are the ground state energies of the
fermionic and bosonic systems respectively?
a) 6 0
b) 2 0
c) 4 0
d) 0
Ans: b

3 0
3 0
Exp: Fermion system
; Boson system
E 4 0 ; E 2 0 E F E B 2 0

Q55. The input to a lock-in amplifier has the form Vi (t ) Vi sint i where Vi , , i the amplitude, frequency
and phase of the input signal respectively. This signal is multiplied by a reference signal of the same
frequency , amplitude Vr and phase r . If the multiplied signal is fed to a low pass filter or cut-off
frequency , then the final output signal is

1
a) ViVr cos i r
2
c) ViVr sin i r

b) ViV r cos i r cos t i r


2

d) ViVr cos i r cos t i r


2

Ans: a
Exp: V Vr sin(t r ) Vi sin(t i )
filter

ViVr
cos( i r ) cos(2t i r ) output of low pass
2

ViVr
cos( i r )
2

2
2
Q56. The solution of the partial differential equation 2 u ( x, t ) 2 u ( x, t ) 0 satisfying the boundary conditions
t
x
u(0,t) = 0 = u(L, t) and initial conditions u( x,0) sinx / L and

u ( x, t ) t 0 sin 2x / L is
t

a) sin x / L cos t / L

L
sin 2x / L cos 2 t / L
2
b) 2 sinx / Lcos t / L sinx / Lcos2 t / L
L
c) sin x / L cos2 t / L
sin 2x / L sin t / L
2
L
sin 2x / L sin 2 t / L
d) sin x / L cost / L
2
Ans: d

Exp: Consider the option (d), which satisfying. u (0, t ) 0 and u ( L, t ) 0; u ( x,0) sin

U
t

0
t 0

L
2x 2t 2 U
sin
;
cos

2
L L L t

sin
t 0

2x
L

x
L

UV PHYSICS ACADEMY
Shivam Road, New Nallakunta, Hyderabad, Ph: 04032458147, 09885072826
www.uvphysics.com
Q57. Consider the hydrogen-deuterium molecule HD. If the mean distance between the two atoms is 0.08nm
and the mass of the hydrogen atom is 938MeV/c2, then the energy difference E between the two lowest
rotational states is approximately
a) 10-1eV
b) 10-2eV
c) 2 x 10-2eV
d) 10-3eV
Ans: b
2
2
2
Exp: E ( J )
J 1 J 0 I r 2 mH mD r 2 2 mH r 2
J ( J 1); E
mH mD
3
2I
I
I

2
2
9 2
10 2 eV
I mH (0.08 10 ) ; E
I
3
Q58. Four digital outputs V, P, T and H monitor the speed v, tyre pressure p, temperature t and relative
humidity h of a car. These outputs switch from 0 to 1 when the values of the parameters exceed 85km/hr, 2
bar, 40oC and 50%, respectively. A logic circuit that is used to switch ON a lamp at the output E is shown
below. Which of the following conditions will switch the lamp ON?

a) v 85km / hr , p 2bar , t 40 o C, h 50%


b) v 85km / hr , p 2bar , t 40 o C, h 50%
c) v 85km / hr, p 2bar , t 40 o C, h 50% d) v 85km / hr, p 2bar , t 40 o C, h 50%
Ans: a
Exp: Output E V P T H ; from option (a) v < 85km/hr, p < 2 bar, t > 400C, h > 50%.
E 1 1 0 0 0 1 1 . So lamp will switch ON.
dx
Q59. The solution of the differential equation
x 2 with the initial condition x(0) = 1 will blow up as t tends
dt
to
a) 1
b) 2
c)
d)
Ans: a
dx
dx
1
1
1
Exp:
as t 1, x
x 2 , x(0) 1 2 dt t c since x(0) 1 c 1
t 1 x
dt
x
x
1 t
x
blows up.
Q60. Let u be a random variable uniformly distributed in the interval [0, 1] and V c ln u, where c is a real
constant. If V is to be exponentially distributed in the interval [0, ) with unit standard deviation, then the
value of c should be
a) ln 2
b)
c) 1
d) -1
Ans: c

UV PHYSICS ACADEMY
Shivam Road, New Nallakunta, Hyderabad, Ph: 04032458147, 09885072826
www.uvphysics.com
Q61. The inverse Laplace transform of

1
a) t 2 e t
2
Ans: c

1
is
s s 1
2

1
b) t 2 1 e t
2

1
d) t 2 1 e t
2

c) t 1 e t

t
1 t t
1
1
1
t
t
t
L
f (t ) e t ; L1

e
dt

1
);

2
(e 1)dt e t 1

s 1
s( s 1) 0
s ( s 1) 0
Q62. The number of degrees of freedom of rigid body in d-space-dimensions is
a) 2d
b) 6
c) d (d + 1)/2
d) d!
Ans: c
d (d 1)
Exp: Degree of freedom
, where d=dimension of space.
2
Q63. A particle of mass m is at the stable equilibrium position of its potential energy V ( x) ax bx 3 where a, b
are positive constants. The minimum velocity that has to be imparted to particle to render its motion
unstable is
Exp: f ( s)

a) 64a 3 / 9m 2 b
Ans: c

1/ 4

Exp: V ( x) ax bx 3 ;

b) 64a 3 / 27m 2 b

1/ 4

c) 16a 3 / 27m 2 b

1/ 4

d) 3a 3 / 64m 2 b

1/ 4

1
V
2V
0 x 2 a / 3b and 2 6bx; minimum velocity mv 2 V ( x);
x
2
x
16a 3
16a 3


27m 2 b
27m b

2 a
a
2
2
(a b ); v 2
v (ax bx 3 ) x(a bx 2 ); v 2
m 3b
3b
m
m
2

1/ 4

Q64. If the operators A and B satisfy the commutation relation A, B I , where I is the identity operator, then

a) e A , B e A
b) e A , B e B , A
Ans: a
Exp: e A , B A, B(e A )1 I .e A e A

c) e A , B e B , A

d) e A , B I

1
p x ay 1 p y bx 2 where a and b are constants and
2
2
px, py are momenta conjugate of x and y respectively. For what values of a and b will the
quantities p x 3 y and p y 2 x be conserved?
a) a = -3, b = 2
b) a = 3, b = -2
c) a = 2, b = -3
d) a = -2, b = 3
Ans: d
Exp: p x 3 y, H 0 ( p y bx)(b 3) 0 b 3 ; p y 2 x, H 0 ( p x ay)(a 2) 0 a 2
Q66. Using the frequency-dependent Drude formula, what is the effective kinetic inductance of a metallic wire
that is to be used as a transmission line? [In the following, the electron mass is m, density of electrons is n,
and the length and cross-sectional area of the wire are and A respectively.]
a) mA / ne 2
b) zero
c) m / ne 2 A
d) m A / ne 2 2
Ans: c
1
1
ml
Exp: Law of conservation energy LI 2 N mvd2 Ln 2 e 2 A2 vd2 nAlmv d2 L 2 2 (Inductance)
2
2
ne A

Q65. A system is governed by the Hamiltonian H

UV PHYSICS ACADEMY
Shivam Road, New Nallakunta, Hyderabad, Ph: 04032458147, 09885072826
www.uvphysics.com
Q67. The phonon dispersion for the following one-dimensional diatomic lattice with masses M1 and M2 (as

1
4M 1 M 2
1
2 qa
1 1
shown in the figure is given by 2 (q) K

sin

where a is the lattice


M 1 M 2 2
2
M 1 M 2
parameter and K is the spring constant. The velocity of sound is

a)

K M 1 M 2
a
2M 1 M 2

b)

K
a c)
2( M 1 M 2 )

K M 1 M 2
a d)
M 1M 2

KM 1 M 1
a
2( M 1 M 2 ) 3

Ans: b
2
1
4M 1 M 2 qa
1
qa qa
2
1 1
Exp: For small q, sin

; (q) k
;
(M 1 M 2 ) 2
2 2
M 1 M 2

2 2
1
M 1M 2
1
q a
1 1
2 (q) k

for Acoustic Branch


2
2
(M 1 M 2 )
M 1 M 2

1
M 1M 2
1
q 2 a 2
;
1 1

2
2
M 1 M 2 ( M 1 M 2 )
M M 2 M 1 M 2
q2a2
ka2

2 (q) k 1

q2;
2

2 2( M 1 M 2 )
M 1 M 2 ( M 1 M 2 )

2 (q) k

qa Vg
2( M 1 M 2 )
q

k
a
2( M 1 M 2 )

Q68. The binding energy of a light nucleus Z, A in MeV is given by the approximate formula

N Z where N = A Z is the neutron number. The value of Z


3
B A, Z 16 A 20 A 2 / 3 Z 2 A 1 / 3 30
4
A
of the most stable isobar for a given A is
2

A
A2 / 3

a) 1
2
160
Ans: a

b) A/ 2

A
A2 / 3

c) 1
2
120

d)

A
A4 / 3
1

2
64

3
30( N Z ) 2
B
0
Exp: B( A, Z ) 16 A 20 A 2 / 3 Z 2 A 1 / 2
for most stable Isobar
4
A
z

80 A
3
30

A 1 / 3 (2Z ) ( A 2Z )(2) 0 Z
4
A
160 A 2 / 3

A A2 / 3

A 2 / 3 2 160
1601

160
80 A

UV PHYSICS ACADEMY
Shivam Road, New Nallakunta, Hyderabad, Ph: 04032458147, 09885072826
www.uvphysics.com
Q69. Muons are produced through the annihilation of particle a and its antiparticle, namely the
process a a A muon has a rest mass of 105MeV/c2 and its proper life time is 2s. If the center
of mass energy of the collision is 2.1GeV in the laboratory frame that coincides with the center-of-mass
frame, then the fraction of muons that will decay before they reach a detector placed 6km away from the
interaction point is
a) e-1
b) 1-e-1
c) 1- e-2
d) e-10
Ans: b
t

Exp: N N 0 e

N
2.1
;
e t e where ( ) is the dilated time
10 3 2 10 6 ;
N0
105

6 10 3
t
1
N
2 10 5 sec;
e 2 0.606 1 e 1
8
2
N0
3 10
Q70. The conductors in a 0.75km long two-wire transmission line are separated by a centre-to-centre distance of
0.2m. If each conductor has a diameter of 4cm, then the capacitance of the line is
a) 8.85F
b) 88.5nF
c) 8.85 pF
d) 8.85nF
Ans: b
Q71. The electron dispersion relation for a one-dimensional metal is given by
ka 1 2

k 2 0 sin 2
sin ka where k is the momentum, a is the lattice constant, 0 is a constant having
2 6

dimensions of energy and ka . If the average number of electrons per atom in the conduction band is
1/3, then the Fermi energy is
a) 0 / 4
b) 0
c) 2 0 / 3
d) 5 0 / 3
Ans: a
Q72. The electronic energy levels in a hydrogen atom are given by En 13.6 / n 2 eV . If a selective excitation to
the n = 100 level is to be made using a laser, the maximum allowed frequency line-width of the laser is
a) 6.5MHz
b) 6.5GHz
c) 6.5Hz
d) 6.5KHz
Ans: b
13.6 2
13.6 2 1.6 10 19
13.6 2 1.6 10 19
eV

6.59GHz
Exp: En
n3
n3
10 6 6.6 10 34
Q73. If the energy dispersion of a two-dimensional electron system is E uk where u is the velocity and k is
the momentum, then the density of states D(E) depends on the energy as
a) 1 / E
b) E
c) E
d) constant
Ans: c
1

Exp: Density of states D( ) s


1 D( ) .

where d is the dimension and s is the power of k in energy. Here d= 2, s =

UV PHYSICS ACADEMY
Shivam Road, New Nallakunta, Hyderabad, Ph: 04032458147, 09885072826
www.uvphysics.com
Q74. Consider the laser resonator cavity shown in the figure. If I1 is the intensity of the radiation at mirror M1
and is the gain coefficient of the medium between the mirrors, then the energy density of photons in the
plane P at a distance x from M1 is

a) I1 / c e x

b) I1 / c e x

c) I1 / c (e x e x )

d) I1 / c e 2 x

Ans: c

I1 x I1 x
I
e e 1 ex e x
c
c
c
Q75. A spin-1/2 particle A undergoes the decay A B C D where it is known that B and C are also spin-1/2
particles. The complete set of allowed values of the spin of the particle D is
1 3 5
1 3 5 7
a) ,1, ,2, ,3,....
b) 0, 1
c) only
d) , , , ,....
2 2 2
2 2 2 2
Ans: c
Exp: A B C D Since A, B and C are fermions with spin=1/2 from the conservation of Fermionic number
D must be a Fermion. Because odd no.of fermions will give a fermion and even no.of fermions will give a
boson.

Exp: Energy density E I / c

UV PHYSICS ACADEMY
Shivam Road, New Nallakunta, Hyderabad, Ph: 04032458147, 09885072826
www.uvphysics.com
CSIR DEC-2012
PART - A
Q1. A granite block of 2m 5m 3m size is cut into 5cm thick slabs of 2m 5m size. These slabs are
laid over a 2m wide pavement. What is the length of the pavement that can be covered with these
slabs?
a) 100m
b) 200m
c) 300m
d) 500m
Ans: C
Q2. Which is the least among the following?
a) 0.330.33
b) 0.44 0.44
c) 1 /
d) e 1 / e
Ans: D
Q3. What is the next number in this see and tell sequence?
1 11 21 1211 111221
a) 312211
b) 1112221
c) 1112222
d) 1112131
Ans: A
Q4. A vertical pole of length a stands at the centre of a horizontal regular hexagonal ground of side a.
A rope that is fixed taut in between a vertex on the ground and the tip of the pole has a length
a) a
b) 2a
c) 3a
d) 6a
Ans: B
Q5. A peacock perched on the top of a 12m high tree spots a snake moving towards its hole at the base
of the tree. The peacock flies towards the snake in a straight line and they both move at the same
speed. At what distance from the base of the tree will the peacock catch the snake?
a) 16m
b) 18m
c) 14m
d)2m
Ans: A
Q6. The cities of a country are connected by intercity roads. If a city is directly connected to an odd
number of other cities, it is called an odd city. If a city is directly connected to an even number of
other cities, it is called an even city. Then which of the following is impossible?
a) There are an even number of odd cities
b) There are an odd number of odd cities
c) There are an even number of even cities
d) There are an odd number of even cities
Ans: B
Q7. In the figure ABC / 2 AD = DE = EB What is the ratio of the area of triangle ADC to that of
triangle CDB?

a) 1 : 1
Ans: B

b)1 : 2

c) 1 : 3

d) 1 : 4

UV PHYSICS ACADEMY
Shivam Road, New Nallakunta, Hyderabad, Ph: 04032458147, 09885072826
www.uvphysics.com
Q8. A rectangular sheet ABCD is folded in such a way that vertex A meets vertex C, thereby forming a
line PQ. Assuming AB=3 and BC=4, find PQ. Note that AP=PC and AQ QC.

a) 13/4
b) 15/4
c) 17/4
d) 19/4
Ans: B
Q9. A string of diameter 1mm is kept on a table in the shape of a close flat spiral i.e. a spiral with no
gap between the turns. The area of the table occupied by the spiral is 1m 2 . Then the length of the
string is
a) 10m
b) 10 2 m
c) 10 3 m
d) 10 6 m
Ans: C
Q10. 25% of 25% of a quantity is x% of the quantity where x is
a) 6.25%
b) 12.5%
c) 25%
d) 50%
Ans: A
a
Q11. In sequence a n every term is equal to the sum of all its pervious terms. If a0 3, then lim n 1 is
n a
n
a) 3
b) 2
c) 1
d) e
Ans: B
Q12. In the figure below, angle ABC / 2. I, II, III are the areas of semicircles on the sides opposite

angles B, A, and C, respectively. Which of the following is always true?


a) II 2 III 2 I 2
b) II III I
c) II 2 III 2 I 2
d) II III I
Ans: B
Q13. What is the minimum number of days between one Friday the 13th and the next Friday the 13th ?
(Assume that the year is a leap year).
a) 28
b) 56
c) 19
d) 84
Ans: C
Q14. Suppose a person A is at the North-East corner of a square (see the figure below). From that point
he moves along the diagonal and after covering1/3rd portion of the diagonal, he goes to his left and
after sometime he stops, rotates 90 o clockwise and moves straight. After a few minutes he stops,
rotates 180 o anticlockwise. Towards which direction he is facing now?

a) North-East
Ans: A

b) North-West

c) South-East

d) South-West

UV PHYSICS ACADEMY
Shivam Road, New Nallakunta, Hyderabad, Ph: 04032458147, 09885072826
www.uvphysics.com
Q15. Cucumber contains 99% water. Ramesh buys 100kg of cucumbers. After 30days of storing, the
cucumbers lose some water. They now contain 98% water. What is the total weight of cucumbers
now?
a) 99kg
b) 50kg
c) 75kg
d) 2kg
Ans: B
Q16. In a museum there were old coins with their respective years engraved on them, as follows.
( A) 1837 AD
(B) 1907 AD
(C ) 1947 AD
(D) 200 BC
Identify the fake coin(s)
a) coin A
b) coin D
c) coins A and B
d) coin C
Ans: B
Q17. A student observes the movement of four snails and plots the graphs of distance moved as a
function of time as given in figures (A), (B) (C) and (D). Which of the following is not correct?

a) Graph (A)
b) Graph (B)
Ans: C
Q18. Find the missing letter:

c) Graph (C)

d) Graph (D)

a) H
b) L
c) Z
d) Y
Ans: C
Q19. Consider the following equation x 2 4 y 2 9 z 2 28 y 42 z 147 where x, y and z are real
numbers. Then the value of x + 2y+ 3z is
a) 7
b) 14
c) 21
d) not unique
Ans: C
C
Q20. The map given below shows a meandering river following a semi-circular path, along which two
villages are located at A and B. The distance between A and B along the east-west direction in the
map is 7cm. What is the length of the river between A and B in the ground?

a) 1.1 km
Ans: C

b) 3.5 km

c) 5.5 km

d) 11.0 km

UV PHYSICS ACADEMY
Shivam Road, New Nallakunta, Hyderabad, Ph: 04032458147, 09885072826
www.uvphysics.com
PART- B
Q21. A 2 2 matrix A has eigenvalues e i / 5 and e i / 6 . The smallest value of n such that A n 1 is
a) 20
b) 30
c) 60
d) 120
Ans: C
Exp: The eigen value of A are given e i / 5 and e i / 6 . An e in / 5 and e ingp/ 6 . If we put n = 60 and 120 we
get 1 but the smallest value of n out of two is 60. e i 60 / 5 e i12 cos 12 i sin 12 1 ;
e i 60 / 6 e i10 cos 10 i sin 10 1
Q22. The graph of the function f(x) shown below is best described by

a) The Bessel function J 0 ( x)

b) cos x

c) e x cos x

1
d) cos x
x

Ans: A
Exp: The Bessel function J0(x) we know J 0 (0) 1, so it starts from 1 and it varies exponentially.
Q23. In a series of five Cricket matches, one of the captains calls Heads every time when the loss is
taken. The probability that he will win 3 times and lose 2 times is
a) 1/8
b) 5/8
c) 3/16
d) 5/16
Ans: D
3

Exp: n = 5, p = , q = ; x = 3; P C x P q
n

n x

5! 1 1 5
1 1
5C x

3!2! 8 4 16
2 2

UV PHYSICS ACADEMY
Shivam Road, New Nallakunta, Hyderabad, Ph: 04032458147, 09885072826
www.uvphysics.com
a b c
Q24. The unit normal vector at the point
,
,
on the surface of the ellipsoid
3 3 3
x2 y2 z 2

1, is
a2 b2 c2
bci caj abk
ai bj ck
bi cj ak
i j k
a)
b)
c)
d)
3
a 2b 2 b 2 c 2 c 2 a 2
a2 b2 c2
a2 b2 c2
Ans: A

x2 y2 z2
2x
2y
2z
; 2 2 2 ; 2 i 2 j 2 k;
Exp: Unit normal n

a
b
c
a
b
c
2 a 2 b 2 c

2
i 2
j 2
k ;
3 ,b / 3 , c / 3
a 3 b 3 c 3
2 2 2

i
j
k
2 2 2
a 3 b 3 c 3

i
j
k ; n

4
4
4
a 3 b 3 c 3
2 2
2
3a
3b
3c

2 1 1 1 bci acj abk


bci acj abk
i j k

bci acj abk


b
c
3 a
abc
abc

2 1
1
1
1
1
1
b 2 c 2 a 2 c 2 a 2b 2
b 2c 2 a 2c 2 a 2b 2
2 2
2
2

2
b
c
3 a
a
b
c
a 2b 2 c 2
Q25. A solid cylinder of height H, radius R and density , floats vertically on the surface of a liquid of
density 0 . The cylinder will be set into oscillatory motion when a small instantaneous downward
force is applied. The frequency of oscillation is
0 g
g
g
g
a)
b)
c)
d)
0 H
0 H
0 H
H
Ans: C
gA
Exp: FR p a Where p is the excess pressure. FR gx A; a
x ;
m
gA
gA
g
2

; 2
m
0 A H
0 H
a /

UV PHYSICS ACADEMY
Shivam Road, New Nallakunta, Hyderabad, Ph: 04032458147, 09885072826
www.uvphysics.com
Q26. Three particles of equal mass m are connected by two identical massless springs of stiffiness
constant k as shown in the figure: If x1 , x2 and x3 denote the horizontal displacements of the masses
from their respective equilibrium positions, the potential energy of the system is

1
1
b) k x12 x22 x32 x2 x1 x3
k x12 x22 x32
2
2
1
1
c) k x12 x22 x32 2 x2 x1 x3
d) k x12 2 x22 x32 2 x2 x1 x3
2
2
Ans: D
1
1
1
1
2
Exp: V k x2 x1 k ( x3 x2 ) 2 k x22 x12 2 x1 x2 k x32 x22 2 x2 x3
2
2
2
2
1
1
k x12 2 x22 x32 2 x1 x2 2 x2 x3 ; V k x12 2 x22 x32 2 x2 ( x1 x3 )
2
2
Q27. Let V, p and E denote the speed, the magnitude of the momentum, and the energy of a free particle
of rest mass m. Then

a)

a) dE / dP cons tan t
Ans: C

b) p = mv

c) v cp / p 2 m 2 c 2

d) E mc 2

dE
c
cp

2 p; v
2
2
2
2
dp 2 p m c
p m2c 2
Q28. A binary star system consists of two stars S1 and S 2 , with masses m and 2m respectively separated
by a distance r. If both S1 and S 2 individually follow circular orbits around the centre of mass with
instantaneous speeds v1 and v2 respectively, the speeds ratio v1 / v2 is
Exp: E

a) 2
Ans: D

p 2 c 2 m 2 c 4 ; E c p 2 m 2 c 2 ;V

b) 1

c)

Exp: m1r1 m2 r2 ; mr1 2mr2 ; mr1 2mr2 mv1 2mv2

v1
2
v2

d) 2

UV PHYSICS ACADEMY
Shivam Road, New Nallakunta, Hyderabad, Ph: 04032458147, 09885072826
www.uvphysics.com
Q29. Three charges are located on the circumference of a circle of radius R as shown in the figure
below. The two charges Q subtend an angle 90 o at the centre of the circle. The charge q is
symmetrically placed with respect to the charges Q. If the electric field at the centre of the circle is
zero, what is the magnitude of Q?

a) q / 2
b) 2q
c) 2q
d) 4q
Ans: A
kQ
kQ
kQ
Exp: E1 2 ; E2 2 ; E12 E12 E22 E1 E2 cos 90 0 ( E1 E2 ); 90 0 ; E12 2 E1 2 2 ;
R
R
R
kQ
2kQ kQ
E3 2 ; E12 E3 0
2 0 2Q q; Q q / 2
R
R2
R
Q30. Consider a hollow charged shell of inner radius a and outer radius b. The volume charge density
k
is (r ) 2 (k is a constant) in the region a<r<b. The magnitude of the electric field produced at
r
distance r > a is
k b a
k b a
kb
a)
b)
for all r a
for a r b and
for r b
2
2
0r
0r
0r 2
k b a
k b a
k b a
k b a
c)
for a r b and
for r b d)
for a r b and
for r b
2
2
2
0r
0r
0a
0a2
Ans: C
r
q
k
k
Exp: (r ) 2 ; (i) r a E.ds enc ; E.4r 2 1 / 0 2 r 2 sin dd
0
r
a r
q
4k r 4k
k
E.4r 2
[r ] a
(r a); E
(r a) (2) (r b) E.ds enc ;
2
0
0
0
0r
b
1 k 2
4k b
4k
k
E 4r 2
r sin dd
[r ]a ; 4r 2
(b a); E
(b a) for r b

2
0
0 a rr
0
0r 2
Q31. Consider the interference of two coherent electromagnetic waves whose electric field vectors are

given by E1 iE0 cos t and E2 jE0 cost where is the phase difference. The intensity of
the resulting wave is given by

b) 0 E

a) 0
Ans: B

E 2 , where E 2 is the time average of E 2 . The total intensity is

2
0

c) 0 E02 sin 2

d) 0 E02 cos 2

Exp: Coherent 0; Eamp E02 E02 2E02 (2E0 ) 2 2E0 ; E 2Eo cos(t );
I

0
2

E 2

0
2

4 02 1 / 2 0 E02

UV PHYSICS ACADEMY
Shivam Road, New Nallakunta, Hyderabad, Ph: 04032458147, 09885072826
www.uvphysics.com
Q32. For charges (two +q and two q) are kept fixed at the four vertices of a square of side a as shown.
At the point P which is at a distance R from the centre (R>>a), the potential is proportional to

b) 1 / R 2

a) 1/R
Ans: C
Exp: Vmono

1
4 0 r

c) 1 / R 3

Q since total charge is zero. V=0 ; Vdip

d) 1 / R 4

p.r; dipole moment

4 0 r

1
1
r 2 (3 cos 2 1) dl
p q 0 qai q(ai aj ) q(aj ) 0; Vquadru
3
2
4 0 r
1
Vqudr 0 V 3
R
Q33. A point charge q of mass m is kept at a distance d below a grounded infinite conducting sheet
which lies in the xy-plane. For what value of d will the charge remains stationary?
a) q / 4 mg 0 b) q / mg 0
c) There is no finite value of d
d) mg 0 / q
Ans: A
kqq1
1 q2
1 q2
q2
q
2
Exp: F

;
F

mg

mg

;
d

;d
2
2
2
4 0 4d
4 0 4d
16 o mg
( 2d )
4 mg 0
2

Q34. The wavefunction of a state of the hydrogen atom is given


by 200 2 211 3 210 2 211 where nlm is the normalized eigen function of the state with
quantum numbers n, l and m in the usual notation. The expectation value of L z in the state is
a) 15 / 16
b) 11 / 16
c) 3 / 8
d) / 8
Ans: D
1
4
9
2
Exp: 200 2 211 3 210 2 211 ; p1 ; p2 ; p3 and p4 ;
16
16
16
16
4
9
2

4 2
Lz E s p s ; E ml ; Lz 0 1 0 1
16
16
16
8
s
16

16 16

UV PHYSICS ACADEMY
Shivam Road, New Nallakunta, Hyderabad, Ph: 04032458147, 09885072826
www.uvphysics.com
1
m 2 x 2 ax are
2
1
a2

b) E n n
2
2m 2

d) E n n
2

Q35. The energy eigenvalues of a particle in the potential V ( x)

1
a2

a) E n n
2
2m 2

1
a2

c) E n n
2
m 2

Ans: A
1
Exp: V ( x) m 2 x 2 a x ; E 0 (n 1 / 2); E (1) H 1 x 0 ; Second order correction
2
a2
a2
to energy E ( 2)
The
energy
eigen
value,
E

(
n

1
/
2
)

2m 2
2m 2
Q36. If a particle is represented by the normalized wave function
15 a 2 x 2
for a x a

the uncertainty p in its momentum is


x 4a 5 / 2
0 otherwise

a) 2 / 5a
b) 5 / 2a
Ans: D
15 a 2 x 2
for a x a

Exp: x
; p
4a 5 / 2
0 otherwise

p 2 p 2 2 *
a

p2
15 2
8a 5

c) 10 / a

p2 p

d)

; p 0;

15
2 15
;

2
x
;

dx
;
x 4a 5 / 2
x 2
2a 5 / 2
x 2

15 2 a 2
15
15 2 a 2
2

(a x 2 )dx ;

(
a

x
)
dx

5
5/ 2
5/ 2
8a a
4a
2a
a
a

2
x3
15 2
a
x

3 a 8a 5

3 2a 3
15 2
2a 3
15 2
15 2
2
2
a

;
;
p

3
3
8a 5
6a 2
2a 2

5 2
15
p p p
; p
2
2a
2a
Q37. Given the usual canonical commutation relations, the commutator
A, Bof A ixp y yp x and B yp z zp y is
2

a) xp z p x z
b) xp z p x z c) xp z p x z
Ans: C
Exp: A i(np y yp x ); B ( yp x zp y ) ; AB i ( xp y yp x ), ( yp z zp y )

5 / 2a

d) xp z p x z

i( x p y , y p z p x y, p y z ) i( x ip z p x iz ) [ A, B] [ xp z p x z ]

UV PHYSICS ACADEMY
Shivam Road, New Nallakunta, Hyderabad, Ph: 04032458147, 09885072826
www.uvphysics.com
Q38. The entropy of a system, S, is related to the accessible phase
volume by S k B ln E, N ,V where E, N and V are the energy, number of particles and volume
respectively. From this one can conclude that
a) does not change during evolution to equilibrium
b) oscillates during evolution to equilibrium
c) is a maximum at equilibrium
d) is a minimum at equilibrium
Ans: C
Exp: S k ln Entropy is maximum in equilibrium
Q39. Let W be the work done in a quasistatic reversible thermodynamic process. Which of the
following statements about W is correct?
a) W is a perfect differential if the process is isothermal
b) W is a perfect differential if the process is adiabatic
c) W is always a perfect differential
d) W cannot be a perfect differential
Ans: B
Exp: dQ = du + dW for adiabalic process, dQ = 0, du dW so w is a perfect differential.
Q40. Consider a system of three spins S1 , S 2 and S 3 each of which can take values +1 and -1. The energy
of the system is given by E J S1 S 2 S 2 S 3 S 3 S1 , where J is a positive constant. The minimum
energy and the corresponding number of spin configurations are, respectively,
a) J and 1
b) -3J and 1
c) -3J and 2
d) -6J and 2
Ans: C
Exp: E J S1 S 2 S 2 S 3 S 3 S1 minimum energy corresponding to and
E J (`1 1 1) 3J
Q41. The minimum energy of a collection of 6 non-interacting electrons of spin-1/2 and mass m placed
in a one dimensional infinite square well potential of width L is
a) 14 2 2 / mL2
b) 91 2 2 / mL2
c) 7 2 2 / mL2
d) 3 2 2 / mL2
Ans: A
2 2 14 2 2

Exp: 6 Parlides total energy = 2 E1 2 4 E1 2 9 E1 28E1 ; E 28


2mL2
2mL2
Q42. A live music broadcast consists of a radio-wave of frequency 7MHz, amplitude-modulated by a
microphone output consisting of signals with a maximum frequency of 10KHz. The spectrum of
modulated output will be zero outside the frequency band
a) 7.00MHz to 7.01MHz
b) 6.99MHz to 7.01MHz
c) 6.99MHz to 7.00MHz
d) 6.995MHz to 7.005MHz
Ans: B
Exp: Carrier frequency, f c 7MHz ; Modulation frequency, f m 10kHz = 0.01MHz. The spectrum
obtained will have modulated frequency in between ( f c f m ) and f c f m
7.01MHZ and 6.99MHz

UV PHYSICS ACADEMY
Shivam Road, New Nallakunta, Hyderabad, Ph: 04032458147, 09885072826
www.uvphysics.com
Q43. In the op-amp circuit shown in the figure, Vi is a sinusoidal input signal of frequency 10Hz and V0
is the output signal. The magnitude of the gain and the phase shift, respectively, close to the values

a) 5 2 and / 2
b) 5 2 and / 2
c) 10 and zero
d) 10 and
Ans: D
Exp: Phase difference must
V
1
1
; 2 ; X c
be . gan 0 ; R f R || X c ; X c
Vin
c
2 10 10 3 0.01 10 6
Rf
10k X c
V
10k
10
Rf

; 0

10
10k X C 1 6.28 10 3 Vin
R1
1 6.28 10 3
Q44. The logic circuit shown in the figure below implements the Boolean expression

a) y A B
b) y A B
c) y = A . B
d) y = A + B
Ans: A
Exp: A C AC CA; C B CB BC ;
A C C B AC CA CB BC C ( A B) C ( A B ) ; Given c is always high
C 1, C 0 o / p becomes 1( A B ) A B out of the options only option (a ) satisfies this.
y A B
Q45. A diode D as shown in the circuit has an i - v relation that can be approximated by
v 2 2v D , for v D 0
iD D
The value of v D in the circuit is
0
,
for
v

0
D

a) 1 11 V
b) 8V
c) 5V
d)2V
Ans: D
2
Exp: Apply KVL 10 i p 1 VD 0;10 i D VD Give iD VD 2VD VD2 3VD 10 0;

(VD 50)(VD 2) 0 VD 2 Since given for VD < 0 , iD=0 so neglect VD=-5

UV PHYSICS ACADEMY
Shivam Road, New Nallakunta, Hyderabad, Ph: 04032458147, 09885072826
www.uvphysics.com
PART C
Q46. The Taylor expansion of the function ln(cosh x), where x is real, about the point x=0 starts with the
following terms:
1
1
1
1
1
1
1
1
a) x 2 x 4 .... b) x 2 x 4 .... c) x 2 x 4 .... d) x 2 x 4 ....
2
12
2
12
2
6
2
6
Ans: B
Exp: Taylors expansion of f(x) about x = a
e x ex
f 11 (a)
f 111(a)
f ( x) f (a) f 1 ( a)
( x a) 2
( x a) 3 ... f ( x) log
0;
2!
3!
2 x 0
1 2 1 4
x x ...
2
12

i
Q47. Given a 2 2 unitary matrix U satisfying U U UU 1 with det U e , one can construct a
unitary matrix V V V VV 1 with detV=1 from it by
f 1 ( x)

x 0

tanh x 0; f 11 ( x)

1 tanh x 1; f 111( x) 2 f ( x)

x 0

i / 2

a) multiplying U by e
b) multiplying any single element of U by e i
c) multiplying any row or column of U by e i / 2
d) multiplying U by e i
Ans: A

e i / 2
0
;
Exp: U is a 2 x 2 matrix and given |U| e i so we have to let U
i / 2
0
e

i / 2
i / 2
i / 2
e
e
0
0 e
0 1 0
| U | e i and U
;UU

I If we
i / 2
i / 2
i / 2
e
e 0
e
0

0
0 1
1 0
+
multiply U by e i / 2 it becomes V V
So that |V| = 1 and VV
0
1

1 0 1 0 1 0
i / 2
=
0 1 0 1 I Multiply U by e
0
1

z 3 dz
, where C is a closed contour defined by the
Q48. The value of the integral 2
C z 5z 6
equation 2 z 5 0, traversed in the anti-clockwise direction, is(MMP)
a) 16 i
Ans: A

b) 16 i

c) 8 i

d) 2 i

z 3 dz
z 3 dz

; z = 2 is a pole z= 3 is not a pole

2
C z 5z 6
C ( z 2)( z 3)
z3
8
z 3 dz
Lim 2
dz
8 ;
2 i ( sum of residues) = 2 i 8 16 i
z 2 z 5 z 6
1
C ( z 2)( z 3)

Exp: Given 2 | z | 5 0 z 5 / 2;

UV PHYSICS ACADEMY
Shivam Road, New Nallakunta, Hyderabad, Ph: 04032458147, 09885072826
www.uvphysics.com
d2 f
3 4i f 0 and satisfies the
dx 2
conditions f 0 1and f x 0 as x . The value of f is
a) e 2
b) e 2
c) e 2
d) e 2 i
Ans: C
2
d2 f
df
mx
mx d f
Exp:
Let

(
3

4
i
)

0
f

ce

cme
,
cm 2 e mx Substituting
dx
dx 2
dx 2
m 2 (3 4i) 0 m (2 i); f ( x) Ae ( 2i 0 x Be ( 2i ) x . f (0) 1 A B 1;
f ( x) 0, where x A 0, B 1
Q50. A planet of mass m moves in the gravitational field of the Sun(mass M). If the semi-major and
semi-minor axes of the orbit are a and b respectively, the angular momentum of the planet is:
Q49. The function f(x) obeys the differential equation

a) 2GMm2 a b

b)

2GMm2 a b

c)

2GMm 2 ab
a b

d)

2GMm 2 ab
ab

Ans: D
Exp:

mv 2 GmM
GM
GM
Gm 2 M 2

;
v

;
L

mvr

m
r

r Gm 2 Mr from the options only


2
r
r
r
r
r
(d) gives the same result by putting a = b = r;

2GMm 2 ab
when a = b = r;
ab

2GMm 2 r 2
Gm 2 Mr
2r
Q51. The Hamiltonian of a simple pendulum consisting of a mass m attached to a massless string of
p02
dL
mg1 cos . If L denotes the Lagrangian, the value of
length is H
is:
2
dt
2m
2g
g
g
2
a)
b) p sin 2
c) p cos
d) p cos
p sin

Ans: A
p2
H

Exp: H 2 mgl(1 cos ); L pi q i L p L; p

(mgl cos ) mgl sin ;

2ml

2 p
p
p2
p2
p2
H

mgl
(
1

cos

)
L

mgl(1 cos ) ;
;
;
p 2ml 2 ml 2
ml 2 2ml 2
2ml 2
p
p
dL 2 p
dL 2 p

p mgl( sin )
g sin g sin ;

g sin
2
dt 2ml
l
l
dt
l

UV PHYSICS ACADEMY
Shivam Road, New Nallakunta, Hyderabad, Ph: 04032458147, 09885072826
www.uvphysics.com
Q52. Which of the following set of phase-space trajectories is not possible for a particle obeying
Hamiltons equations of motion?

a)
b)
c)
d)
Ans: C
Exp: Phase trajectories cannot intersect each other.
Q53. Two bodies of equal mass m are connected by a massless rigid rod of length lying in the xy-plane
with the centre of the rod at the origin. If this system is rotating about the z-axis with a
frequency , its angular momentum is
a) m 2 / 4
b) m 2 / 2
c) m 2
d) 2m 2
Ans: B
ml 2 ml 2 ml 2
ml 2
2
Exp: L I; I mr ; r l / 2 I

; L
4
4
2
2
Q54. An infinite solenoid with its axis of symmetry along the z-direction carries a steady current I. The

vector potential A at a distance R from the axis

a) is constant inside and varies as R outside the solenoid


b) varies as R inside and is constant outside the solenoid
c) varies as 1/R inside and as R outside the solenoid
d) varies as R inside and as 1/R outside the solenoid
Ans: C




Exp: We have B A B.ds A .ds ; B.ds A.dl inside B r 2 A.2r;

Br 0 nI

r Ainsider sin ce r R AinsideR; outside


2
2
nIr 2
1
B.r 2 A2R A 0
; Aoutside
2R
R
A

UV PHYSICS ACADEMY
Shivam Road, New Nallakunta, Hyderabad, Ph: 04032458147, 09885072826
www.uvphysics.com
Q55. Consider an infinite conducting sheet in the xy-plane with a time dependent current
density Kti, where K is a constant. The vector potential at (x, y, z) is given

K
2
by A 0 ct z i. The magnetic field B is
4c
Kt
Kz
K
K
a) 0 j
b) 0
c) 0 ct z i
d) 0 ct z j
j
2
2c
2c
2c
Ans: D
j
i
k
k
k

Exp: A 0 (ct z ) 2 i; B A 0
= j / z[ct z]2 k / y[ct z]2
4c
4c
x
y z
2
(ct z )
0
0

0 k
(ct z ) j
4c
2c

Q56. When a charged particle emits electromagnetic radiation, the electric field E and the Poynting
1
vector S
E B at a large distance r from the emitter vary as 1 / r n and 1 / r m respectively. Which
j 2(ct z ) 1 2 j (ct z )

0 k

;B

of the following choice for n and m are correct?


a) n=1 and m=1
b) n=2 and m=2
c) n=1 and m=2
d) n=2 and m=4
Ans:C
1
1
Exp: S
E B; S 2
0
r
Q57. The energies in the ground state and first excited state of a particle of mass m=1/2 in a potential
V(x) are -4 and -1, respectively, (in units in which 1). If the corresponding wavefunctions are
related by 1 ( x) 0 ( x) sinh x, then the ground state eigenfunction is
a) 0 ( x) sec hx
b) 0 ( x) sec hx
c) 0 ( x) sec h 2 x
Ans: C
Exp: Given E0 4, E1 1solving schrodinger equation for m 1 / 2, 1;

d) 0 ( x) sec h 3 x

2 0
2 2 0

V 0 4 0 .......(1)
0
0 0
2m x 2
x 2
2 0
2 2 1

V 1 1 1 .........(2) Put 1 0 sinh x in equation (2)


1
1 1
2m x 2
x 2
2 0

0
2 sinh x 2
cosh x 0 V 0 sinh x 0 sinh x ;
x
x

0
0
20 2
coth x 0 V 0 0 ; 20 V 0 2
coth x 0 0 By
x
x
x

d o
2 0

2 tanh xdx 0 sec h 2 x

using
0
0
2
0
x

UV PHYSICS ACADEMY
Shivam Road, New Nallakunta, Hyderabad, Ph: 04032458147, 09885072826
www.uvphysics.com
ba x , a x a
Q58. The perturbation H
acts on a particle of mass m confined in an infinite
0, otherwise
0, a x a
square well potential V ( x)
The first order correction to the ground-state energy of
, otherwise.
the particle is
a) ba/2
b) ba / 2
c) 2ba
d) ba
Ans: D
2
x
1
x
1
x
Exp:
cos

cos ; E (1) x( 0) H 1 x( 0) b(a x) cos 2
dx
2a
2a
a
2a
a
2a
a
b a
x
ba
x
x
(a x) cos 2
dx a cos 2
dx x cos 2
dx
a a
2a
a a
2a
2a
a

2x
2x

x1 cos

a
(
1

cos
)

a
a
b
b a
2x 2a
b 1 a
x
b
2
a

2
a

sin
.

dx
x x cos dx

2a 2 a a 2 a
a
a a
2
2
a 2
a a

2
a

b
b x
x b
b a2 a2
2a 0
0; E (1) ba
x cos dx 2a

2
2a 2 a
2a 2
2a 2
2

Q59. Let 0 and 1 denote the normalized eigenstates corresponding to the ground and the first excited

0 1 is
2
d) x / 4m

states of a one-dimensional harmonic oscillator. The uncertainty x in the state


a) x / 2m
b) x / m
c) x 2 / m
Ans: A

; x x 2 x 2 ; x
; x 2
;
Exp: x
2m
2m
m

; x
m 2m

2m

UV PHYSICS ACADEMY
Shivam Road, New Nallakunta, Hyderabad, Ph: 04032458147, 09885072826
www.uvphysics.com
Q60. What would be the ground state energy of the Hamiltonian H

2 d 2
x if variational
2m dx 2

parameter?

n .]
2

b) 2m 2 / 2

[Hint: x 2 n e 2bx dx 2b
2

a) m / 2 2
Ans: C

1
2

c) m 2 / 2

d) m 2 / 2

2
d2
b 2
2b
b 2
2b

(
x
)

;
H

for
2
2m
2m

2m

dx
E
2m 2 2
m 2
minimization,
0b
; E min
b
2
2
Q61. The free energy difference between the superconducting and the normal states of a material is
2b

Exp: 1 A 2

given by F FS FN | | 2

| | 4 , where is an order parameter and and are

2
constants such that 0 in the normal and 0 in the superconducting state, while 0 always.
The minimum value of F is

b) 2 / 2

a) 2 /
Ans: B
Exp: E

Let (F ) min ,

(F ) min (x 2

x4 )
x 2

c) 3 2 / 2

d) 5 2 / 2

(F )

2x 2x 3 0 x 2 ;
x

2
2

Q62. A given quantity of gas is taken from the state A C reversibly, by two paths, A C directly
and A B C as shown in the figure below. During the process A C the work done by the gas
is 100J and the heat absorbed is 150J. If during the process A B C the work done by the gas
is 30J, the heat absorbed is
a) 20J
b) 80J
c) 220J
d) 280J
Ans: B
Exp: dQ1 dU dW1 ;150 dU 100 J dU 50 J ; dQ2 dU dW1 ; dQ2 50 J 30 J 80 J

UV PHYSICS ACADEMY
Shivam Road, New Nallakunta, Hyderabad, Ph: 04032458147, 09885072826
www.uvphysics.com
Q63. Consider a one-dimensional Ising model with N spins, at very low temperatures when almost all
the spins are aligned parallel to each other. There will be a few spin flips with each flip costing an
energy 2J. In a configuration with r spin flips, the energy of the system is E = -NJ + 2rJ and the
number of configuration is N C r ; r varies from 0 to N. The partition function is
N

a)
b) e NJ / k BT
c)
k BT
Ans: D
Exp: Er 2 J 2rJ ; Let N 2 r 0,1,2; E0 2 J and
2

E2 2 J and 2c2 1; N

Nc r e

Er

r 0

Nc r

J
sinh

k
T
B

d) cosh
k
T
B

2c0 1; E1 0 and 2c1 2;

e J e J
(e 2 J 2 e 2 J )

; Z
4
2

(cos J ) 2 ; for N

r 0

particles Z (cos J )
Q64. A magnetic field sensor based on the Hall effect is to be fabricated by implanting As into a Si film
of thickness 1m. The specifications require a magnetic field sensitivity of 500mV/Tesla at an
excitation current of 1mA. The implantation does is to be adjusted such that the average carrier
density, after activation, is
a) 1.25 10 26 m 3
b) 1.25 10 22 m 3
c) 4.1 10 21 m 3 d) 4.1 10 20 m 3
Ans: B
V
1
IB
Exp: H 500mV / T 500 10 3 V / T ; I 1mA 10 3 A; t 1 10 6 m; VH
; carrier
EB
ne
tne
B
IB / m
10 3

1.25 10 22 / m 3
density
netV H 1.6 10 19 1 10 6 500 10 3
Q65. Band-pass and band-reject filters can be implemented by combining a low pass and a high pass
filter in series and in parallel, respectively. If the cut-off frequencies of the low pass and high pass
filter are 0LP and 0HP , respectively, the condition required to implement the band-pass and bandreject filters are, respectively,
a) 0HP 0LP and 0HP 0LP
b) 0HP 0LP and 0HP 0LP
N

c) 0HP 0LP and 0HP 0LP


Ans: B
Exp: 0Hp 0Lp and 0Hp 0Lp

d) 0HP 0LP and 0HP 0LP

UV PHYSICS ACADEMY
Shivam Road, New Nallakunta, Hyderabad, Ph: 04032458147, 09885072826
www.uvphysics.com
Q66. The output characteristics of a solar panel at a certain level of irradiance is shown in the figure
below. If the solar cell is to power a load of 5, the power drawn by the load is

a) 97W
b) 73W
c) 50W
d) 45W
Ans: D
Exp: For 5 resistance, power, P = VxI = 15 x 3 = 45watt.
Q67. Consider the energy level diagram shown below, which corresponds to the molecular nitrogen
laser. If the pump rate R is 10 20 atom cm 3 s 1 and the decay routes are as shown
with 21 20ns and 1 1s, the equilibrium populations of states 2 and 1 are, respectively

a) 1014 cm 3 and 2 1012 cm 3


b) 2 1012 cm 3 and 1014 cm 3
c) 2 1012 cm 3 and 2 10 6 cm 3
d) zero and 10 20 cm 3
Ans: B
Exp: Given R 10 20 atoms / cm 3 sec .; 21 20 10 9 s; 1 1 10 6 s; R A21N 2 ;
N
1
A21
R 2 ; N 2 21 R 20 10 9 10 20 2 1012 cm 3 similarly

21

10 20

21

N1 ; N1 1 10 20 10 6 10 20 1014 cm 3

Q68. Consider a hydrogen atom undergoing a 2P 1S transition. The lifetime t sp of the 2P state for
spontaneous emission is 1.6ns and the energy difference between the levels is 10.2eV. Assuming
that the refractive index of the medium n0 1, the ratio of Einstein coefficients for stimulated and
spontaneous emission B21 / A21 is given by
a) 0.683 1012 m3 J 1 s 1
b) 0.146 10 12 Jsm 3
c) 6.83 1012 m3 J 1 s 1
d) 1.463 10 12 J s m 3
Ans: A
B
2c3
2 2c3 2 2c3

Exp: n0 1, E 10.2eV ; 21
A21 3 n03 3 3 n03 (E ) 3 n03

(3.14 6.62 10 34 ) (3 10 8 ) 3
0.676 1012
19 3
2
(10.2 1.6 10 ) 1 4 (3.14)

UV PHYSICS ACADEMY
Shivam Road, New Nallakunta, Hyderabad, Ph: 04032458147, 09885072826
www.uvphysics.com
Q69. Consider a He-Ne laser cavity consisting of two mirrors of reflectivities R1 1 and R2 0.98. The
mirrors are separated by a distance d=20cm and the medium in between has a refractive
index n0 1 and absorption coefficient 0. The values of the separation between the
modes v and the width v p of each mode of the laser cavity are:
a) v 75kHz, v p 24kHz

b) v 100kHz, v p 100kHz

c) v 750kHz, v p 2.4kHz
d) v 2.4kHz, v p 750kHz
Ans: C
Exp: R1 1, R2 0.98; L d 20cm 20 10 2 m; n0 1, 0, Seperation between modes
c
c
1
where t c cavity life
v

750MHz Width of each mode v p


2
2 Ln0 2 20 10
2t c

2n 0 d
2 1 20 10 2
2
10 7 s
where 1 R1 R2 e 2d ; 0.02 ; t c
1 3
1
3 10 8 ln

c ln
1 0.02
1
1
1
v p

2.38KHz
2t 2 3.14 2 10 7
Q70. Non-interacting bosons undergo Bose-Einstein Condensation (BEC) when trapped in a threedimensional isotropic simple harmonic potential. For BEC to occur, the chemical potential must be
equal to
a) / 2
b)
c) 3 / 2
d) 0
Ans: C
3
Exp: For 3D harmonic potential En (n 3 / 2); ground state energy, E0 ; For BEC occur all
2
the particles must be in ground state. Chemical potential = Ground state of the bosonic system
chemical potential = 3 / 2
Q71. In a band structure calculation, the dispersion relation for electrons is found to be
k cos k x a cos k y a cos k z a where is a constant and a is the lattice constant. The effective
mass at the boundary of the first Brillouin zone is
2 2
4 2
2
2
a)
b)
c)
d)
5 a 2
5 a 2
2a 2
3 a 2
Ans: D
2
2
d 2E
*
2
m

a
Exp: E (k ) (cos k x a cos k y a cos k z a); m *
and
;
dk 2 / a , / a , / a
3a 2
d 2E
2
dk
time. t c

UV PHYSICS ACADEMY
Shivam Road, New Nallakunta, Hyderabad, Ph: 04032458147, 09885072826
www.uvphysics.com
Q72. The radius of the Fermi sphere of free electrons in a monovalent metal with an fcc structure, in
which the volume of the unit cell is a 3 , is
1/ 3

1/ 3

1/ 3

12 2
3 2
2

a) 3
b) 3
c) 3
a
a

a
Ans: A
2
2
Exp: E f
(3 2 n) 2 / 3 ; f
(k f ) 2 k f (3 2 n)1 / 3 (3 2 N / V ) 2
2m
2m

d)

1
a

1/ 3

12 2
N
4
k f 3
( for fcc
3)
V a
a
2
Q73. The muon has mass 105MeV / c and mean lifetime 2.2s in its rest frame. The mean distance

traversed by a muon of energy 315MeV / c 2 before decaying is approximately


a) 3 10 5 km
b) 2.2cm
c) 6.6m
d) 1.98km
Ans: D
m0 c 2
Exp: E 315MeV , m0 105MeV / c 2 ; E mc 2 E m0 c 2 315
v2
1 2
c
6
t0
2.2 10
v 0.94c; t
t
6.6s ; distance traveled
2
1 8/ 9
v
1 2
c
6
vt 0.94c 6.6 10 1.9km.
Q74. Consider the following particles: the proton p, the neutron n, the neutral pion 0 and the delta
resonance . When ordered in terms of decreasing lifetime, the correct arrangement is as follows:
a) 0 , n, p,
b) p, n, , 0
c) p, n, 0 ,
d) , n, 0 , p
Ans: C
Q75. The single particle energy difference between the p-orbitals (i.e., p3 / 2 and p1 / 2 ) of the
nucleus 114
50 Sn is 3MeV. The energy difference between the states in its 1f orbitals is
a) -7MeV
b) 7MeV
c) 5MeV
d) -5MeV
Ans: B
E p 3
7
7
E f E p 3MeV 7 MeV
Exp: E (2l 1);
E f 7
3
3

UV PHYSICS ACADEMY
Shivam Road, New Nallakunta, Hyderabad, Ph: 04032458147, 09885072826
www.uvphysics.com
Q1.

CSIR JUNE-2012
In still air, fragrance of a burning incense stick will be smell by an observer quickest when the
experiment is carried out at
a) low altitude and high air temperature
b) high altitude and low air temperature
c) low altitude and low air temperature
d) high altitude and high air temperature

Ans: d
Q2.
How many squares are there in this figure?
a) 9
b) 14
c) 15
d) 17
Ans: c
Q3.
A mountain road has 3 sections of different slopes as shown. What is the average slope m of the
entire climb?

a) 1

b) (1/3) < m < (1/2)

c) 1 m 3

d) 1 / 3 m 1

ANS:D
Q4.
Which of the following graphs shows the concentration of a sugar solution as a function of the
cumulative amount of sugar added in the process of preparing a saturated solution (the
temperature remaining constant)?

a)

b)

c)

d)

ANS:B
Q5.
There are sand-piles which are geometrically similar but of different heights. The ratio of the
masses of the sand comprising two randomly chosen piles will be equal to the ratio of the
a) pile heights
b) squares of the pile height
c) cubes of the pile height
d) cube-roots of the pile heights
ANS:C
Q6.
There are two identical vessels of volume V each, one empty, and the other containing a block of
wood of weight w. The vessels are then filled with water up to the brim. The two arrangements
are shown as A and B in the figure. If the density of water is and g is the acceleration due to
gravity, then
a) A and B have equal weights
b) A is heavier than B by an amount w.
c) A is heavier than B by an amount Vg w.
d) B is heavier than A by an amount Vg w.
ANS:A

UV PHYSICS ACADEMY
Shivam Road, New Nallakunta, Hyderabad, Ph: 04032458147, 09885072826
www.uvphysics.com
Q7.

If the father has blood group O and the mother has blood group AB, what are the possible blood
groups of their children?
a) O, AB, A
b)A, B
c) A, O
d) B,AB

ANS:B
Q8.
Nuclei of 32 P and 32 S , accelerated through the same potential difference enter a uniform,
transverse magnetic field (Z = 15 for P and Z = 16 for S). As they emerge from the magnetic
field
a) both nuclei emerge undeflected
b) 32P is deflected less than 32S.
32
32
c) P is deflected more than S
d) both are equally deflected
ANS:B
Q9.
A person chewing a bubble gum did not experience ear pain in a jet plane while landing whereas
another person not chewing a gum had ear pain. The reason could be
a) chewing gum is a pain killer
b) chewing equilibrates pressure on both sides of the ear drum
c) chewing gum closes the ear drum
d) chewing distracts the person
ANS:B
Q10. The reason why an lunar eclipse does not occur in every full moon is
a) the position of the sum is not favourable at all full moons.
b) the orbital planes of the moon and that of the earth are inclined to each other by a small angle.
c) the shape of the earth is not a perfect sphere. d) the moon reflects only from one hemisphere
ANS:B
Q11. A boy throws a stone vertically upwards with a certain initial velocity. Which of the following
graphs depicts the velocity as a function of time, if the acceleration due to gravity is assumed to
be uniform and constant?

a)
b)
c)
d)
ANS:A
Q12. A rigid uniform bar of a certain mass has two bobs of the same size, but with different
densities and 2 suspended kinetically from its ends. When the bar is level on a fulcrum as
shown in the figure, d and d are related by

a) 2d d
ANS:D

b) d 2d

c) d 2d

d) d 2d

UV PHYSICS ACADEMY
Shivam Road, New Nallakunta, Hyderabad, Ph: 04032458147, 09885072826
www.uvphysics.com
Q13.

There are two points A and A on the equator at longitudes 0o and 90o E, and two other points B
and B on the same longitudes, respectively, but at latitude 60o S . The distances (along the
latitudes) between the points A, A and B, B are related by
a) AA BB

b) AA 2BB

c) AA

3 BB d) AA 2 BB

ANS:B
Q14. Water is flowing through a tube as shown. The cross-sectional areas at A and C are equal, and
greater than the cross-sectional area at B . If the flow is steady, then the pressure on the walls at
B is

a) less than that at A and that at C


c) same as that at A and that at C

b) more than at A and that at C


d) more than that at A but less than that at C.

ANS:A
Q15. Match the two lists
Raw Material
A. Limestone
B. Gypsum
C. Silica sand
D. Clay

Product
a. Porcelain
b. Glass
c. Plaster of Paris
d. Cement

a) A a; B b; C c; D d
c) A a; B c; C d ; D b

b) A d ; B c; C b; D a
d) A d ; B a; C c; D b

ANS:B
Q16. The 14C dating method is not usually used for dating organic substances older than ~60,000
years, because
a) such objects rarely contain carbon
b) such objects accumulated 14C after their formation.
c) in those times there was no production of 14C.
d) most of the 14C in the sample would have decayed.
ANS:D
Q17. A seismograph receives a S-wave 60s after it receives the P-wave. If the velocities of P and Swaves are 7km/s and 6km/s respectively, then the distance of the seismic focus from the
seismograph is
a) 2520km
b) 42km
c) 7070km
d) 72km
ANS:A
Q18. The decay of a radioactive isotope P produces a stable daughter isotope D. The ratio of the
number of atoms of D to the number of atoms of P after 2 half lives would be
a)
b)
c) 3
d) 2
ANS:C

UV PHYSICS ACADEMY
Shivam Road, New Nallakunta, Hyderabad, Ph: 04032458147, 09885072826
www.uvphysics.com
Q19.

The scatter plots represent the values measured by two similar instruments. Point A for the
figures represents the true value. Which of the following is a correct description of the quality of
these measurements?

a) Fig 1: good accuracy, good precision, Fig 2: good accuracy, good precision
b) Fig 1: poor accuracy, poor precision, Fig 2: good accuracy, poor precision
c) Fig 1: poor accuracy, good precision, Fig 2: poor accuracy, poor precision
d) Fig 1: poor accuracy, poor precision, Fig 2: poor accuracy, good precision
ANS:C
Q20. Even though the concentration of CO2 is the same at sea level and high altitude, the
photosynthetic rate is higher in a plant grown at sea level than in a plant (of the same species)
grown at high altitude. The reason for this is
a) light intensity is more at sea level.
b) temperature is lower at higher altitude.
c) atmospheric pressure is higher at sea level.
d) relative humidity is higher at sea level.
ANS:C
PART B
Q21.

A vector perpendicular to any vector that lies on the plane defined by x + y + z = 5, is


a) i j
b) j k
c) i j k
d) 2i 3 j 5k

ANS:C


Exp: x y z i j k ( x y z 5) i j k
x
x
x
1 2 3

Q22. The eigenvalues of the matrix A 2 4 6 are


3 6 9

a) (1, 4, 9)
b) (0, 7, 7)
c) (0, 1, 13)
ANS:D
1
2
3

Exp: | A I | 0 2
3

d) (0, 0, 14)

0 3 142 0 2 ( 14) 0; 0,0,14

UV PHYSICS ACADEMY
Shivam Road, New Nallakunta, Hyderabad, Ph: 04032458147, 09885072826
www.uvphysics.com
Q23.

The first few terms in the Laurent series for

z 1z 2

in the region 1 | z | 2 and around z= 1 is

a)

z z2 z2

1
1 z z 2 ..... 1 ....
2
2 4 8

b)

c)

1 1 1
2 4

1 2 ....1 2 .....
2
z z z
z z

d) 2z 1 5z 1 7z 1 ......

1
2
3
z 1 z 1 z ....
1 z
2

ANS:B

A
B

;1 A( z 2) B( z 1); Put z=2 B 1; put z=1 A 1;


z 1z 2 z 1 z 2
1
1
1
1
1
put z-1=u;

( z 1)( z 2) z 1 z 2 1 z ( z 1 1)
1
1
1

1(1 u ) 1 11 u u 2 u 3 ...
z 1 1 u 1 1 u
11 ( z 1) ( z 1) 2 ( z 1)3 ... z ( z 1) 2 ( z 1)3 ...
1
1
1

z ( z 1) 2 ( z 1) 3 .....
z (1 z ) 2 (1 z ) 3 ...
( z 1)( z 2) 1 z
1 z
Q24. The radioactive decay of a certain material satisfies Poisson statistics with a mean rate of per
second. What should be the minimum duration of counting (in seconds) so that the relative error
is less than 1%?
a) 100 /
b) 10 4 / 2
c) 10 4 /
d) 1 /
ANS:C
1
Q25. Let u x, y x x 2 y 2 be the real part of an analytic function f(z) of the complex variable z =
2
x + iy. The imaginary part of f(z) is
2
2
a) y + xy
b) x y
c) y
d) y x
ANS:A
1
u
u
u v
u v
Exp: u ( x, y) x ( x 2 y 2 ) ;
1 x....(1); y...(2) ; CR equation is

and

2
x
y
x y
y
x
v
v
y f 1 ( x).....(3) comparing with (2) we
1 x; V y xy f (x);
x
y
1
get f ( x) 0 f ( x) const 0( Let ) so v y xy f (x) becomes V y xy (Imaginary
part)
Exp:

UV PHYSICS ACADEMY
Shivam Road, New Nallakunta, Hyderabad, Ph: 04032458147, 09885072826
www.uvphysics.com
Q26.

Let y(x) be a continuous real function in the range 0 and 2 , satisfying the inhomogeneous

d2y
dy

cos x x . The value of dy/dx at the point x / 2


2
dx
2
dx

a) is continuous
b) has a discontinuity of 3
c) has a discontinuity of 1/3
d) has a discontinuity of 1
differential equation: sin x

ANS:D

d2y
dy
d
dy
dy
cos n ( x / 2);
subx ( x / 2) sin x ( x / 2)
2

dx
dx
dx
dx
dx
dy
dy
dy

when x / 2, ( x / 2) (0) 1 sin x 1 i.e. sin


1;
1
dx
2 dx
dx

A ball is picked at random from one of two boxes that contain 2 black and 3 white and 3 black
and 4 white balls respectively. What is the probability that it is white?
a) 34/70
b) 41/70
c) 36/70
d) 29/70

Exp: sin

Q27.

ANS:B
Exp: Box-A; Probability of getting one white ball from Box A = 3/5.
Box B; Probability of getting one white ball from Box B=4/7;
1 4 3 41
Probability of getting one white ball out of the two boxes =
2 7 5 70
Q28. The bob of a simple pendulum, which undergoes small oscillations, is immersed in water. Which
of the following figures best represents the phase space diagram for the pendulum?

a)
b)
c)
d)
ANS:D
Exp: When simple pendulum is immersed in water, it behaves like a damped oscillator.
Q29. Two events, separated by a (spatial) distance 9 10 9 m, are simultaneous in one inertial frame.
The time interval between these two events in a frame moving with a constant speed 0.8c (where
the speed of light c 3 108 m / s ) is
a) 60s
b) 40s
c) 20s
d) 0s
ANS:B
v
(t 2 t1 ) 2 ( x2 x1 )
c
Exp: x2 x1 9 109 m ;Simultaneous events so t1 t 2 i.e., t 2 t1 0; t 21 t11
1 v2 / c2
0.8c
0 2 9 10 9
2
2
t v / c x
24
c
i.e., t
; v 0.8; t 1
; t 1 t 21 t11
40 sec onds
2
2
0.6
1 v / c
0.8c
1

UV PHYSICS ACADEMY
Shivam Road, New Nallakunta, Hyderabad, Ph: 04032458147, 09885072826
www.uvphysics.com
Q30.

If the Lagrangian of a particle moving in one dimensions is given by L


Hamiltonian is
1
a) xp 2 V ( x)
2

b)

x 2
V ( x)
2x

c)

1 2
x V ( x)
2

x 2
V ( x) , the
2x
d)

p2
V ( x)
2x

ANS:A

x 2
L 2 x x
( px) 2
V ( x); H px L; p

x px; L
V ( x);
2x
x 2 x x
2x
( px) 2

p2 x
xp 2
H p( px)
V ( x) p 2 x
V ( x)
V ( x)
2
2
2x

A horizontal circular platform rotates with a constant angular velocity directed vertically
upwards. A person seated at the centre shoots a bullet of mass m horizontally with speed v. The
acceleration of the bullet, in the reference frame of the shooter, is
a) 2v to his right
b) 2v to his left
c) v to his right
d) v to his left

Exp: L

Q31.

ANS:A



Exp: Coriolis force 2m(v ) since v given 2m(v ) ; Bullet is projected upward i.e,
y direction y. Let the person is situated along x, v vx

F 2m(vx y ); F 2mvz; F ma; ma 2mvz a 2vz


Thus the acceleration is towards right.

UV PHYSICS ACADEMY
Shivam Road, New Nallakunta, Hyderabad, Ph: 04032458147, 09885072826
www.uvphysics.com
Q32.

1 10

The magnetic field corresponding to the vector potential A F r 3 r where F is a constant


2
r
vector, is
30
30

a) F
b) - F
c) F 4 r
d) F 4 r
r
r

ANS:A
1 10 1

r
r
Exp: A ( F r ) 3 r ; B A; B ( F r ) 10 3 But we know 3 0;
2
r
2
r
r
j k
i
1

B ( F r ) ; F r Fx Fy Fz izFy yFz j zFx xFz kyFx xFy
2
x
y z

i
j
k

i ( yFx xFy ) ( yFx xFy )


( F r ) / x
/ y
/ z
z
y

( zFy y ( zFx xFz ) ( yFx xFy )

j ( yFx xFy ) ( zFy yFz ) k ( zFx xFz ) ( zFy yFz )


z
y
y

= i Fx Fx j Fy Fy k Fz Fz 2Fx i 2Fy j 2Fz k ;

Q33.


1


1
( F r ) 2( Fx i Fy j Fz k) 2 F ; B ( F r ) 2 F ; B F
2
2
An electromagnetic wave is incident on a water-air interface. The phase of the perpendicular
component of the electric field, E , of the reflected wave into the water is found to remain the
same for all angles of incidence. The phase of the magnetic field H.
a) does not change
b) changes by 3 / 2
c) changes by / 2
d) changes by

ANS:D
Q34. The magnetic field at a distance R from a long straight wire carrying a steady current I is
proportional to
a) IR
b) I / R 2
c) I 2 / R 2
d) I / R
ANS:D
I
I
I
1
Exp: B 0 sin sin for a long straight wire 900 B 0 2 0 ; B
4R
4R
2R
R
Q35. The component along an arbitrary direction n , with direction cosines nx , n y , nz , of the spin of a
spin-1/2 particle is measured. The result is

a) 0
b) n x
c) n x n y n z d)
2
2
2
ANS:D
Exp: The eigen values of and ( .n ) are same. Hence eigen values of ( .n) / 2

UV PHYSICS ACADEMY
Shivam Road, New Nallakunta, Hyderabad, Ph: 04032458147, 09885072826
www.uvphysics.com
Q36.

A particle of mass m is in a cubic box of size a. The potential inside the box
0 x a, 0 y a, 0 z a is zero and infinite outside. If the particle is in an eigen state of
energy E

14 2 2
, its wavefunction is
2ma 2

2
a)
a

3/ 2

2
c)
a

3/ 2

3x
5y
6z
sin
sin
sin
a
a
a

2
b)
a

3/ 2

4x
8y
2z
sin
sin
sin
a
a
a

2
d)
a

3/ 2

sin
sin

7x
4y
3z
sin
sin
a
a
a

x
a

sin

2y
3z
sin
a
a

ANS:D

x 2y 3z
2
Exp: n n n 14 from the option (d) 1 + 4 + 9 =14; sin sin
sin
a
a
a
a
Q37. Let n ln denote the eigenfunctions of a Hamiltonian for a spherically symmetric potential V(r).
1
The wavefunction 210 5 211 10 211 is an eigenfunction only of
4
2
a) H , L and Lz
b) H and Lz
c) H and L2
d) L2 and Lz
ANS:C
1
13.6
Ans: 210 5 21, 4 212 ; H
; L2 l (l 1) 2 ; Lz ml
2
4
n
since n and l are same for all states no change in wave function. So H and L2 are eigen value of
. but ml value are different, so . is not an eigen function of Lz.
3/ 2

2
x

2
y

2
z

The commutator x , p is
a) 2 i x p
b) 2 i x p p x

Q38.

d) 2 i xp px

c) 2 i px

ANS:B
Exp: x 2 , p 2 xx, p 2 x x, p 2 x, p 2 x x 2ip 2ipx 2i( xp px)
Q39. Consider a system of non-interacting particles in d obeying the dispersion
relation Ak , where is the energy, k is the wavevector, s is an in meter and A is constant. The
density of states, N , is proportional to

a) d
ANS:C
Exp:

b)

d
1
s

Ak ; density of state

Q40.

dim
1
power of k

c)
d

; N ( ) s

d
1
s

d) d

The number of ways in which N identical bosons can be distributed in two energy levels, is
a) N + 1
b) N (N 1)/2
c) N(N + 1)/2
d) N

ANS:A
Exp: WMB

(ni g i 1)!
( N 2 1)! ( N 1)!
; g i 2, ni N ; WMB

N 1
n!( gi 1)!
N!(2 1)!
N!1!

UV PHYSICS ACADEMY
Shivam Road, New Nallakunta, Hyderabad, Ph: 04032458147, 09885072826
www.uvphysics.com
Q41.

The free energy of a gas of N particles in a volume V and at a temperature T


5/ 2
is F Nk BT ln a0V k BT / N , where a0 is a constant and k B denotes the Boltzmann constant.
The internal energy of the gas is
3
5
3
5/ 2
5/ 2
a) Nk BT
b) Nk BT c) Nk BT ln a0V k BT / N Nk BT d) Nk BT ln a0V / k BT
2
2
2

ANS:B

a0 v ( k B T ) 5 / 2
a 0 v ( k BT ) 5 / 2
Exp: Given F Nk BT ln
we know F kT ln z Nk BT ln

N
N

a0 v
5/ 2

a v(k T )
2
5
2
1

ln z N ln 0 B
we know
ln z N
ln
; U
N
2
2 2 N
kT

5
1
5
N
NkT
2

2
Q42. In the op-amp circuit shown in the figure below, the input voltage V1 is 1V. The value of the
output V0 is
a) -0.33 V
b) -0.50 V
c) -1.00 V
d) -0.25V
ANS:C
VA V1 VA VB
0 V1 0 V0

0;

0 V1 V0 ; V0 1V
1
1
1
1
Q43. An LED operates at 1.5V and 5mA in forward bias. Assuming an80% external efficiency of the
LED, how many photons are emitted per second?
a) 5.0 1016
b) 1.5 1016
c) 0.8 1016
d) 2.5 1016
ANS:D
Exp: V = 1V; I=3mA; P = VIx efficiency =1.5x5x0.8x10-3=6x10-3watt

Exp:

p nhv; hv for LED, Eg 1.43eV ; n


Q44.

p
6 10 3

2.5 1016 / s
1.43eV 1.43 1.6 10 19

The transistor in the given circuit has h fc 35 and hlc 1000. If the load
resistance RL 1000, the voltage and current gain are, respectively.

a) 35 35
b) 35 35
c) 35 0.97
d) 0.98 35
ANS: A
Exp: h fe 35; hie 1000; RL 1000; for common emitter h fe current gain current gain = 35.
voltage gain=

Av

h fe .rL
hie

h fe .rL
hie (1 hoe .rL ) h fe .hre .rL

; ; hre and hoe are negligible. Then

35 1000
35 . Therefore current gain and voltage gain become 35 and - 35
1000

UV PHYSICS ACADEMY
Shivam Road, New Nallakunta, Hyderabad, Ph: 04032458147, 09885072826
www.uvphysics.com
Q45.

The experimentally measured transmission spectra of metal, insulator and semiconductor thin
film are shown in the figure. It can be inferred that I, II and III correspond, respectively, to

a) insulator, semiconductor and metal


c) metal, semiconductor and insulator

b) semiconductor, metal and insulator


d) insulator, metal and semiconductor

ANS:A
Exp: Transmittance of metal does not depend on wave length.
PART C

Q46.

n3 n2
0

0 n1 where n1 , n2 and n3 are the


The eigenvalues of the antisymmetric matrix, A n3
n2 n1
0
components of a unit vector, are
a) 0, i, -i
b) 0,1, -1
c) 0, 1+i,-1-i
d) 0, 0, 0

ANS:A

n3 n2
0

0 n1 sum=0. Since it is an antisymmetric matrix. Eigen values are zero and


Exp: A n3
n2 n1
0
purely imaginary. eigen values 0, i, - i.
Q47. Which of the following limits exists?
N 1
N 1

N 1

N 1

ln N
a) lim ln N b) lim ln N
c) lim
d) lim
N N 1 m
N N 1 m
N N 1 m
N N 1 m

ANS:B
Q48. A bag contains many balls, each with a number painted on it. There are exactly n balls which
have the number n(namely one ball with 1, two balls with 2, and so on until N balls with N on
them). An experiment consists of choosing a ball at random, noting the number on it and
returning it to the bag. If the experiment is repeated a large number of times, the average value of
the number will tend to
N 1
N N 1
2N 1
N
a)
b)
c)
d)
3
2
2
2
ANS:A
Exp: N Es Ps 1

1
1
1
2
3
.....N N N
N
N
N

1
N 2 N ( N 1)(2 N 1) 2 N 1
(12 2 2 32 ..... N 2 )
N ( N 1)
3
N
N
6
2

UV PHYSICS ACADEMY
Shivam Road, New Nallakunta, Hyderabad, Ph: 04032458147, 09885072826
www.uvphysics.com
1
t
cos
dt is
2
2R
t R
b) / R

The value of the integral

Q49.

a) 2 / R

d) 2 / R

c) / R

ANS:A
iz

1
1
1
t
t
e 2 R ; z=R and z=-R are the poles
cos
cos
dt ; 2
can be written as 2
2
2
2
2
2
R
2
R
t

R
t

R
z

Exp:

1
1
z 1 2
z 1 2
cos
e ; At z = -R 2
cos
e ; sum of

2
2
2
z R
z R
2R 2R
2R 2R

At z=R Residue

1 i / 2 i / 2
i e i / 2 e i / 2
i
i
e
e

sin ;
2R
R
2i
R
2 R

1
i
2
z
z 2 R 2 cos 2R dz 2 i(sum of residues) 2i R R

The Poisson bracket | r | . | p |has the value


a) | r || p |
b) r. p
c) 3
residues

Q50.

d) 1

ANS:B
3 | r | | p |
| r | | p | 3 | r | | p |

Exp: | r | . | p |
;

pi
pi qi i 1 qi pi
i 1 qi

| r | | p | | r | | p | | r | | p | x px
y py
z p z xp x yp y zpz
r.p

r. p
x p x
y p y
z p z
|r|| p| |r|| p| |r|| p|
| r || p |
| r || p |
Q51. Consider the motion of classical particle in a one dimensional double-well potential
2
1
V ( x) x 2 2 . If the particle is displaced infinitesimally from the minimum on the positive x4
axis (and friction is neglected), then
a) the particle will execute simple harmonic motion in the right well with an angular
frequency 2
b) the particle well execute simple harmonic motion in the right well with an angular
frequency 2
c) the particle will switch between the right and left wells
d) the particle will approach the bottom of the right well and settle there
ANS:A, B
2 V ( x)
1
1
2( x 2 2) 2 x x( x 2 2) ;
Exp: V ( x) x 2 2 ;
4
x
4
2V ( x)
V ( x)
3x 2 2 | x0 2;
0 x( x 2 2) 0 x 0, x 2 ;
2
x x0
x

2V ( x)
x 2

3 2 2 1; 2 4; 2 .
x 2

The particle will execute SHM in the right well with an angular frequency 2

UV PHYSICS ACADEMY
Shivam Road, New Nallakunta, Hyderabad, Ph: 04032458147, 09885072826
www.uvphysics.com
Q52.

What is the proper time interval between the occurrence of two events if in one inertial frame the
events are separated by 7.5 108 m and occur 6.5s apart?
a) 6.50s
b) 6.00s
c) 5.75s
d) 5.00s

ANS:B
Exp: When observed from rest frame the 2 events occurs at same location x2 x1 0, the proper time
t 0
proper ;
interval is measured from that frame. t 1
1 v2 / c2
t 0
x vt 0
2
2
1
6.5
t 0 6.5 1 v / c ; v ? x
1 v2 / c2
1 v2 / c2

x1

x v 6.5 1 v 2 / c 2
1 v2 / c2

0 v 6.5; given

x1 7.5 108 ; 7.5 108 6.5v v

7.5 108
1.15 108 m / s;
6.5

Q53.

1.15 108
6.5 1 0.1467 6.00 sec onds
t 0 6.5 1
8
3 10
A free particle described by a plane wave and moving in the positive z-direction undergoes
V if r R
scattering by a potential V (r ) 0
If V0 is changed to 2V0 keeping R fixed, then the
0 if r R
differential scattering cross-section, in the Born approximation.
a) increases to four times the original value
b) increases to twice the original value
c) decreases to half the original value
d)decreases to one fourth the original value

ANS:A

d
2mV
Exp: The differential scattering cross-section
| f ( , ) |2 2 0
d
k

r sin krdr ;
0

d
2mV0

2 r sin krdr
2
2 R
d

V0
k 0
2m2V0
d
d
d

;
4

2 r sin krdr ;
2
2 R
d
d 2V0
d V0
d 2V0 k 0
2
m
2
V

r sin krdr
d 2V0 2 k 0
(a) increases 4 times the original value.
2 R

UV PHYSICS ACADEMY
Shivam Road, New Nallakunta, Hyderabad, Ph: 04032458147, 09885072826
www.uvphysics.com
Q54.

A variational calculation is done with the normalized trial wavefunction


0 if | x | a
15
The ground
( x) 5 / 2 a 2 x 2 for the one dimensional potential well V ( x)
4a
if | x | a
state energy is estimated to be
3 2
3 2
5 2
5 2
a)
b)
c)
d)
2ma 2
4ma 2
3ma 2
5ma 2
ANS:D
0 if | x | a
15
2 a * 2
Exp: ( x) 5 / 2 (a 2 x 2 );V ( x)
dx ;
; H * Hdx

2m a x 2
4a
if | x | a

2 15
15 a * 2 15 a 2

15
5 2 5 / 2 ;
5 (a x 2 )dx ;
5 / 2 2 x;
2
2
x
8a
2a
x 4a
x
8a a
a
H

Q55.

2 a * 2
2 5
5 2

dx

2m a x 2
2m 2a 2 4ma 2

if x 0

A particle in one-dimensional is in the potential V ( x) V0 if 0 x t If there is a least one


0 if c t

bound state, the minimum depth of the potential is


2 2 2
2 2
2 2
2 2
a)
b)
c)
d)
mt 2
8mt 2
2mt 2
mt 2

ANS:A

if x 0
2 2

Exp: F ( x) V0 if 0 x t ground state energy


; (n-1)2 z.p.E V0 n 2 zp.E no.of bound states
2
2ml
0 if c t

2 2
2 2

= 1 so 0 V0
0
2ml 2
8ml 2
Q56. Which of the following is a self-adjoint operator in the spherical polar coordinate
system r , , ?
a)
ANS:C

i 2
sin 2

b) i

c)

i 2
sin

d) i sin

UV PHYSICS ACADEMY
Shivam Road, New Nallakunta, Hyderabad, Ph: 04032458147, 09885072826
www.uvphysics.com

*
Exp: A A * A d A d

i
i

i 2
* 2
*
r drdd

r dr sin dd i

sin
sin d
sin
Q57. Which of the following quantities is Lorentz invariant?
a) E B

b) E | B |2

c) E | B |2

d) E . | B |2

ANS:B
Exp: | E 2 | | B |2 is Larentz invariant
Q58. Charges Q, Q and -2Q are placed on the vertices of an equilateral triangle ABC of sides of length
a, as shown in figure. The dipole moment of this configuration of charges, irrespective of the
choice of origin, is

a) 2aQ i

b) 3aQ j

c) 3aQ j

d) 0

ANS:C
Exp: P Q 0 Qai (2Q)(a / 2i 3 / 2aj ) Qai Qai 3aQj 3aQj
Q59.

m r
. If m is
r3
directed along the positive z-axis, the x-component of the magnetic field, at the point r, is
3m z 2 xy
3myz
3mxy
3mxz
a) 5
b) 2
c)
d)
r
r
r5
r5

The vector potential A due to a magnetic moment m at a point r is given by A

ANS:C



m

mx
r
m r
m r

Exp: A 3 ; B A 3 ; m r m yi xj ; B 3 i 3
r
r
r
z r
1 r
3 z 3mxz
mx 3 mx 4 ; B 5 i
r r z
r
r r
Q60. A system has two normal modes of vibration, with frequencies 1 and 2 21. What is the
probability that at temperature T, the system has an energy less than 41 ?

[In the following x e i 1 and Z is the partition function of the system]


a) x 3 / 2 x 2 x 2 / Z
b) x1/ 2 1 x x 2 / Z c) x 3 / 2 1 2 x 2 / Z

d) x 3 / 2 1 x 2 x 2 / Z

ANS:D
3
Exp: E nx 2n y 3 / 2 consider the energies less than 4 ; for (nx , n y ) (0,0) E ;
2
5
7
for (nx , n y ) (0,1) E ; for (nx , n y ) (1,0) E ;
2
2

UV PHYSICS ACADEMY
Shivam Road, New Nallakunta, Hyderabad, Ph: 04032458147, 09885072826
www.uvphysics.com

g s e e 2 e 2 2e 2
7

pute x
for (nx , n y ) (2,0) E ; p
z
z
2
3/ 2
5/ 2
7/2
3/ 2
2
x x 2x
x (1 x 2 x )
; p
p
z
z
3

Q61.

The magnetization M of a ferromagnet, as a function of the temperature T and the magnetic field
H
T
H, is described by the equation M tanh c M . In these units, the zero-field magnetic
T
T
susceptibility in terms of M (0) M H 0 is given by
a)

1 M 2 (0)
T Tc 1 M 2 (0)

b)

1 M 2 (0)
T Tc

c)

1 M 2 (0)
T Tc

d)

1 M 2 (0)
T

ANS:A
H M
H T M 1
T
T
Exp: M tanh c M ;
sec 2 h c M c
;
T H
T T H T
T
T
H T M 1 M
T
T M 1
1 tan 2 h c M c
;
(1 M 2 ) c
;
T T H T H
T
T H T
T M M T Tc (1 M 2 ) (1 M 2 )
(1 M 2 )

(1 M 2 ) c
;
;

T
T H H
T
T

M
(1 M 2 )
M

;
2
H T Tc (1 M ) H
Q62.

ANS:B

H 0

(1 M 2 (0))
T Tc (1 M 2 (0))

Bose condensation occurs in liquid He 4 kept at ambient pressure at 2.17K. At which temperature
will Bose condensation occur in He 4 in gaseous state, the density of which is 1000 times smaller
than that of liquid He 4 ?(Assume that it is a perfect Bose gas.)
a) 2.17Mk
b) 21.7mK
c) 21.7K
d) 2.17K

UV PHYSICS ACADEMY
Shivam Road, New Nallakunta, Hyderabad, Ph: 04032458147, 09885072826
www.uvphysics.com
1
Exp: TBn 2 / 3 ;TB 2.17k ; ng
nl ;
100

Q63.

2/3

n
(T )
nl

(1000) 2 / 3 B liquid 100 ;


l
(TB ) gas ng
(TB ) gas
nl / 1000
(T )
2.17k
(TB ) gas B l
21.7mk
100
100
Consider black body radiation contained in a cavity whose walls are at temperature T. The
radiation is in equilibrium with the walls of the cavity. If the temperature of the walls is
increased to 2T and the radiation is allowed to come to come to equilibrium at the new
temperature, the entropy of the radiation increases by a factor of
a) 2
b) 4
c) 8
d) 16
(TB ) liquid

2/3

ANS:C
3

S T
S T 1
Exp: In black body radiation, entropy T ; ST ; 1 1 1 S 2 8S1
S 2 T2
S 2 2T 8
3

Q64.

The output, O, of the given circuit in cases I and II, where


Case I: A, B =1; C, D = 0; E, F = 1 and G = 0
Case II: A, B = 0; C, D = 0; E, F = 0 and G = 1 are respectively

a) 1, 0
b) 0, 1
c) 0, 0
d) 1, 1
ANS: D
Q65. A resistance strain gauge is fastened to a steel fixture and subjected to a stress of 1000kg / m 2 . If
the gauge factor is 3 and the modulus of elasticity of steel is 2 1010 kg / m 2 , then the fractional
change m resistance of the strain gauge due to the applied stress is
(Note: The gauge factor is defined as the ratio of the fractional change in length.)
a) 1.5 107
b) 3.0 107
c) 0.16 10 10
d) 0.5 10 2
ANS:A
stress
100kg / m 2
Young ' s mod ulus;
2 1010 kg / m 2 ; fractional changes in resistance = gauge
Exp:
strain
strain
factor x strain = 3 x 0.5 x 10-7=1.5 x 10-7
Q66. Consider a sinusoidal waveform of amplitude 1V and frequency f 0 . Starting from an arbitrary
initial time, the waveform is sampled at intervals of 1/ 2 f 0 . If the corresponding Fourier

spectrum peaks at a frequency f and an amplitude A, then

UV PHYSICS ACADEMY
Shivam Road, New Nallakunta, Hyderabad, Ph: 04032458147, 09885072826
www.uvphysics.com
a) f 2 f 0 and A 1V b) f f 0 and 0 A 1V c) f 0 and A 1V d) f

f0
1
and A
V
2
2

ANS:B
12
16
13
16
Q67. The first absorption spectrum of C O is at 3.842cm 1 while that of C O is at 3.673cm 1 . The
ratio of their moments of inertia is
a) 1.851
b) 1.286
c) 1.046
d) 1.038
ANS:C
B
I
12 16 192
13 16 208
h
1
Exp: we have B 2 ; B ; I r 2 ; 1 2 ; 1

; 2

;
I
B2 I1
12 16 28
13 16 29
8 Ic
1 192 29

1.046
2 28 208

Q68.

The spin-orbit interaction in an atom is given by H = a L.S, where L and S denote the orbital and
spin angular momenta, respectively, of the electron. The splitting between the
levels 2 P3 / 2 and 2 P1 / 2 is
3
1
5
a) a 2
b) a 2
c) 3a 2
d) a 2
2
2
2

ANS:A

a 2
J ( J 1) l (l 1) s(s 1)
Exp: H aL.S ; H a L.S
2
a 2
2
s 1/ 2, l 1, J 3 / 2
for P3/2 E
2
a 2
3
(a 2 ) a 2
for 2P1/2 E a 2 s 1/ 2, l 1, J 1/ 2; E
2
2
Q69. The spectral line corresponding to an atomic transition from J =1 to J = 0 states splits in a
magnetic field of 1kG into three component separated by 1.6 10 3 Ao . If the zero field spectral
line corresponds to 1849Ao, what is the g-factor corresponding to the J=1 state? (You may
hc
2 10 4 cm.)
use

a) 2

b) 3/2

c) 1

d)

ANS:C
hc
hc
1
hc
Exp: E g B B; E hv; E (hv) hc 2 | | 2 | | g B B; g
Given
B B 2


hc
2 10 4 cm; B 1103 G; 1849 10 10 , 1.6 10 3 10 10 ; g=1

UV PHYSICS ACADEMY
Shivam Road, New Nallakunta, Hyderabad, Ph: 04032458147, 09885072826
www.uvphysics.com
Q70.

The energy required to create a lattice vacancy in a crystal is equal to 1eV. The ratio of the
number densities of vacancies n(1200K)/n(300K), when the crystal is at equilibrium at 1200K
and 300K, respectively, is approximately
a) exp(-30)
b) exp(-15)
c) exp(15)
d) exp(30)

ANS:D

1.38 10 23
Exp: E = 1eV; k in eV
8.614 10 5 , no.of vacancies, n = N exp E / kT ;
19
1.6 10
n1200 exp 1/ 1200k
1
1
1 1

exp
exp

n300 exp( 1/ 300k )


4 10 2
k 400
8.614 10
1

exp
exp 29.022 The ratio of vacancies = exp[30]
3
33 10

Q71.

The dispersion relation of phonons in a solid is given


by 2 k 02 3 cos k1a cos k 2 a cos k3 a . The velocity of the phonons at large wavelength is

a) 0 a / 3
b) 0 a
c) 30 a
d) 0 a / 2
ANS: D
Exp: Dispersion relation is given by: 2 (k ) 02 (3 cos k x a cos k y a cos k z a) If we put kx = ky = kz=0

k 2 a 2 k y 2 a 2 k 2 a 2
1 z
1

(k ) 3 1 x
0 so as 0, cos 1

2!
2
2
2

2
0

2a2k 2
a2 2
a2 2
d 0 a
02 3 3
k x k y2 k z2 02
k x k y2 k z2 2 0
; v

2
2
2
dk
2

Consider an electron in a box of length L with periodic boundary condition ( x) x L . If

Q72.

2k 2
1 ikx
, what is the correction to its energy,
e with energy k
2m
L
to second order of perturbation theory, when it is subjected to a weak periodic
potential V ( x) V0 cos gx, where g is an integral multiple of the 2 / L ?
the electron is in the k ( x)

a) V02 k / 02 b)
ANS:B

mV02
1
1

c) V02 k g / 02
2
2 2
2 g 2kg g 2kg

d) V02 / k g

UV PHYSICS ACADEMY
Shivam Road, New Nallakunta, Hyderabad, Ph: 04032458147, 09885072826
www.uvphysics.com
1 V ( x) 2
2k 2
1 ikx
Exp: k ( x)
, ( 2)
e ;k
2 1
2m
L
L

V02
4 L2

ik1x
ik2 x
1 e V0 cos gxe
2
;
2
2
L
2k2
2 k1

2m
2m

e (e e
ik1 x

igx

igx

)e

ik 2 x

i ( k2 k1 g ) x dx e i ( k2 k1 g ) x dx 2

V0 2m e

; 2 2
2

4L
k 2 k12

2 2
(k 2 k12 )
2m
possible values of k1 and k2 ; k1 = k k 2 g and k1 k k 2 k g;

Q73.

( 2)

2
V0
mV0
2m L2
L2
2 2 2

; ( 2)
2
2
2
4 L k 2 k1 k 2 k1
2 2

The ground state of 82 Pb nucleus has spin-parity J P


207

1
1
g 2 2kg g 2 2kg

1
, where the first excited state
2

5
has J . The electromagnetic radiation emitted when the nucleus makes a transition from the
2
first excited state to the ground state are
a) E2 and E3
b) M2 and E3
c) E2 and M3
d) M2 and M3
P

ANS:C

Q74.

5
;
2
5
J i 1 / 2 J f and i f 1; | J i J f | L | J i J f | L 2,3
2
Since parity doesnt change in electric multipole transition, the possible Transitions are E2, M3
The dominant interactions underlying the following processes
A. K p

Exp: J i 1/ 2 J f

B. K K
C. p 0 are
a) A: strong, B: electromagnetic and C: weak
b) A: strong, B: weak and C: weak
c) A: weak, B: electromagnetic and C: strong
d) A: weak, B: electromagnetic and C: weak

ANS:A
Exp: K p since all quantum numbers are conserved, It is strong interaction.
K K Due to the presence of particle and anti particle, it is. p 0 , decay
of strange particle, it is weak.

UV PHYSICS ACADEMY
Shivam Road, New Nallakunta, Hyderabad, Ph: 04032458147, 09885072826
www.uvphysics.com
Q75.

If a Higgs boson of mass mH moving with a speed

v
decays into a pair of photons, then the
c

invariant mass of the photon pair is


2
[Note: The invariant mass of a system of two particles, with four-momenta p and p2 is p1 p2 ]
a) mH
ANS:B

b) mH

c) mH / 1 2

d) mH / 1 2

fiziks
Forum for CSIR-UGC JRF/NET, GATE, IIT-JAM, GRE in PHYSICAL SCIENCES
NET DEC- 2011(Physical Sciences)
PART A
Q1.

The most abundant element by mass in the human body is


(a) carbon

Q2.

(b) hydrogen

(c) calcium

(d) oxygen

A number system consists of digits 0, 1, 2, 3, 4 and 5. What is the decimal equivalent of


15 in this system?
(a) 15

Q3.

(b) 13

(c) 11

(d) 12

A segment of a circle (slightly greater than a semicircle, whose centre is O) is given


below. Identify the correct statement regarding the three angles A, B and C.

B
C

O
170 o
(a) A is equal to B but not equal to C
(b) A, B and C are equal and have a value of 85 o
(c) A, B and C are unequal
(d) A, B and C are each equal to 95o.
Q4.

After bubbling air through pure water (pH = 7.0), its pH decreased. Which of the
following is responsible for the pH change?
(a) Nitrogen

Q5.

(b) Carbon dioxide

(c) Oxygen

(d) Helium

Which of the following caused disruption of air-traffic in Europe after the volcanic
eruption in Iceland in the year 2010?
(a) Risk of engine damage by volcanic ash
(b) Increased local temperature
(c) Reduced visibility due to volcanic gases
(d) Presence of toxic gases

fiziks c/o Anand Institute of mathematics, 28-B/6 Jia Sarai


Near IIT, Hauz Khas, New Delhi, PIN- 110016 (INDIA)
Phone: 011-32718565, +91-9871145498
Website: http://www.physicsbyfiziks.com
Email: fiziks.physics@gmail.com

fiziks
Forum for CSIR-UGC JRF/NET, GATE, IIT-JAM, GRE in PHYSICAL SCIENCES
Q6.

Three boys A, B and C kicked three balls horizontally from the edge of the roof of a
building. The horizontal distances traversed by these balls before hitting the ground are
dA, dB, dC respectively, with dA > dB > d C. If tA, t B and tC are the times taken to hit the
ground respectively then
(a) tA > tB > tC

Q7.

(b) tA < tB < tC

(c) tA > tB < tC

(d) tA = tB = tC

What is the minimum number of cards you need to uncover from the top of a wellshuffled deck of 52 playing cards, to ensure that you have two cards of a suit?
(a) 41

Q8.

(b) 15

(c) 7

(d) 5

A swinging door is to be stopped by driving a wedge between the floor and the door. The
most favourable position of the wedge is close to the
(a) outer edge of the door because even a small frictional force can provide adequate
torque to stop the door swing.
(b) outer edge of the door because the frictional force is the largest at the outer edge.
(c) hinged edge of the door because the moment of force is smallest near the hinge.
(d) hinged edge of the door because there is friction in the hinge.

Q9.

The simple representation of a detergent molecule is given below:

ionic
head

tail

Which one of the following representations describes the interaction of the detergent
molecule in a system composed of grease and water?
(a)

Air

(b)

Water

Water

Grease

Grease

Air

(c)

Air

Air

(d)

Water
Grease

Water
Grease

fiziks c/o Anand Institute of mathematics, 28-B/6 Jia Sarai


Near IIT, Hauz Khas, New Delhi, PIN- 110016 (INDIA)
Phone: 011-32718565, +91-9871145498
Website: http://www.physicsbyfiziks.com
Email: fiziks.physics@gmail.com

fiziks
Forum for CSIR-UGC JRF/NET, GATE, IIT-JAM, GRE in PHYSICAL SCIENCES
Q10.

The figure shows cumulative weight percent curves for different types of sediments.

Cumulative wt ( %)

Which type of sediment has the narrowest size distribution?


100

Flood
Gravel

0
(a) Dune sand
Q11.

(b) Glacial till

Glacial
Tilt

Loess

Size
(c) Flood gravel

(d) Loess

Which of the following animals does not have modified legs used for flight?
(a) Sparrow

Q12.

Dune
Sand

(b) Bat

(c) Flying squirrel

(d) Butterfly

A polypeptide of 300 amino acids has tyrosine at the 157th position. If the tyrosine codon
mutates to a nonsense codon, what would be the size of the polypeptide in this mutated
organism?
(a) 157

Q13

(b) 156

(c) 299

(d) 144

A room has a west-facing window with glass panes. Which of the following is the most
effective way to prevent the room from getting hot in summer?
(a) Cover the inside of the glass pane by a black paper.
(b) Cover the outside of the glass pane by an aluminum foil.
(c) Cover the outside of the glass pane by a white thermocol sheet.
(d) Cover the inside of the glass pane by a white thermocol sheet.

Q14.

A jar containing an iron block B was floating on water in a bigger container. The block
was taken out and put into water. As a result, the level of water in the container will

(a) rise
B

(b) fall
(c) remain the same
(d) rise or fall depending upon the mass of the block.

fiziks c/o Anand Institute of mathematics, 28-B/6 Jia Sarai


Near IIT, Hauz Khas, New Delhi, PIN- 110016 (INDIA)
Phone: 011-32718565, +91-9871145498
Website: http://www.physicsbyfiziks.com
Email: fiziks.physics@gmail.com

fiziks
Forum for CSIR-UGC JRF/NET, GATE, IIT-JAM, GRE in PHYSICAL SCIENCES

Q15.

Suppose the Sun somehow becomes a black hole without change in its mass. Then this
black hole will pull into itself
(a) all the planets
(b) only Mercury
(c) all planets from Mercury to Mass
(d) none of the planets

Q16.

Absolute water content in the air in the equatorial region is measured to be 18 g m-3 and
the same is 4 g m-3 in the polar region. However, the values of relative humidity reported
are 60% and 78%, respectively. This could be because
(a) equatorial region is warmer and therefore the atmosphere has a higher water holding
capacity.
(b) winds are stronger in the polar region
(c) polar region is ice-covered and therefore its atmosphere has a higher water content
(d) of higher snowfall in the polar region.

Q17.

Oxidation of alcohols to acids involves formation and cleavage of bonds. Which of the
following possibilities is valid in the process?
(a) Formation of C = O bond and cleavage of O H and C H bonds
(b) Formation of C = O bond and cleavage of O H bond.
(c) Formation of C = O bonds and cleavage of C H bonds.
(d) Formation of C O bond and cleavage of O H and C H bonds.

fiziks c/o Anand Institute of mathematics, 28-B/6 Jia Sarai


Near IIT, Hauz Khas, New Delhi, PIN- 110016 (INDIA)
Phone: 011-32718565, +91-9871145498
Website: http://www.physicsbyfiziks.com
Email: fiziks.physics@gmail.com

fiziks
Forum for CSIR-UGC JRF/NET, GATE, IIT-JAM, GRE in PHYSICAL SCIENCES
Q18.

Droplets of a herbicide solution form various shapes on a leaf as shown.

Assuming that the droplets have the same volume, the trend in the rates of herbicide
uptake would be
(a) A > B > C

Q19.

(b) B > A > C

(c) B > C > A

(d) C > A > B

Parents with blood groups A and AB have two children. Which of the following cannot
be the blood groups of their children?
(a) A and AB

Q20.

(b) B and AB

(c) A and B

(d) O and B

A long cylinder has an axially placed two-bladed fan spinning inside it. Bullets are shot
through the cylinder at a constant rate. If the number of blades is increased to four, the
number of bullets
(a) missing the blades is halved
(b) missing the blades is reduced by one-fourth
(c) hitting the blades is doubled
(d) hitting the blades remains the same.

fiziks c/o Anand Institute of mathematics, 28-B/6 Jia Sarai


Near IIT, Hauz Khas, New Delhi, PIN- 110016 (INDIA)
Phone: 011-32718565, +91-9871145498
Website: http://www.physicsbyfiziks.com
Email: fiziks.physics@gmail.com

fiziks
Forum for CSIR-UGC JRF/NET, GATE, IIT-JAM, GRE in PHYSICAL SCIENCES
Part B
Q21.

Consider three polarizers P1, P2 and P3 placed along an axis as shown in the figure.
P2

P1

P3

(unpolarized)
I0

The pass axis of P1 and P3 are at right angles to each other while the pass axis of P2
makes an angle with that of P1. A beam of unpolarized light of intensity I0 is incident
on P1 as shown. The intensity of light emerging from P3 is
(a) 0
Q22.

(b)

I0
2

(c)

I0
sin 2 2
8

(d)

I0
sin 2 2
4

A double pendulum consists of two point masses m attached by strings of length l as


shown in the figure:
The kinetic energy of the pendulum is
(a)
(b)
(c)
(d)

1 2 2 2
ml 1 2
2

1 2 2 2
ml 21 2 212 cos1 2
2

1 2 2
ml 1 222 212 cos 1 2
2

1 2 2 2
ml 21 2 212 cos 1 2
2

Q.23. In the operational amplifier circuit below, the voltage at point A is


5V
1K
A
1V

1V
1K
1K 5V

(a) 1.0 V

(b) 0.5 V

(c) 0 V

(d) 5.0 V

fiziks c/o Anand Institute of mathematics, 28-B/6 Jia Sarai


Near IIT, Hauz Khas, New Delhi, PIN- 110016 (INDIA)
Phone: 011-32718565, +91-9871145498
Website: http://www.physicsbyfiziks.com
Email: fiziks.physics@gmail.com

fiziks
Forum for CSIR-UGC JRF/NET, GATE, IIT-JAM, GRE in PHYSICAL SCIENCES
Q24.

A point particle of mass m carrying an electric charge q is attached to a spring of


stiffness constant k. A constant electric field E along the direction of the spring is
switched on for a time interval T (where T m / k ). Neglecting radiation loss, the
amplitude of oscillation after the field is switched off is:
(b) qET 2 / m

(a) qE / k

(c) qE m / Tk 3 / 2

(d) qET / mk

Q.25. A constant force F is applied to a relativistic particle of rest mass m. If the particle starts
from rest at t = 0, its speed after a time t is

Ft
(b) c tanh

mc

(a) Ft / m

Q26.

(c) c 1 e Ft / mc

(d)

Fct
2 2

F t m 2c 2

The potential of a diatomic molecule as a function of the distance r between the atoms is
given by V r

a
b
12 . The value of the potential at equilibrium separation between
6
r
r

the atoms is:


(a) 4a 2 / b

Q27.

(b) 2a 2 / b

(c) a 2 / 2b

(d) a 2 / 4b

Four equal point charges are kept fixed at the four vertices of a square. How many neutral
points (i.e. points where the electric field vanishes) will be found inside the square?
(a) 3

Q28.

(b) 4

(c) 5

(d) 7

A static charge distribution gives rise to an electric field of the form E 1 e r / R

r
,
r2

where and R are positive constants. The charge contained within a sphere of radius R,
centred at the origin is
(a) 0

e
R2

(b) 0

e2
R2

(c) 4 0

R
e

(d) 0

R2
e

fiziks c/o Anand Institute of mathematics, 28-B/6 Jia Sarai


Near IIT, Hauz Khas, New Delhi, PIN- 110016 (INDIA)
Phone: 011-32718565, +91-9871145498
Website: http://www.physicsbyfiziks.com
Email: fiziks.physics@gmail.com

fiziks
Forum for CSIR-UGC JRF/NET, GATE, IIT-JAM, GRE in PHYSICAL SCIENCES
Q29.

A counter consists of four flip-flops connected as shown in the figure:


A0
A1
A2
A3
J Q

J Q

J Q

J Q

K Q

K Q

K Q

K Q

CLK

If the counter is initialized as A0 A1 A2 A3 = 0110, the state after the next clock pulse is
(a) 1000
Q30.

(b) 0001

(c) 0011

(d) 1100

In a Youngs double slit interference experiment, the slits are at a distance 2L from each
other and the screen is at a distance D from the slits. If a glass slab of refractive index
and thickness d is placed in the path of one of the beams, the minimum value of d for the
central fringe to be dark is
(a)

Q31.

(b)

D L

D
1L

(c)

(d)

2 1

The pins 0, 1, 2 and 3 of part A of a microcontroller are connected with resistors to drive
an LED at various intensities as shown in the figure.
V CC

A3

A2
A1
A0

0 . 75 k

1 .5 k
3k
6k

For VCC = 4.2 V and a voltage drop of 1.2 V across the LED, the range (maximum
current) and resolution (step size) of the drive current are, respectively,
(a) 4.0 mA and 1.0 mA

(b) 15.0 mA and 1.0 mA

(c) 7.5 mA and 0.5 mA

(d) 4.0 mA and 0.5 mA

fiziks c/o Anand Institute of mathematics, 28-B/6 Jia Sarai


Near IIT, Hauz Khas, New Delhi, PIN- 110016 (INDIA)
Phone: 011-32718565, +91-9871145498
Website: http://www.physicsbyfiziks.com
Email: fiziks.physics@gmail.com

fiziks
Forum for CSIR-UGC JRF/NET, GATE, IIT-JAM, GRE in PHYSICAL SCIENCES
Q32.

An unbiased dice is thrown three times successively. The probability that the numbers of
dots on the uppermost surface add up to 16 is
(a)

Q33.

1
16

1
36

(b)

(c)

1
108

(d)

1
216

The generating function

F x, t Pn x t n
n 0

for the Legendre polynomials Pn x is F x, t 1 2 xt t 2


(a) 5/2

Q34.

(b) 3/2

12

. The value of P3 1 is

(c) + 1

(d) - 1

Given that the ground state energy of the hydrogen atom is 13.6 eV, the ground state
energy of positronium (which is a bound state of an electron and a positron) is
(a) + 6.8 eV

Q35.

(b) 6.8 eV

(c) 13.6 eV

(d) 27.2 eV

Two particles of identical mass move in circular orbits under a central


potential V r

1 2
kr . Let l1 and l2 be the angular momenta and r1, r2 be the radii of the
2

orbits respectively. If l1/l 2 = 2, the value of r1 / r2 is:


(a)

Q36.

(b) 1 / 2

(d) 1 / 2

The equation of the plane that is tangent to the surface xyz = 8 at the point (1, 2, 4) is
(a) x + 2y + 4z = 12

Q37.

(c) 2

(b) 4x + 2y + z = 12

(c) x + 4y + 2 = 0

(d) x + y + z = 7

The energy of the first excited quantum state of a particle in the two-dimensional
potential V x, y
(a) 2

1
m 2 x 2 4 y 2 is
2

(b) 3

(c)

(d)

fiziks c/o Anand Institute of mathematics, 28-B/6 Jia Sarai


Near IIT, Hauz Khas, New Delhi, PIN- 110016 (INDIA)
Phone: 011-32718565, +91-9871145498
Website: http://www.physicsbyfiziks.com
Email: fiziks.physics@gmail.com

fiziks
Forum for CSIR-UGC JRF/NET, GATE, IIT-JAM, GRE in PHYSICAL SCIENCES
Q38.

The internal energy E of a system is given by E

bS 3
, where b is a constant and other
VN

symbols have their usual meaning. The temperature of this system is equal to

bS 2
(a)
VN
Q39.

3bS 2
(b)
VN

bS 3
(c) 2
V N

S
(d)
N

Consider a particle in a one dimensional potential that satisfies V x V x . Let 0

and

denote the ground and the first excited states, respectively, and let

0 0 1 1 be a normalized state with 0 and 1 being real constants. The


expectation value x of the position operator x in the state is given by

Q40.

(a) 02 0 x 0 12 1 x 1

(b) 0 1 0 x 1 1 x 0

(c) 02 12

(d) 2 0 1

A 3 3 matrix M has Tr[M] = 6, Tr[M 2] = 26 and Tr[M 3] = 90. Which of the following
can be a possible set of eigenvalues can be a possible set of eigenvalues of M?
(a) {1, 1, 4}

Q41.

(b) {-1, 0, 7}

(c) {-1, 3, 4}

(d) {2, 2, 2}

The perturbation H ' bx 4 , where b is a constant, is added to the one dimensional


harmonic oscillator potential V x

1
m 2 x 2 . Which of the following denotes the
2

correction to the ground state energy to first order in b?


[Hint: The normalized ground state wave function of the one dimensional harmonic
m
oscillator potential is 0

2n

ax 2

dx a

(a)

3b 2
4 m 2 2

1
2

1/ 4

e m x

/ 2

. You may use the following integral

n .
2

(b)

3b 2
2 m 2 2

(c)

3b 2
2 m 2 2

(d)

15b 2
4 m 2 2

fiziks c/o Anand Institute of mathematics, 28-B/6 Jia Sarai


Near IIT, Hauz Khas, New Delhi, PIN- 110016 (INDIA)
Phone: 011-32718565, +91-9871145498
Website: http://www.physicsbyfiziks.com
Email: fiziks.physics@gmail.com

10

fiziks
Forum for CSIR-UGC JRF/NET, GATE, IIT-JAM, GRE in PHYSICAL SCIENCES

Q42.

A battery powers two circuits C1 and C2 as shown in the figure.

C2

C1

I2

I
I1

The total current I drawn from the battery is estimated by measuring the currents I1 and I2
through the individual circuits. If I1 and I2 are both 200 mA and if the errors in their
measurement are 3 mA and 4 mA respectively, the error in the estimate of I is
(a) 7.0 mA

Q43.

(b) 7.5 mA

(c) 5.0 mA

(d) 10.5 mA

Consider a Maxwellian distribution of the velocity of the molecules of an ideal gas. Let
Vmp and Vrms denote the most probable velocity and the root mean square velocity,
respectively. The magnitude of the ratio Vm / Vrms is
(a) 1

Q44.

(b) 2/3

(c)

2/3

(d) 3 / 2

If the number density of a free electron gas in three dimensions is increased eight times,
its Fermi temperature will
(a) increase by a factor of 4

(b) decrease by a factor of 4

(c) increase by a factor of 8

(d) decrease by a factor of 8

fiziks c/o Anand Institute of mathematics, 28-B/6 Jia Sarai


Near IIT, Hauz Khas, New Delhi, PIN- 110016 (INDIA)
Phone: 011-32718565, +91-9871145498
Website: http://www.physicsbyfiziks.com
Email: fiziks.physics@gmail.com

11

fiziks
Forum for CSIR-UGC JRF/NET, GATE, IIT-JAM, GRE in PHYSICAL SCIENCES
Q45.

A system of N noninteracting spin -

1
particles is placed in an external magnetic field H.
2

The behaviour of the entropy of the system as a function of energy is given by


(a)

(b)

B H

BH

(c)

B H

E B H
S

BH
(d)

B H

BH

fiziks c/o Anand Institute of mathematics, 28-B/6 Jia Sarai


Near IIT, Hauz Khas, New Delhi, PIN- 110016 (INDIA)
Phone: 011-32718565, +91-9871145498
Website: http://www.physicsbyfiziks.com
Email: fiziks.physics@gmail.com

12

fiziks
Forum for CSIR-UGC JRF/NET, GATE, IIT-JAM, GRE in PHYSICAL SCIENCES
PART C
Q46.

A particle of mass m moves inside a bowl. If the surface of the bowl is given by the
equation z

Q47.

1
a x 2 y 2 , where a is a constant, the Lagrangian of the particle is
2

(a)

1
m r 2 r 2 2 gar 2
2

(b)

1
m 1 a 2 r 2 r 2 r 2 2
2

(c)

1
m r 2 r 2 2 r 2 sin 2 2 gar 2
2

(d)

1
m 1 a 2 r 2 r 2 r 2 2 gar 2
2

An electron of energy 27 GeV collides with a proton of energy 820 GeV. The heaviest
particle which can be produced in this collision has mass close to
(a) 300 GeV

Q48.

(b) 821 GeV

(c) 850 GeV

(d) 1127 GeV

Let x1 t and x2 t be two linearly independent solutions of the differential equation


dx t
dx t
d2x
dx
2 f t x 0 and let wt x1 t 2 x 2 t 1 . If w(0) = 1, then w(1) is
2
dt
dt
dt
dt

given by
(b) e2

(a) 1

Q49.

(d) 1/e2

(c) 1/e

Assume that the free energy of a magnetic system has an expansion in the order
parameter M of the form F(M,T) = a(T TC) M2 + bM4 +cM6, with a, b and c > 0. As the
temperature is lowered below TC, the system undergoes a phase transition. The behaviour
of the order parameter just below the transition, where (TTC) is very small, is best
described by
(a) M TC T
(c) M TC T

1 / 2

(b) M TC T

1/ 2

(d) M TC T

fiziks c/o Anand Institute of mathematics, 28-B/6 Jia Sarai


Near IIT, Hauz Khas, New Delhi, PIN- 110016 (INDIA)
Phone: 011-32718565, +91-9871145498
Website: http://www.physicsbyfiziks.com
Email: fiziks.physics@gmail.com

13

fiziks
Forum for CSIR-UGC JRF/NET, GATE, IIT-JAM, GRE in PHYSICAL SCIENCES
Q50.

A planet of mass m moves in the inverse square central force field of the Sun of mass M.
If the semi-major and semi-minor axes of the orbit are a and b, respectively, the total
energy of the planet is:

Q51.

(a)

GMm
ab

1 1
(b) GMm
a b

(c)

GMm 1 1

a b a

ab
(d) GMm
2
a b

Let 0 and 1 denote the normalized eigenstates corresponding to the ground and first
excited states of a one dimensional harmonic oscillator. The uncertainty p in the
state

Q52.

1
2

1 , is

(a) p m / 2

(b) p m / 2

(c) p m

(d) p 2m

A laser operating at 500 nm is used to excite a molecule. If the Stokes line is observed at
770 cm-1, the approximate positions of the Stokes and the anti-Stokes lines are

Q53.

(a) 481.5 nm and 520 nm

(b) 481.5 nm and 500 nm

(c) 500 nm and 520 nm

(d) 500 nm and 600 nm

The graph of the function


for 2n x 2n 1

1
f x
0

for 2n 1 x 2n 2

where n = 0, 1, 2, .) is shown below.


f x
1
0

~
Its Laplace transform f s is

(a)

1 e s
s

(b)

1 e s
s

(c)

1
s 1 e s

(d)

1
s 1 e s

fiziks c/o Anand Institute of mathematics, 28-B/6 Jia Sarai


Near IIT, Hauz Khas, New Delhi, PIN- 110016 (INDIA)
Phone: 011-32718565, +91-9871145498
Website: http://www.physicsbyfiziks.com
Email: fiziks.physics@gmail.com

14

fiziks
Forum for CSIR-UGC JRF/NET, GATE, IIT-JAM, GRE in PHYSICAL SCIENCES
Q54.

The energy levels of electrons of mass m and charge e confined in an area A in the xyplane with a uniform magnetic field B applied in the z-direction are given by

1 eB
eBA

En n
, n 0,1,2,.... The degeneracy of each level is
. The lowest level is
2 mc
c

completely filled and the others are empty. The Fermi energy

2 N
, where N is the total
2mA

number of electrons, is
(a) coincident with the n = 0 level
(b) coincident with the n = 1 level
(c) midway between the n = 0 and the n = 1 levels
(d) midway between the n = 1 and the n = 2 levels

Q55.

An annulus of mass M made of a material of uniform density has inner and outer radii a
and b respectively. Its principle moment of inertia along the axis of symmetry
perpendicular to the plane of the annulus is :

(b)

1
M b 2 a 2
2

(d)

1
M b2 a2
2

(a)

1
b4 a4
M 2
2
b a2

(c)

1
M b2 a2
2

Q56.

The value of the real integral I

(a)

Q57.

3
2

2
(b) ln
5

dx
is
3x 2

(c)

(d) 0

The minimum energy of an electron (the rest mass of which is 0.5 MeV) that can emit
Cerenkov radiation while passing through water (of refractive index 1.5) is approximately
(a) 1.0 MeV

(b) 3.0 MeV

(c) 0.6 MeV

(d) 0.5 MeV

fiziks c/o Anand Institute of mathematics, 28-B/6 Jia Sarai


Near IIT, Hauz Khas, New Delhi, PIN- 110016 (INDIA)
Phone: 011-32718565, +91-9871145498
Website: http://www.physicsbyfiziks.com
Email: fiziks.physics@gmail.com

15

fiziks
Forum for CSIR-UGC JRF/NET, GATE, IIT-JAM, GRE in PHYSICAL SCIENCES
Q58.

A heater and a thermocouple are used to measure and control the temperature T of a
sample at T0 = 250o C. A feedback circuit supplies power P to the heater according to the
equation P P0 G T0 T D

dT
with approximately tuned values of the coefficients
dt

G and D. In order to maintain temperature stability in the presence of an external heat


perturbation which causes small but rapid fluctuations of temperature, it is necessary to
(a) decrease D

Q59.

(b) increase D

(c) decrease G

(d) increase G

The trajectory on the zpz-plane (phase-space trajectory) of a ball bouncing perfectly


elastically off a hard surface at z = 0 is given by approximately by (neglect friction):
(a)

PZ

(b)

PZ

(c)

(d)

PZ

PZ

Q60.

According to the shell model the spin and parity of the two nuclei

125
51

Sb and

89
38

Sr are,

respectively,

5
5
(a) and
2
2

7
5
(c) and
2
2

5
7
(b) and
2
2

7
7
(d) and
2
2

fiziks c/o Anand Institute of mathematics, 28-B/6 Jia Sarai


Near IIT, Hauz Khas, New Delhi, PIN- 110016 (INDIA)
Phone: 011-32718565, +91-9871145498
Website: http://www.physicsbyfiziks.com
Email: fiziks.physics@gmail.com

16

fiziks
Forum for CSIR-UGC JRF/NET, GATE, IIT-JAM, GRE in PHYSICAL SCIENCES
Q61.

The wave function of a particle at time t = 0 is given by 0


u1 and u 2

Q62.

1
2

u 2 , where

are the normalized eigenstates with eigenvalues E1 and E2 respectively,

E 2

E1 . The shortest time after which t will become orthogonal to 0 is

(a)


2E 2 E1

(b)

E 2 E1

(c)

2
E 2 E1

(d)

2
E 2 E1

A gas of N non-interacting particles is in thermal equilibrium at temperature T. Each


particle can be in any of the possible non-degenerate states of energy 0, 2 and 4. The
average energy per particle of the gas, when << 1, is
(a) 2

Q63.

(b) 3

(d)

(c) 2 / 3

The first few terms in the Taylor series expansion of the function f(x) = sinx around

are
4

2
3

1
1

(a)
1 x x x .....
4 2!
4 3!
4
2

2
3

1
1

(b)
1 x x x .....
4 2!
4
3!
4
2

(c) x x .....
4 3!
4

(d)
Q64.

1
x 2 x3
1

.....

2! 3!
2

A one-dimensional chain consists of a set of N rods each of length a. When stretched by a


load, each rod can align either parallel or perpendicular to the length of the chain. The
energy of a rod is when perpendicular to it. When the chain is in thermal equilibrium
at temperature T, its average length is
(a) Na / 2

(b) Na

(c) Na / 1 e 2 / kBT

(d) Na 1 e 2 / k BT

fiziks c/o Anand Institute of mathematics, 28-B/6 Jia Sarai


Near IIT, Hauz Khas, New Delhi, PIN- 110016 (INDIA)
Phone: 011-32718565, +91-9871145498
Website: http://www.physicsbyfiziks.com
Email: fiziks.physics@gmail.com

17

fiziks
Forum for CSIR-UGC JRF/NET, GATE, IIT-JAM, GRE in PHYSICAL SCIENCES
Q65.

If the hyperfine interaction in an atom is given by H a S e S p where S e and S p denote


the electron and proton spins, respectively, the splitting between the 3 S1 and 1 S 0 state is
(b) a 2

(a) a 2 / 2
Q66.

(c) a 2 / 2

(d) 2 a 2

Consider a solenoid of radius R with n turns per unit length, in which a time dependent
current I = I0 sin t (where R/c << 1) flows. The magnitude of the electric field at a
perpendicular distance r < R from the axis of symmetry of the solenoid, is
(a) 0
(c)

Q67.

1
0 nI 0 r sin t
2

(b)

1
0 nI 0 R 2 cos t
2r

(d)

1
0 nI 0 r cos t
2

The difference in the Coulomb energy between the mirror nuclei

49
24

Cr and

49
25

Mn is

6.0 MeV . Assuming that the nuclei have a spherically symmetric charge distribution and
that e2 is approximately 1.0 MeV-fm, the radius of the

Q68.

Mn nucleus is

(a) 4.9 10-13 m

(b) 4.9 10-15 m

(c) 5.1 10-13 m

(d) 5.1 10-15 m

The ratio of intensities of the D1 and D2 lines of sodium at high temperature is


(a) 1:1

Q69.

49
25

(b) 2:3

(c) 1:3

(d) 1:2

A constant perturbation as shown in the figure below acts on a particle of mass m


confined in an infinite potential well between 0 and L.

V0

V0
2

L/2

The first-order correction to the ground state energy of the particle is


(a)

V0
2

(b)

3V0
4

(c)

V0
4

(d)

3V0
2

fiziks c/o Anand Institute of mathematics, 28-B/6 Jia Sarai


Near IIT, Hauz Khas, New Delhi, PIN- 110016 (INDIA)
Phone: 011-32718565, +91-9871145498
Website: http://www.physicsbyfiziks.com
Email: fiziks.physics@gmail.com

18

fiziks
Forum for CSIR-UGC JRF/NET, GATE, IIT-JAM, GRE in PHYSICAL SCIENCES
Q70.

An atom of mass M can be excited to a state of mass M+ by photon capture. The


frequency of a photon which can cause this transition is

Q71.

(a) c2 / 2h

(b) c2 / h

(c) 2 c2 / 2Mh

(d) ( + 2 M)c2 / 2Mh

The excitations of a three-dimensional solid are bosonic in nature with their frequency
and wave-number k are related by k2 in the large wavelength limit. If the chemical
potential is zero, the behaviour of the specific heat of the system at low temperature is
proportional to
(a) T1/2

Q72.

(c) T3/2

(b) T

(d) T3

Gas molecules of mass m are confined in a cylinder of radius R and height L (with

R L ) kept vertically in the Earths gravitational field. The average energy of the gas
at low temperatures (such that mgL >> kBT) is given by
(a) NkBT / 2

(c) 2NkBT

(d) 5NkBT / 2

The figure below shows a voltage regulator utilizing a Zener diode of breakdown voltage
5 V and a positive triangular wave input of amplitude 10 V.
500
Vi
12
10

i
1K

i(mA)

Q73.

(b) 3NkBT / 2

8
6
4
2
0
0

t (s)
For Vi > 5V, the Zener regulates the output voltage by channeling the excess current
through itself. Which of the following waveforms shows the current i passing through the
Zener diode?

fiziks c/o Anand Institute of mathematics, 28-B/6 Jia Sarai


Near IIT, Hauz Khas, New Delhi, PIN- 110016 (INDIA)
Phone: 011-32718565, +91-9871145498
Website: http://www.physicsbyfiziks.com
Email: fiziks.physics@gmail.com

19

fiziks
Forum for CSIR-UGC JRF/NET, GATE, IIT-JAM, GRE in PHYSICAL SCIENCES
12
10
8
6
4
2
0
0

i(mA)

(b)
i(mA)

(a)

12
10
8
6
4
2
0
0

t (s )

(d)
i(mA)

i(mA)

(c) 12
10
8
6
4
2
0
0

12
10
8
6
4
2
0
0

t (s)

t (s)
Q74.

t (s)

A constant electric current I in an infinitely long straight wire is suddenly switched on at


t = 0. The vector potential at a perpendicular distance r from the wire is given
by A

k 0 I 1

ln ct c 2 t 2 r 2 . The electric field at a distance r (< ct) is


2
r

(a) 0

(c)

Q75.

(b)
c 0 I

2 c 2 t 2 r 2

1
ij
2

(d)

0 I 1
ij
2 t 2

c 0 I
2 c 2 t 2 r 2

Monochromatic light of wavelength 660 nm and intensity 100 mW/cm2 falls on a solar
cell of area 30 cm2. The conversion efficiency of the solar cell is 10%. If each converted
photon results in an electron-hole pair, what is the maximum circuit current supplied by
the solar cell? (Take h = 6.6 10-34 J-s, c = 3 108 m/s and e = 1.6 10 -19 C).
(a) 160 mA

(b) 320 mA

(c) 1600 mA

(d) 3200 mA

fiziks c/o Anand Institute of mathematics, 28-B/6 Jia Sarai


Near IIT, Hauz Khas, New Delhi, PIN- 110016 (INDIA)
Phone: 011-32718565, +91-9871145498
Website: http://www.physicsbyfiziks.com
Email: fiziks.physics@gmail.com

20

UV PHYSICS ACADEMY
Shivam Road, New Nallakunta, Hyderabad, Ph: 04032458147, 09885072826
www.uvphysics.com
CSIR DEC-2011
Q1. The most abundant element by mass in the human body is
a) carbon
b) hydrogen
c) Calcium
d) Oxygen
Ans: d
Q2. A number of system consists of digits 0, 1, 2, 3, 4 and 5. What is the decimal equivalent of 15 in this
number system.
a) 15
b) 13
c) 11
d) 12
Ans: c
Q3. A segment of circle (slightly greater than a semicircle, whose centre is O) is given below. Identify
the correct statement regarding the three angles A, B and C.

a) A is equal to B but not equal to C.


b) A,B and C are equal and have a value of 85 0
c) A, B and C are unequal.
d) A, B and C are each equal to 950 .
Ans: b
Q4. After bubbling air through pure water (pH = 7.0), its pH decreased. Which of the following is
responsible for the pH change
a) Nitrogen
b) Carbon dioxide
c) Oxygen
d) Helium
Ans: b
Q5. Which of the following caused disruption of air-traffic in Europe after the volcanic eruption in
Iceland in the year 2010
a) Risk of engine damage by volcanic ash.
b) Increased local temperature.
c) Reduced visibility due to volcanic gases. d) Presence of toxic gases.
Ans: a
Q6. Three boys A, B and C kicked three balls horizontally from the edge of the roof of a building. The
horizontal distances traversed by these balls before hitting the ground are d A , d B , d C respectively,
with d A d B d C If t A , t B and tC are the times taken to hit the ground respectively, then
a) t A t B t C
b) t A t B t C
c) t A t B t C
d) t A t B tC
Ans: d
Q7. What is the minimum number of .cards you need to uncover from the top of a well-shuffled deck of
52 playing cards, to ensure that you have two cards of a suit
a) 41
b) 15
c) 7
d) 5
Ans: a
Q8. A swinging door is to be stopped by driving a wedge between the floor and the door. The most
favorable position of the wedge is close to the
a) outer edge of the door because even a small frictional force can provide adequate torque to stop
the door swing.
b) outer edge of the door because the frictional force is the largest at the outer edge.
c) hinged edge of the door because the moment of force is smallest near the hinge.
d) hinged edge of the door because there is friction in the hinge.
Ans: a

UV PHYSICS ACADEMY
Shivam Road, New Nallakunta, Hyderabad, Ph: 04032458147, 09885072826
www.uvphysics.com
Q9. The simple representation of a detergent molecule is given below:
Which one of the following representations describes the interaction of the detergent molecule in a
system composed of grease and water.

a)
b)
c)
d)
Ans: c
Q10. The figure shows cumulative weight percent curves for different types of sediments. Which type of
sediment has the narrowest size distribution?

a) Dune sand
b) Glacial till
c) Flood gravel
d) Loess
Ans: a
Q11. Which of the following animals does not have modified legs used for flight?
a) Sparrow
b) Bat
c) Flying squirrel
d) Butterfly
Ans: a
Q12. A polypeptide of 300 amino acids has tyrosine at the 157th position. If the tyrosine codon mutates
to a nonsense codon, what would be the size of the polypeptide in this mutated organism?
a) 157
b) 156
c) 299
d) 144
Ans: c
Q13. A room has a west-facing window with glass panes. Which of the following is the most effective
way to prevent the room from getting hot in summer?
a) Cover the inside of the glass pane by a black paper.
b) Cover the outside of the glass pane by an aluminum foil.
c) Cover the outside of the glass pane by a white thermocol sheet.
d) Cover the inside of the glass pane by a white thermocol sheet.
Ans: c
Q14. A jar containing an iron block B was floating on water in a bigger container. The block was taken
out and put into water. As a result, the level of water in the container will

a) rise b) fall
c) remain the same d) rise or fall depending upon the mass of the block
Ans: b
Q15. Suppose the Sun somehow becomes a black hole without change in its mass. Then this black hole
will pull into itself
a) all the planets b) only Mercury c) all planets from Mercury to Mars d) none of the planets
Ans: d

UV PHYSICS ACADEMY
Shivam Road, New Nallakunta, Hyderabad, Ph: 04032458147, 09885072826
www.uvphysics.com
Q16. Absolute water content in the air in the equatorial region is measured to be 18 g irf3 and the same is
4 g irf3 in the polar region. However, the values of relative humidity reported are 60% and 78%,
respectively. This could be because
a) equatorial region is warmer and therefore the atmosphere has a higher water holding capacity.
b) winds are stronger in the polar region.
c) polar region is ice-covered and therefore its atmosphere has a higher water content.
d) of higher snowfall in the polar region.
Ans: a
Q17. Oxidation of alcohols to acids involves formation and cleavage of bonds. Which of the following
possibilities is valid in the process?
a) Formation of C=O bond and cleavage of O-H and C-H bonds.
b) Formation of C=O bonds and cleavage of O-H bond.
c) Formation of C=O bonds and cleavage of C-H bonds.
d) Formation of C=O bond and cleavage of O-H and C-H bonds.
Ans: c
Q18. Droplets of a herbicide solution form various shapes on a leaf as shown.
Assuming that the droplets have the same volume, the trend in the rates of herbicide uptake would be

a) A> B > C
b) B > A > C
c) B > C > A
d) C > A > B
Ans: c
Q19. Parents with blood groups A and AB have two children. Which of the following cannot be the
blood groups of their children?
a) A and AB
b) B and AB
c) A and B
d) O and B.
Ans: d
Q20. A long cylinder has an axially placed two-bladed fan spinning inside it. Bullets are shot through
the cylinder at a constant rate. If the number of blades is increased to four, the number of bullets
a) missing the blades is halved.
b) missing the blades is reduced by one-fourth.
c) hitting the blades is doubled.
d) hitting the blades remains the same.
Ans: d

UV PHYSICS ACADEMY
Shivam Road, New Nallakunta, Hyderabad, Ph: 04032458147, 09885072826
www.uvphysics.com
PART-B (COMPULSORY)
EACH QUESTION CARRIES 3.5 MARKS
EACH WRONG ANSWER HAS A PENALTY OF 25% OF MARKS
Q21. Consider three polarizers P1 , P2 and P3 placed along an axis as shown in the figure.
The pass axis of P1 and P3 are at right angles to each other while pass axis of P2 makes an angle
with that of P1 . A beam of unpolarized light of intensity I 0 is incident on P1 as shown. The intensity
of light emerging from P3 is

a) 0

b)

I0
2

c)

I0
sin 2 2
8

d)

I0
sin 2 2
4

Ans: c
Exp: Malus Law I I 0 Cos 2
I
I
I
I0
, I 2 0 Cos 2 , I 3 0 Cos 2 Cos 2 (90 ) I 3 0 Sin 2 2
2
2
8
2
Q22. A double pendulum consists of two point masses m attached by massless strings of length l as
shown in the figure. The kinetic energy of the pendulum is
I1

a)

1 2 2 2
ml 1 2
2

c)

1 2 2
ml 1 222 212 cos(1 2 )
2

Ans: b

1 2 2 2
ml 21 2 212 cos(1 2 )
2

d)

1 2 2 2
ml 21 2 212 cos(1 2 )
2

1 2 2
1
ml 1 and T2 ml 2 12 22 212 Cos(1 2 )
2
2
1
Total Kinetic Energy T ml 2 212 22 212 Cos(1 2 )
2
Q23. In the operational amplifier circuit below, the voltage at point A is

Exp: T1

a) 1.0V
b) 0.5V
c) 0V
Ans: b
V 1 VA 0
1
Exp: A

0 2V A 1 V A
1
1
2

b)

d) -5.0V

UV PHYSICS ACADEMY
Shivam Road, New Nallakunta, Hyderabad, Ph: 04032458147, 09885072826
www.uvphysics.com
Q24. A point particle of mass m carrying an electric charge q is attached to a spring of stiffness constant
k. A constant electric field E along the direction of the spring is switched on forr a time interval
T (where T m / k ). Neglecting radiation loss, the amplitude of oscillation after the field is
switched off is :
a) qE / k
b) qET 2 / m
c) qE m / Tk 3 / 2
d) qET / mk
Ans: b
dP
qET
Exp: F
FT P mv
qET mv v
dt
m
1
1
mv 2 q 2 E 2T 2
For the amplitude of oscillations, Kx 2 mv 2 x 2

2
2
K
mK
2
qET
qET
x

( K m / T 2 )
m
mK
Q25. A constant force F is applied to a relativistic particle of rest mass m. If the particle starts from rest
at t = 0, its speed after a time t is
Fct
Ft
a) Ft/m
b) c tanh
c) c 1 e Ft / mc
d)

2
mc
F t 2 m 2c 2
Ans: d
m2c 2 2
1
m 2 c 2 m 2 c 2 F 2t 2
dP
mv
mc
Exp: F
F 2t 2

Ft P

dt
1 2
2
F 2t 2
F 2t 2
1 v2 / c2
1 2

Ft

F ct

F 2t 2 m 2 c 2
F 2t 2 m 2 c 2
Q26. The potential of a diatomic molecule as a function of the distance r between the atoms is given by
a
b
V (r ) 6 12 . The value of the potential at equilibrium separation between the atoms is:
r
r
2
a) 4a / b
b) 2a 2 / b
c) a 2 / 2b
d) a 2 / 4b
Ans: d
2b
a
b
a2
a
b V (r )
6a 12b
V (r )

Exp: V (r ) 6 12 ;
0 7 13 0 r 6
2b / a 2b / a 2
a
4b
r
r
r
r
r
Q27. Four equal point charges are kept fixed at the four vertices of a square. How many neutral points
(i.e., points where the electric field vanishes) will be found inside the square
a) 3
b) 4
c) 5
d) 7
Ans: c

UV PHYSICS ACADEMY
Shivam Road, New Nallakunta, Hyderabad, Ph: 04032458147, 09885072826
www.uvphysics.com

, where
r2
and R are positive constants. The charge contained within a sphere of radius R, centered at the origin
is
e2
R2
e
R
a) 0 2
b) 0 2
c) 4 0
d) 0
e
e
R
R
Ans: c
2


q
r / R 1
q

r 2 Sindd q 4 0 1 e 1 , at r= R
Exp: E. ds
q 0 E. d s
0 1 e
2
0
r
0 0
Q29. A counter consists of four flip flops connected as shown in the figure.
If the counter is initialized as A0 A1 A2 A3 0110 , the state after the next clock pulse is
a) 1000
b) 0001
c) 0011
d) 1100
Ans: c
Exp: Initially A0A1A2A3 = 0110
For 1st Flip Flop, J=0, k=1, then the output = 0
For 2nd Flip Flop J=A0=0, K= Q =1, then the output =0
Q28. A static charge distribution gives rise to an electric field of the form E 1 e r / R

For 3rd Flip Flop J= Q =1, K=A1=1, then the output =1


For 4th Flip Flop J=A2=1, K= Q =1, then the output =1
The state after the clock pulse is 0011
Q30. In a Youngs double slit interface experiment, the slits are at a distance 2L from each other and
the screen is at a distance D from the slits. If a glass slab of refractive index and thickness d is
placed in the path of one of the beams, the minimum value of d for the central fringe to be dark is

D
D

a)
b)
c)
d)
( 1)
( 1) L
2( 1)
( 1) D 2 L2
Ans: d
Exp: When a glass slab of refractive index and thickness t is introduced, the path difference become
1d . For the central fringe to be dark, path difference = n , n=1 d
2
2 1

UV PHYSICS ACADEMY
Shivam Road, New Nallakunta, Hyderabad, Ph: 04032458147, 09885072826
www.uvphysics.com
Q31. The pins 0, 1, 2 and 3 of part A of a microcontroller are connected with resistors to drive an LED
at various intensities as shown in the figure. For Vcc 4.2V and a voltage drop of 1.2V across the
LED, the range (maximum current) and resolution (step size) of the drive current are, respectively.

a) 4.0mA and 1.0mA


Ans: c
Exp: For maximum current

b) 15.0mA and 1.0mA


A3

A1 A2

c) 7.5mA and 0.5mA d) 4.0mA and 0.5mA

A0

0 0 0
0
4.2 1.2 4.2 1.2 4.2 1.2 4.2 1.2
I max

7.5mA
0K
1K
3K
6K
A A1 A2 A0
4.2 1.2
For step size 3
, I0
0.5mA
0 0 0
1
6K
Q32. An unbiased dice is thrown three times successively. The probability that the numbers of dots on
the uppermost surface add up to 16 is
a) 1/16
b) 1/36
c) 1/108
d) 1/216
Ans: b
Exp: Favourable outcomes : (4,6,6), (6,4,6), (6,6,4), (5,5,6), (5,6,5), (6,5,5)
6
1
The probability of getting sum 16 is = 3
36
6

Q33.

The generating function F ( x, t ) Pn ( x)t n for the Legendre polynomials Pn (x) is


n 0

2 1 / 2

. The value of P3 (1) is


F ( x, t ) (1 2 xt t )
a) 5/2
b) 3/2
c) +1
Ans: d

1
Exp: F ( x, t ) Pn ( x)t n
n 0
1 2 xt t 2

d) -1

Put x=1 , 1 t 1 t t 2 t 3 Pn (1)t n Comparing we get P3 (1) 1


1

Q34. Given that the ground state energy of the hydrogen atom is -13.6eV, the ground state energy of
positronium ( which is a bound state of an electron and positron) is
a) +6.8eV
b) -6.8eV
c) -13.6eV
d) -27.2eV
Ans: b
E
Exp: E ( positronium) E ( Hydrogen) H 6.8eV
m
2

UV PHYSICS ACADEMY
Shivam Road, New Nallakunta, Hyderabad, Ph: 04032458147, 09885072826
www.uvphysics.com
1 2
kr . Let
2
l1 and l2 be the angular momenta and r1 , r2 be the radii of the orbits respectively. If l1 / l 2 2 , the
value of r1 / r2 is

Q35.

Two particles of identical mass move in circular orbit under a central potential V (r )

a) 2
Ans: a

b) 1/ 2

c) 2

d) 1/2

L1 r12
r
2 1 2
L2 r2
r2
Q36. The equation of the plane that is tangent to the surface xyz = 8 at the point (1, 2, 4) is
a) x+2y+4z = 12
b) 4x+2y +z = 12
c) x+4y+2z = 0
d) x + y+ z = 7
Ans: b
4i 2 j k

Exp: xyz 8 , Normal to the surface n

21

The tangent to the plane at the point (1,2,4) is r (i 2 j 4k . n 0
Exp: Angular momentum L mvr mr 2

( x 1)i ( y 2) j ( z 4)k. 4i 221j k 0 4x 2y z 12


The energy of first excited quantum state of a particle in the two dimensional potential
1
V ( x, y ) m 2 ( x 2 4 y 2 ) is
2
3
5
a) 2
b) 3
c)
d)
2
2
Ans: d
Exp: Energy of the system, E n x 1 / 2 x n y 1 / 2 y
Q37.

E n x 1 / 2 n y 1 / 22

nx n y

nx , n y

0
1

(0,0)
(0,1)
(1,0)

3 / 2
7 / 2
5 / 2

2n y 3 / 2

The energy of the 1st excited state is 5 / 2

bS 3
Q38. The internal energy E of a system is given by E
, where b is a constant and other symbols
VN
have their usual meaning. The temperature of the system is equal to
bS 2
3bS 2
a)
b)
VN
VN
Ans: b
b
E

3S 2
Exp: T
S N ,V VN

bS 3
c) 2
V N

S
d)
N

UV PHYSICS ACADEMY
Shivam Road, New Nallakunta, Hyderabad, Ph: 04032458147, 09885072826
www.uvphysics.com
Consider a particle in a one dimensional potential that satisfies V(x) = V(-x). Let 0 and 1

Q39.

denote the ground and the first excited states, respectively, and let 0 0 1 1 be a
normalized state with 0 and 1 being real constants. The expectation value x of the position
operator x in the state is given by

a) 02 0 x 0 12 1 x 1 b) 01 0 c 1 1 x 0
Ans: b
Exp: x x 0 0 1 1 x 0 0 1 1

x 01 0 x 1 1 x 0

c) 02 12

d) 2 01

Q40. A 3 3 matrix M has Tr[M] = 6, Tr[M 2 ] 26 andTr[M 3 ] 90 . Which of the following can be
a possible set of eigenvalues of M.
a) {1, -1, 4}
b) {-1, 0, 7}
c) {-1, 3, 4}
d) {2, 2, 2}
Ans: c
Exp: From the option (c) , Tr(M)= -1+3+4= 6; Tr(M2)=1+9+16=26; Tr(M3)=-1+27+64=90
Q41. The perturbation H 1 bx 4 , where b is a constant, is added to the one dimensional harmonic
1
oscillator potential V ( x) m 2 x 2 . Which of the following denotes the correction to the ground
2
state energy to first order in b .
[Hint: The normalized ground state wave function of the one dimensional harmonic oscillator

m mx 2 / 2
potential is 0
. You may use the following integral
e

1

n
1
2 n ax2
2
x
e
dx

a
n

1/ 4

a)

3bh 2
4m 2 2

b)

3bh 2
2m 2 2

c)

3bh 2
2 m 2 2

d)

15bh 2
4m 2 2

Ans: a

b 2
3b 2
2
(1)
6
n

6
n

3
For
the
ground
state,
n=1

4m 2 2
4m 2 2
Q42. A battery powers two circuits C1 and C2 , as shown in the figure.
The total current I drawn from the battery is estimated by measuring the currents I1 and I 2 through
the individual circuits. If I1 and I 2 are both 200 mA and if the errors in their measurement are 3 mA
and 4mA respectively, the error in the estimate of I is
Exp: H bx 4 ; E (1) b n x 4 n

a) 7.0mA
b) 7.5mA
Ans: a
Exp: I1 200 3mA; I 2 200 4mA

c) 5.0mA

I1 I 2 (400 7)mA

d) 10.5mA

UV PHYSICS ACADEMY
Shivam Road, New Nallakunta, Hyderabad, Ph: 04032458147, 09885072826
www.uvphysics.com
Q43. Consider a Maxwellian distribution of the velocity of the molecules of an ideal gas. Let
Vmp and Vrms denote the most probable velocity and the root mean square velocity, respectively. The
magnitude of the ratio Vmp / Vrms is
a) 1
Ans: c

b) 2/3

c)

d) 3/2

2/3

v mp
2 KT
3KT
2
Root mean square velocity v rms

m
m
v rms
3
Q44. If the number density of a free electron gas in three dimensions is increased eight times, its Fermi
temperature will
a) increase by a factor of 4
b) decrease by a factor of 4
c) increase by a factor of 8
d) decrease by a factor of 8
Ans: a
2/3
2
Exp: Fermi Energy E f
3 2 n
2m
If n is increased by eight times, Fermi energy is increased by 8 times. Since E f n 2 / 3
Q45. A system of N noninteracting spin-1/2 particles is placed in an external magnetic field H. The
behavior of the entropy of the system as a function of energy is given by
Exp: Most probable velocity v mp

a)
Ans: b

b)

c)

d)

S
N U N U N U N U

ln

ln
, where H .
NK
2 N
2 2 N 2
Entropy is symmetric about E
PART-C
Q46. A particle of mass m moves inside a bowl. If the surface of the bowl is given by the equation
1
z ax 2 y 2 , where a is a constant, the Lagrangian of the particle is
2
1
1
2 2
2
2 2
a) m r 2 r 2 2 gar 2
b) m 1 a r r r
2
2
Exp:

c)

1
m r 2 r 2 2 r 2 sin 2 2 gar 2
2

Ans: d
Exp: Lagrangian L

z arr L

d)

1
m r 2 r 2 2 z 2 mgz
2

1
ar 2
m r 2 r 2 2 a 2 r 2 r 2 mg
2
2

1
m 1 a 2 r 2 r 2 r 2 2 gar 2
2
z

1
1
a x 2 y 2 ar 2 r 2 2 z / a
2
2
1
L m (1 a 2 r 2 )r 2 r 2 2 gar 2
2

UV PHYSICS ACADEMY
Shivam Road, New Nallakunta, Hyderabad, Ph: 04032458147, 09885072826
www.uvphysics.com
Q47. An electron of energy 27GeV collides with a proton of energy 820GeV. The heaviest particle
which can be produced in this collision has mass close to
a) 300GeV
b) 821GeV
c) 850GeV
d) 1127GeV
Ans: b
Exp: From the law of conservation of energy, the mass of the heaviest particle should be less than
847GeV/c2.
Q48. Let x1 (t ) and x2 (t ) be two linearly independent solutions of the differential equation

d 2x
dx
dx (t )
dx (t )
2 f (t ) x 0 , and let w(t ) x1 (t ) 2 x2 (t ) 1 . If w(0) = 1, then w(1) is given by
2
dt
dt
dt
dt
2
a) 1
b) e
c) 1/e
d) 1/ e 2
Ans: d
Exp: Given x1 and x2 are the solutions of the differential equation.
d 2 x1 2dx1
d 2 x2 2dx2
--(1)
and

f
(
t
)
x

f (t ) x2 0 ---(2)
1
dt
dt
dt 2
dt 2
Multiply (2) by x1 and (1) by x2, then subtracting, we get
d
x1 x2 x2 x1 2( x1 x 2 x2 x1 ) 0 x1 x 2 x2 x1 2( x1 x2 x2 x1 ) 0
dt
dW
2W 0 , since W (t ) x1 (t ) x 2 x2 (t ) x1
dt
W (t ) W (0)e 2t , at t=0, W(0) = 1 ; W (t ) e 2t
Q49. Assume that the free energy of a magnetic system has an expansion in the order parameter M of
the form F (M , T ) a(T TC )M 2 bM 4 cM 6 , with a, b and c > 0. As the temperature is lowered
below TC , the system undergoes a phase transition. The behavior of the order parameter just below
the transition, where (T TC ) is very small, is best described by
a) M (TC T ) 1 / 2
Ans: b

b) M (TC T )1 / 2

Exp: F (M , T ) a(T TC )M 2 bM 4 cM 6

c) M (TC T )

d) M (TC T )3

F
2a(T TC ) M 4bM 3 6cM 5
M

2F
2a(T TC ) 12bM 2 30cM 4 0 (T TC ) M 2
M 2
Q50. A planet of mass m moves in the inverse square central force field of the Sun of mass M. If the
semi major and semi minor axes of the orbit are a and b, respectively, the total energy of the
planet is :
GMm
( a b)
1 1
1 1
a)
b) GMm
c) GMm
d) GMm
ab
( a b) 2
a b
a b
Ans: a
mv 2 GMm
GM
GMm

v2
Exp: For circular orbit,
Total Energy E= KE+ PE=
2
r
r
2r
r
GMm
Option (a) will give
if a=b=r
2r

UV PHYSICS ACADEMY
Shivam Road, New Nallakunta, Hyderabad, Ph: 04032458147, 09885072826
www.uvphysics.com
Let 0 and 1 denote the normalized eigenstates corresponding to the ground and first excited

Q51.

1
0 1 , is
2
d) p 2m

state of a one dimensional harmonic oscillator. The uncertainty p in the state


a) p m / 2
Ans: c
Exp: P

Px2 Px

b) p m / 2
2

; Px 0

Px2

c) p m

1 m
m

(2n1 1)
(2n2 1) ; n1 0, n2 1

2 2
2

1 4m

m Px m
2 2
Q52. A laser operating at 500nm is used to excite a molecule. If the stokes line is observed at
770 cm 1 , the approximate positions of the stokes and the anti Stokes and the anti Stokes lines
are
a) 481.5nm and 520nm
b) 481.5nm and 500nm
c) 500nm and 520nm
d) 500nm and 600nm
Ans: a
Exp: Exciting line is at 500nm. Exciting line is in between stoke and antistoke lines.
1 for 2n x 2n 1
Q53. The graph of the function f ( x)
is (where n = 0, 1, 2, 3,) is shown
0 for 2n 1 x 2n 2
~
in figure. Its Laplace transform f ( s) is
Px2

a)

1 e s
s

b)

1 e s
s

c)

1
s 1 e s

d)

1
s 1 es

Ans: c

Exp: L f ( x) e

Sx

f ( x)dx e

0
1

sx

.1dx e

0
3

sx

.0dx e sx .1dx ......


2

e sx
e sx
1
s
2 s
3 s

........ 1 e e e

s
s

1 1

s 1 e s

UV PHYSICS ACADEMY
Shivam Road, New Nallakunta, Hyderabad, Ph: 04032458147, 09885072826
www.uvphysics.com
Q54. The energy levels of electrons of mass m and charge e confined in an area A in the xy plane
with a uniform magnetic field B applied in the z direction are given by
eBa
1 eB

. The lowest level is


En n
, n 0,1,2,3,.... . The degeneracy of each level is
c
2 mc

completely filed and the others are empty. The Fermi energy

2 N
, where N is the total number of
2 mA

electrons, is
a) coincident with the n = 0 level
b) coincident with the n = 1 level
c) midway between the n = 0 the n = 1 levels d) midway between the n = 1 and the n = 2 levels
Ans: a
eB
eBA
Exp: En n 1 / 2
, n 0,1,2.... and each level is split into
mc
c
2
eB cN
N
E n 1 / 2

n 1 / 2
mc eBA
mA
Q55. An annulus of mass M made of a material of uniform density has inner and outer radii a and b
respectively. Its principal moment of inertia along the axis of symmetry perpendicular to the plane of
the annulus is :
1 (b 4 a 4 )
1
1
1
a) M 2
b) M (b 2 a 2 )
c) M (b 2 a 2 )
d) M (b 2 a 2 )
2
2 (b a )
2
2
2
Ans: d
1
Exp: The moment of inertia of annular ring I M (a 2 b 2 ) , where a is the inner radius and b is the
2
outer radius.
3
dx
Q56. The value of the real integral I 2
is
x

3x

2
3
a)

3
2

2
b) ln
5

c)

d) 0

Ans: b
Q57. The minimum energy of an electron ( the rest mass of which is 0.5MeV) that can emit Cerenkov
radiation while passing through water ( of refractive index 1.5) is approximately
a) 1.0MeV
b) 3.0MeV
c) 0.6MeV
d) 0.5MeV
Ans: c
m0 c 2
0.5c 2

0.6MeV
Exp: v / c 1 / ; E mc 2
1 v2 / c2
1 2

UV PHYSICS ACADEMY
Shivam Road, New Nallakunta, Hyderabad, Ph: 04032458147, 09885072826
www.uvphysics.com
Q58. A heater and a thermocouple are used to measure and control the temperature T of a sample at
T0 250 0 C . A feedback circuit supplies power P to the heater according to the equation
dT
with appropriately tuned values of the coefficients G and D. In order to
P P0 G(T0 T ) D
dt
maintain temperature stability in the presence of an external heat perturbation which causes small
but rapid fluctuations of temperature, it is necessary to
a) decrease D
b) increase D
c) decrease G
d) increase G
Ans: a
Q59. The trajectory on the zp z - plane (phase space trajectory) of a ball bouncing perfectly
elastically off a hard surface at z = 0 is given by approximately by (neglect friction):

a)
b)
c)
d)
Ans: a
Q60. According to the shell model the spin and parity of the two nuclei 125
51 Sb and
respectively,

5
5
a) and
2
2
Ans: d

5
7
b) and
2
2

7
5
c) and
2
2

89
38

Sr are,

7
7
d) and
2
2

7
Exp: From shell model the last nucleon(proton) of 51Sb is in the state 1g 7 / 2 , J
2

7
From shell model the last nucleon(neutron) of 38Sr89 is in the state 1g 7 / 2 , J
2
1
u1 u 2 , where
Q61. The wave function of the particle at time t = 0 is given by (0)
2
u1 and u 2 are the normalized eigenstates with eigenvalues E1 and E2 respectively, ( E2 E1 ) .

125

The shortest time after which (t ) will become orthogonal to (0) is


a)


2( E 2 E1 )

b)

( E 2 E1 )

c)

Ans: b
Exp: (0)

u2

and

(t )

2
2
Orthogonal condition is (0) (t ) 0

2
2 E 2 E1

e iE1t / u 2 e iE2t /

e iE1t / e iE2t / e i ( E2 E1 )t / 1 ( E2 E1 )t /

d)

2
( E2 E1 )

UV PHYSICS ACADEMY
Shivam Road, New Nallakunta, Hyderabad, Ph: 04032458147, 09885072826
www.uvphysics.com
Q62. A gas of N non interacting particles is in thermal equilibrium at temperature T. Each particle
can be in any of the possible non degenerate states of energy 0, 2 and 4 . The average energy
per particle of the gas, when 1 , is
a) 2
b) 3
c) 2 /3
d)
Ans: a
Exp: Partition function Z 1 e 2 e 4 1 1 1 3 , since 1

E E s p s 2
s

Q63.
a)

e 2
e 4
4
2
Z
Z

The first few terms in the Taylor series expansion of the function f(x) = sin x around x
2
3

1
1

1 x x x ...
4 2!
4
3!
4
2

c) x x
4 3!
4

Ans: b

are

2
3
b) 1 1 x 1 x 1 x ...

d)

2!

3!

1
x 2 x3
1

.....

2! 3!
2

( x x0 )
f ( x0 ) .....
2!
f ( x0 ) sin / 4 1 / 2 ; f ( x0 ) cos / 4 1 / 2 ; f ( x0 ) sin / 4 1 / 2

Exp: Taylor expansion f ( x) f ( x0 ) ( x x0 ) f ( x0 )

1
1
1

1 ( x / 4) ( x / 4) 2 ( x / 4) 3 .......

2!
3!
2

Q64. A one-dimensional chain consists of a set of N rods each of length a . When stretched by a load,
each rod can align either parallel or perpendicular to the length of the chain. The energy of a rod is
when 'aligned parallel to the length of the chain and is + when perpendicular to it. When the
chain is in thermal equilibrium at temperature T, its average length is
a) Na/2
b) Na
c) Na 1 e 2 / k BT
d) Na 1 e 2 / k BT
Ans: c
0 e
Na

Exp: Average value of Length L LPs Na



2
s
e e 1 e
f ( x)

UV PHYSICS ACADEMY
Shivam Road, New Nallakunta, Hyderabad, Ph: 04032458147, 09885072826
www.uvphysics.com
Q65.

If the hyperfine interaction in an atom is given by H aS e .S p where Se and S p denote the

electron and proton spins, respectively, the splitting between the 3 S1 and 1 S 0 state is
a) a 2 / 2
b) a 2
c) a 2 / 2
d) 2a 2
Ans: b


a 2
a 2
2
2
Exp: H aS e .S p , H a S e .S p S S e S p
S (S 1) 3 / 2
2
2
a 2
3a 2
For 3S1 , E
and For 1S0 , E
4
4
2
a
So the splitting3 between the states is
(1 3) a 2
4
Q66. Consider a solenoid of radius R with n turns per unit length, in which a time dependent current
I I 0 ( where R / c 1 ) flows. The magnitude of the electric field at a perpendicular distance r<
R from the axis of symmetry of the solenoid, is
a) 0
b) 1 0 nI 0 R 2 cos t c) 1 0 nI 0 r sin t
d) 1 0 nI 0 r cos t
2r

Ans: d
Exp For Solenoid , B 0 nI 0 Sint and

2r


d
E
.
d
l

B.ds

dt

2r

E (2 r )

d
dB
( Br 2 ) r 2
dt
dt

r
Magnitude of Electric field, E 0 nI 0 Cost
2
49
Q67. The difference in the Coulomb energy between the mirror nuclei 49
24 Cr and 25 Mn is 6.0MeV .
Assuming that the nuclei have a spherically symmetric charge distribution and that e2 is
approximately 1.0MeV-fm, the radius of the 49
25 Mn nucleus is
a) 4.9 10 13 m
Ans: b

b) 4.9 10 15 m

c) 5.1 10 13 m

d) 5.11015 m

3 e2
3 e2
( Z12 Z 22 ) R0
( Z12 Z 22 ) , R0 4.9 10 15 m
5 R0
5 Ec
Q68. The ratio of intensities of the D1 and D2 lines of sodium at high temperature is
a) 1:1
b) 2:3
c) 1:3
d) 1:2
Ans: d
I D1 2 1

Exp: Intensity is proportional to (2J+1)
I D2 4 2
Exp: For Mirror nuclei, Ec

UV PHYSICS ACADEMY
Shivam Road, New Nallakunta, Hyderabad, Ph: 04032458147, 09885072826
www.uvphysics.com
Q69. A constant perturbation as shown in the figure below acts on a particle of mass m confined in an
infinite potential well between 0 and L. The first order correction to the ground state energy of the
particle is
a) V0 / 2
b) 3V0 / 4
c) V0 / 4
d) 3V0 / 2
Ans: b
L/2
L
2
x
2 V0
2
x
2 x
Sin

V0 Sin 2
Exp:
Sin ; E (1) H

L 0 2
L L L/2
L
L
L

2
L

L/2

V0
x V0 2 1
2x L
Sin 2

x sin

2
L
2 L 2
L 2 0

L/2

V0
4

L/2
V
3V
2
x
2 1
2x L
V0 Sin 2
V0
x sin
0 E (1) 0

L 0
L
L 2
L 2 L / 2
2
4
Q70. An atom of mass M can be excited to a state of mass M by photon capture. The frequency
of a photon which can cause this transition is
a) c 2 / 2h
b) c 2 / h
c) 2 c 2 / 2Mh
d) ( 2M )c 2 / 2Mh
Ans: d
Exp: Applying Law of conservation energy
L

h mc 2

p 2 c 2 (m ) 2 c 4 h mc 2 p 2 c 2 (m ) 2 c 4
2

( 2m)c 4
( 2m)c 2

2hm
2mc 2
Q71. The excitations of a three dimensional solid are bosonic in nature with their frequency and
wave number k are related by k 2 in the large wavelength limit. If the chemical potential is
zero, the behavior of the specific heat of the system at low temperature is proportional to
a) T 1 / 2
b) T
c) T 3 / 2
d) T 3
Ans: c
Exp: For Dispersion relation k s , the specific heat CV T 3 / s
h 2 2 m 2 c 4 2hmc 2 p 2 c 2 m 2 c 4 2 c 4 2mc 4 h

UV PHYSICS ACADEMY
Shivam Road, New Nallakunta, Hyderabad, Ph: 04032458147, 09885072826
www.uvphysics.com
Q72. Gas molecules of mass m are confined in a cylinder of radius R and height L (with R>>L) kept
vertically in the Earths gravitational field. The average energy of the gas at low temperature (such
that mgL k BT ) is given by
a) Nk BT / 2
b) 3Nk BT / 2
c) 2 Nk BT
d) 5Nk BT / 2
Ans: d
1
Exp: According to equitation theorem, each quadratic term contribute K B T energy
2
The average Kinetic energy =5/2 KBT
mgz

1
KT
mg
mgz
e

(mg / KT ) 2
Average value of Potential Energy V 0

KT
mgz
1
e KT
mg / KT
0

1
a2
0
Q73. The figure below shows a voltage regulator utilizing a Zener diode of breakdown voltage 5V and
a positive triangular wave input of amplitude 10V.
For Vi 5V , the Zener regulates the output voltage by channeling the excess current i passing
through the Zener diode.
Using

xe

ax

dx

a)
b)
c)
d)
Ans: b
Exp: When zener is OFF, Zener current is zero. And when zener is ON, zener current will flow.

UV PHYSICS ACADEMY
Shivam Road, New Nallakunta, Hyderabad, Ph: 04032458147, 09885072826
www.uvphysics.com
Q74. A constant electric current I in an infinitely long straight wire is suddenly switched on at t = 0.
The vector potential at a perpendicular distance r from the wire is given by
k I 1

A 0 ln ct c 2 t 2 r 2 . The electric field at a distance r(<<ct) is


2
r

I 1
c 0 I
c0 I
1
a) 0
b) 0
c)
d)
ij
k
ij
2 2
2
2
2
2
2 t 2
2
2 c t r
2 c t r

Ans: d

I 1
A
A

Exp: E
E
E k 0
ln ct c 2 t 2 r 2
t
t
2 t r

I
c
I
r
1
c 2 2t
E k 0
E k 0
c

2
2
2
2
2
2
2
2 c t 2 r 2
2 ct c t r r
2 c t r

Q75. Monochromatic light of wavelength 660nm and intensity 100mW / cm 2 falls on a solar cell of
area 30cm 2 . The conversion efficiency of the solar cell is 10%. If each converted photon results in an
electron hole pair, what is the maximum circuit current supplied by the solar cell?
(Take h 6.6 10 34 J s, c 3 108 m / s and e 1.6 10 19 C ).
a) 160mA
b) 320mA
c) 1600mA
d) 3200mA
Ans: a

_____________Download from www.JbigDeaL.com Powered By JbigDeaL____________

_____________Download from www.JbigDeaL.com Powered By JbigDeaL____________

_____________Download from www.JbigDeaL.com Powered By JbigDeaL____________

_____________Download from www.JbigDeaL.com Powered By JbigDeaL____________

_____________Download from www.JbigDeaL.com Powered By JbigDeaL____________

_____________Download from www.JbigDeaL.com Powered By JbigDeaL____________

_____________Download from www.JbigDeaL.com Powered By JbigDeaL____________

_____________Download from www.JbigDeaL.com Powered By JbigDeaL____________

_____________Download from www.JbigDeaL.com Powered By JbigDeaL____________

_____________Download from www.JbigDeaL.com Powered By JbigDeaL____________

_____________Download from www.JbigDeaL.com Powered By JbigDeaL____________

_____________Download from www.JbigDeaL.com Powered By JbigDeaL____________

_____________Download from www.JbigDeaL.com Powered By JbigDeaL____________

_____________Download from www.JbigDeaL.com Powered By JbigDeaL____________

_____________Download from www.JbigDeaL.com Powered By JbigDeaL____________

UV PHYSICS ACADEMY
Shivam Road, New Nallakunta, Hyderabad, Ph: 04032458147, 09885072826
www.uvphysics.com
CSIR JUNE-2011
Q1. The cumulative profits of a company since its inception are shown in the diagram. If the net worth of the
company at the end of 4lh year is 99 crores, the principal it had started with was

a) 9.9 crores
b) 91 crores
c) 90 crores
d) 9.0 crores
Ans: c
Q2. Popular use of which of the following fertilizers increases the acidity of soil? '
a) Potassium Nitrate
b) Ammonium sulphate
c) Urea
d) Superphosphate of lime
Ans: c
Q3. Exposing an organism to a certain chemical can change nucleotide bases in a gene, causing mutation. In
one such mutated organism if a protein had only 70% of the primary amino acid sequence, which of the
following is likely.
a) Mutation broke the protein
b) The organism could not make amino acids
c) Mutation created a terminator codon
d) The gene was not transcribed
Ans: c
Q4. A reference material is required to be prepared with 4 ppm calcium. The amount of CaCO3 (molecular
weight = 100) required to prepare 1000 g of such a reference material is
a) 10g
b) 4g
c) 4mg
d) 10mg
Ans: d
Q5. Identify the figure which depicts a first order reaction.

a)
b)
c)
d)
Ans: b
Q6. The speed of a car increases every minute as shown in the following Table. The speed at the end of the
19th minute would be
Time (minutes) 1
2
3
4 24
25
Speed (m/sec) 1.5
3.0 4.5
6.0 ..36.0 37.5
a) 26.5
b) 28.0
c) 27.0
d) 28.5
Ans: d

UV PHYSICS ACADEMY
Shivam Road, New Nallakunta, Hyderabad, Ph: 04032458147, 09885072826
www.uvphysics.com
Q7. If the atmospheric concentration of carbon dioxide is doubled and there are favorable conditions of
water, nutrients, light and temperature, what would happen to water requirement of plants
a) It decreases initially for a short time and then returns to the original value
b) It increases
c) It decreases
d) It increases initially for a short time and then returns to the original value
Ans: d
Q8. If Vinput is applied to the circuit shown, the output would be

a)
b)
c)
d)
Ans: c
Q9. A physiological disorder X always leads to the disorder Y, However, disorder Y may occur by itself. A
population shows 4% incidence of disorder Y. Which of the following inferences is valid
a) 4% of the population suffers from both X & Y
b) Less than 4% of the population suffers from X
c) At least 4% of the population suffers from X
d) There is no incidence of X in the given population
Ans: b
Q10. Water is dripping out of a tiny hole at the bottom of three flasks whose base diameter is the same, and
are initially filled to the same height, as shown. Which is the correct comparison of the rate of fall of the
volume of water in the three flasks

a) A fastest, B slowest b) B fastest, A slowest


c) B fastest, C slowest d) C fastest, B slowest
Ans: c
Q11. How many bonds are present in the following molecule HC CCH CHCH 3
a) 4
b) 10
c) 6
d) 13
Ans: c

UV PHYSICS ACADEMY
Shivam Road, New Nallakunta, Hyderabad, Ph: 04032458147, 09885072826
www.uvphysics.com
Q12. The graph represents the depth profile of temperature in the open ocean; in which region this is likely to
be prevalent

a) Tropical region
b) Equatorial region c) Polar region
d) Sub-tropical region
Ans: c
Q13. The normal boiling point of a solvent (whose vapour pressure curve is shown in the figure) on a planet
whose normal atmospheric pressure is 3 bar, is about
a) 400k
b) 273k
c) 100k
d) 500k
Ans: c
Q14. Diabetic patients are advised a low glycaemic index diet. The reason for this is
a) They require less carbohydrate than healthy individuals
b) They cannot assimilate ordinary carbohydrates
c) They need to have slow, but sustained release of glucose in their blood stream
d) They can tolerate lower, but not higher than normal blood sugar levels
Ans: c
Q15. Glucose molecules diffuse across a cell of diameter d in time . If the cell diameter is tripled, the
diffusion time would
a) increase to 9 b) decrease structure c) increase of cage structures d) formation of tabular structures
Ans: d
Q16. The reason for the hardness of diamond is
a) extended covalent bonding
b) layered structure
c) formation of cage structures
d) formation of tubular structures
Ans: a
Q17. Which of the following particles has the largest range in a given medium, if their
initial energies are the same
a) alpha
b) gamma
c) positron
d) electron
Ans: d
Q18. A ball is dropped from a height h above the surface of the earth. Ignoring air drag, the curve that best
represents its variation of acceleration is

a)
Ans: d

b)

c)

d)

UV PHYSICS ACADEMY
Shivam Road, New Nallakunta, Hyderabad, Ph: 04032458147, 09885072826
www.uvphysics.com
Q19. Standing on a polished stone floor one feels colder than on a rough floor of the same stone. This is
because
a) Thermal conductivity of the stone depends on the surface smoothness
b) Specific heat of the stone changes by polishing it
c) The temperature of the polished floor is lower than that of the rough floor
d) There is greater heat loss from the soles of the feet when in contact with the polished floor than with
the rough floor
Ans: d
Q20. The acidity of normal rain water is due to
a) SO2
b) CO2
c) NO2
d) NO
Ans: a
PART-B (COMPULSORY)
EACH QUESTION CARRIES 3.5 MARKS
EACH WRONG ANSWER HAS A PENALTY OF 25% OF MARKS
b
, where a and b are positive constants. The
x2
angular frequency of small oscillations about the minimum of the potential is
a) 8b
b) 8a
c) 8a / b
d) 8b / a
Ans: b
b
V ( x)
2b
2b
Exp: V ( x) ax 2 2 for minimum potential
0 2ax 3 0;2ax 3 x 4 b / a ;
x
x
x
x
2
2
V ( x)
6b
a
V ( x)
2a 4 0 2a 6b 0 8a ;
8a i.e., 2 8a 8a
2
2
b
x
x
x

Q21. A particle of unit mass moves in a potential V ( x) ax 2

The angular frequency of small oscillations about the minimum of the potential 8a
Q22. A signal of frequency 10 kHz is being digitized by an A/D converter. A possible sampling time which
can be used is
a) 100 s
b) 40 s
c) 60 s
d) 200 s
Ans: d
1
1
Exp: Signal frequency =10KHz; time period = 1/f
4 Hz 100 10 6 s 100s
3
10 10 10
Q23. The electrostatic potential V(x, y) in free space in a region where the charge density is zero is given
by V ( x, y) 4e 2 x f ( x) 3 y 2 . Given that the x component of the electric field E x , and V are zero at
the origin f(x) is
a) 3x 2 4e 2 x 8x
b) 3x 2 4e 2 x 16 x c) 4e 2 x 8
d) 3x 2 4e 2 x
Ans: d
Exp: V 4e 2 x f ( x) 3 y 2 ; 2V 0 16e 2 x f 11( x) 6 0 since Ex = 0 at the origin E V ;
Ex 8e 2 x f 1 ( x) ; Ex (0,0) 8 f 1 (0) 0 f 1 (0) 8 ; V(0, 0)= 0 ; 4+f(0)= 0 ; f(0)=4; solving

16e 2 x f 11( x) 6 0 f 11( x) 6 16e 2 x f ( x) 6 16e


11

2x

f 1 ( x) 6 x 8e 2 x c. since

f1(0)=8 c1 16; f ( x) 3x 2 4e 2 x 16 x c2 ; sin ce f (0) 4 c2 0; f ( x) 3x 2 4e 2 x 16 x

UV PHYSICS ACADEMY
Shivam Road, New Nallakunta, Hyderabad, Ph: 04032458147, 09885072826
www.uvphysics.com
0

Q24. Consider the transition of liquid water to steam as water boils at a temperature of 100 C under a
pressure of 1 atmosphere. Which one of the following quantities does not change discontinuously at the
transition.
a) The Gibbs free energy b) The internal energy c) the entropy
d) The specific volume
Ans: a
Exp: Gibbs free energy does not change discontinuously.
Q25. The value of the internal dz z 2 e z , where C is an open counter in the
C

complex z plane as shown in the figure is


a) (5/e) + e
Ans: c

c) (5/e) e d) (5/e) e

b) e - (5/e)
1

Exp: By Cauchy Integral Theorem z 2 e z dz z 2 e z dz 0


1

z 2 e z dz z 2 e z dz z 2 e z 2 ze z 2e z
c

1
1

5
z 2 e z dz e
e
c

Q26. Which of the following matrices is an element of the group SU (2).

(1 i) / 3 1 / 3
1/ 2
1 1
3 / 2
c) 2 i i

b)
a)
d)
3 1 i

3 / 2 1/ 2
0
1
1
/
3
(
1

i
)
/
3

Ans: b

1 i
1
1 i
Exp: SU (2)
UU 1, This condition is satisfied
; , c, | |2 | |2 ;
,
,
3
3
3

1 i

by 3
1
3

1
3

1 i
3



Q27. For a constant uniform electric and magnetic fields E E0 and B B0 , it is possible to choose a gauge

such that the scalar potential and vector potential A are given by
1
1

a) 0 and A ( B0 r )
b) E0 .r and A ( B0 r )
2
2

c) E0 .r and A 0
d) 0 and A E0t
Ans: b

Exp: If E is uniform E. r E.r sin ce E E. r E E.r .....(1) for static fields





A
0 E ( E.r ) we have ( A B) A(.B) ( B.) A B(. A) ( A.) B ;
t







( A r ) B(.r ) (r .) B r (.B) ( B.)r ( B r ) B(.r ) ( B.)r ( (r .) 0 and .B 0)




( B r ) 3B B 2B 2 A B r 2 A A 1/ 2 B r

UV PHYSICS ACADEMY
Shivam Road, New Nallakunta, Hyderabad, Ph: 04032458147, 09885072826
www.uvphysics.com

Q28. Let a and b be two distinct three dimensional vectors. Then the component of b that is

perpendicular to a is given by




(b .a )b
(b .a )a
a (b a )
b (a b )
a)
b)
c)
d)
b2
a2
a2
b2
Ans: a


a b ab sin n; a (a b ) ab sin (a n ) a 2b sin k
a
(b a )

b
sin

k
;

Exp:
;
a (a b )
a2
b sin k;
2
a
Q29. The wave function of a particle is given by 1 0 i1 , where 0 and 1 are the normalized
2

eigen functions with energies E0 and E1 corresponding to the ground state and first excited state,
respectively. The expectation value of the Hamiltonian in the state is
E
E
E 2 E1
E 2 E1
a) 0 E1
b) 0 E1
c) 0
d) 0
2
2
3
3
Ans: d
1/ 2
1
Exp: 1 0 i1 Normalisation gives (1/2)+1=3/2; p1
1 / 3; p2
2/3
3/ 2
3/ 2
2

1
2 E 2 E1
H Average value of H Es p s ; E1 E0 ; E2 E1 ; H E0 E1 0
s
3
3
3
Q30. A particle is confined to the region x 0 by a potential which increases linearly as u( x) u0 x . The
mean position of the particle at temperature T is
k BT
k T
a) B
b) (k BT ) 2 / uo
c)
d) u0 k BT
u0
u0
Ans: a

Exp: Given u(x)=u0(x); The mean position x

xe

dx

E
e dx
0

u0 x
kT

e
0

xe

u0 x
kT

dx

u0 x
kT

dx

; e ax dx 1 / a
0

2
2
n! u x
x ax
1
kT

1!
kT ; x (kT ) u0 k BT
kT
xe
dx

;
dx

;
xe
dx

2
(a) n1 0
kT
u0
u02
0
u0 u0
u 02
u0


kT


kT

Q31. Circularly polarized light with intensity I 0 is incident normally on a glass prism as shown in the figure.
The index of refraction of glass is 1.5. The intensity I of light emerging from the
prism is
a) I 0
b) 0.96 I 0
c) 0.92 I 0
d) 0.88 I 0
Ans: a
Exp: Since the light is totally reflected at face BC, further at face AB, light incident normally and passes
without refraction. Hence there is no change in intensity I = I0.

UV PHYSICS ACADEMY
Shivam Road, New Nallakunta, Hyderabad, Ph: 04032458147, 09885072826
www.uvphysics.com
Q32. The acceleration due to gravity (g) on the surface of Earth is approximately 2.6 times that on the surface
of Mars is about one half the radius of Earth, the ratio of escape velocity on Earth to that on Mars is
approximately.
a) 1.1
b) 1.3
c) 2.3
d) 5.2
Ans: c
Exp: Given gs=2.6gM; RM=(1/2)RE Escape velocity ve 2 gR ;VE 2 g E RE 2 2.6 g M RE ;
V
1
VM 2 g M RM 2 g M RE ; E 5.2 2.3
2
VM
Q33. A plane of electromagnetic wave is propagating in a lossless dielectric. The electric field is given by

E ( x, y, z, t ) E0 ( x Az) exp ik 0 ct ( x 3z , where c is the speed of light in vacuum, E 0 , A and k 0

are constants and x and z are unit vectors along the x axis and z axis. The relative dielectric
constant of the medium, r and the constant A are
1
1
a) r 4 and A
b) r 4 and A
c) r 4 and A 3
d) r 4 and A 3
3
3
Ans: a

Exp: Given E ( x, y, z, t ) E0 ( x Az) exp ik 0 ct ( x 3z we know

E E0 e i ( k .r t ) E E0 ( x Az)e i ( k0 ( x 3z )k0 (t ) comparing we get k k 0 x 3k 0 z and k 0 c ;


k0
kc
0 c / 2; v c / 2 refractive index c / v c / c / 2 2 ;
velocity of EM waves, v / | k |
k 0 1 3 2k 0


( E ) ( E )
Relative dielectric constant r n 4; Again .E 0 i.e.,

0;
x
y
2

E0 e ik0 ct ( x 3x ik 0 AE0 eik0 ct ( x 3z 3ik 0 0 1 A 3 0; A 3 1; A 1/ 3; 0 4 and A 1/ 3


Q34. Consider the digital circuit shown below in which the input C is always high (I). The truth table for the
circuit can be written as
A
B
Z
0
0
0
1
1
0
1
1
The entries in the Z column (vertically) are
a) 1`010
b) 0100
c) 1111
Ans: d
Exp:
A
B
C
D
A.B
0
0
1
0
0
1
1
0
1
0
1
0
1
1
1
1
=1011

d) 1011

E
B C
1
0
1
0

Z
D+E
1
0
1
1

UV PHYSICS ACADEMY
Shivam Road, New Nallakunta, Hyderabad, Ph: 04032458147, 09885072826
www.uvphysics.com
Q35. The energy levels of the non relativistic electron in a hydrogen atom (i.e. in a Coulomb potential
V (r ) 1 / r ) are given by Enlm 1 / n 2 , where n is the principal quantum number, and the

corresponding wave functions are given by nlm , where l is the orbital angular momentum quantum
number. The spin of the electron is not considered. Which of the following is a correct statement.
a) There are exactly (2l +1) different wave functions nlm , for Enlm .
b) There are l (l +1) different wave functions nlm , for each Enlm .
c) E nlm does not depend on l and m for the Coloumb potential
d) There is a unique wave function nlm for each Enlm .
Ans: c
Exp: Enlm does not depend on l and m for the coulomb potential.


Q36. The Hamiltonian of an electron in a constant magnetic field BH .B where is a positive constant

and ( 1 , 2 , 3 ) denotes the Pauli matrices. Let B / and I be the 2 2 unit matrix. Then the
iHt /

operator e
simplifies to


t i .B t
i .B
a) I cos
b) I cos t
sin
sin t
2
B
2
B

i .B
i .B
c) I sin t
d) I sin 2 t
cos 2 t
cos t
B
B
Ans: b


iHt
iHt
.Bt
.Bt

Ht
Ht

i sin

Exp: e cos i sin ; Given H .B e cos

; we know



2

2
. A .B A.B i A B sin ce .B B ; i.e., .B B ;

iHt
iHt
B
.B
Bt
Bt

e cos
e I cos t i
sin t
i sin
given

B


Q37. The Hamiltonian of a system with n degrees of freedom is given by
H (q1 ,....qn ; p1 ,.... pn ; t ) with an explicit dependence on the time t. Which of the following is correct
a) Different phase trajectories cannot intersect each other.
b) H always represents the total energy of the system and is a constant of the motion.
c) The equations qi H / pi , p i H / qi are not valid since H has explicit time dependence
d) Any initial volume element in phase space remains unchanged in magnitude under time evolution.
Ans: a
Exp: Different phase trajectories cannot intersect each other.

UV PHYSICS ACADEMY
Shivam Road, New Nallakunta, Hyderabad, Ph: 04032458147, 09885072826
www.uvphysics.com
Q38. If the perturbation H 1 ax , where a is a constant, is added to the infinite square well potential
0 for 0 x
. The correction to the ground state energy, to first order in a, is
V ( x)
for otherwise
a
a
a
a)
b) a
c)
d)
2
4
2
Ans: a
0 for 0 x
1
Exp: H ax; V ( x)
first order correction to the ground state energy
for otherwise

E1 0 H 1 0 ; 0

sin

x
2
2a
2a
2

sin x
ax
sin
xdx

x sin 2 xdx

0
0

a x2
a
2a 1
a
x(1 cos 2 x)dx ( x x cos 2 x)dx

2 0
2
20
0

Q39. Let p n (x) (where n = 0, 1, 2, 3,..) be a polynomial of degree n with real coefficients, defined in the
4

interval 2 n 4. If pn ( x) pm ( x)dx nm , then


2

a) p0 ( x) 1 and p1 ( x) 3 (3 x)
2

c) p0 ( x)

1
3
and p1 ( x)
(3 x)
2
2

b) p0 ( x)

d) p0 ( x)

1
3
and p1 ( x)
(3 x)
2
2

and p1 ( x) 3 (3 x)

Ans: d
4

Exp: pn ( x) pm ( x)dx nm pn ( x) 2 dx 1.n m ; pn ( x) pm ( x)dx 0.n m ;


From the option (d)
4
4
1 2 1 4 1
2
|
p
(
x
)
|
dx

0
dx 2 x2 2 2 1; n=m=0
2
2 2
4

34
3
x2
1 3
p
(
x
)
p
(
x
)
dx

(
3

x
)
dx
;
n

0
m

1
;

(
3

x
)
dx

3
x

2 2
2
2 2
2
2 2 2
4

4
4
3
(12 8) (6 2) 3 12 8 6 2 0 only option (d) satisfies P0 ( x) 2 dx 1; p0 ( x) p1 ( x)dx 0
2
2
2
2
Q40. A cavity contains blackbody radiation in equilibrium at temperature T . The specific heat per unit
volume of the photon gas in the cavity is of the form CV T 3 , where is a constant. The cavity is
expanded to twice its original volume and then allowed to equilibrate at the same temperature T . The
new internal energy per unit volume is
a) 4T 4
b) 2T 4
c) T 4
d) T 4 4
Ans: d
Q U
vT 4

U Cv dT vT 3 dT ;U
Exp: Given CV=v+3; we know Cv
T T
4

UV PHYSICS ACADEMY
Shivam Road, New Nallakunta, Hyderabad, Ph: 04032458147, 09885072826
www.uvphysics.com
PART-C
EACH QUESTION CARRIES 5 MARKS
EACH WRONG ANSWER HAS A PENALTY OF 25% OF MARKS
COMPULSORY (Q41 TO Q50)
Q41. Consider a system of N non-interacting spins, each of which has classical magnetic moment of

magnitude . The Hamiltonian of this system in an external magnetic field H is H i .H , where i


i 1

th

is the magnetic moment of the i spin. The magnetization per spin at temperature T is
a)

H
H k BT
H


d) tanh
c) sinh
k
T
k
T

H
k
T
B
B
B

2H

b) coth

k BT

Ans: b

2
N
Exp: H i .H H cos for a classical system z e e ( H cos ) sin dd
i 1

2 e H cos sin d ;
0

0 0

4
sin[hH ]
sinh( H ); F kT ln z NkT ln
H
H

NkT ln
ln sinh H
H

1 F
H kT
Magnetisation per spin
; M coth

N H
kT H

Q42. Which of the following is an analytic function of the complex variable z - x + iy in the domain | z |< 2
a) (3 x iy ) 7
b) (1 x iy ) 4 (7 x iy ) 3 c) (1 2 x iy ) 4 d) (3 x iy ) 3 ( x iy 1)1/ 2
Ans: b
Exp: (1 x iy ) 2 (7 x iy ) 3 (1 z) 4 (7 z) 3 ; f 1 ( z) 4(1 z) 3 (7 z) 3 3(1 z) 4 (7 z) 2 which is
differentiable in the domain |z| < 2
Q43. A particle in one dimension moves under the influence of a potential V ( x) ax 6 where a is a real
constant. For large n the quantized energy level E n depends on n as:
b) En ~ n 4 / 3

a) En ~ n 3
Ans: d

c) En ~ n 6 / 5

d) En ~ n 3 / 2
1/ 5

p2
E
ax 6 p x [2 x( E ax 6 )]1/ 2 at x= 0 p x 2mE ; at px = 0 x
Exp: V ( x) ax , H
2m
a
6

1/ 6

E
ellipse a b 2mE
a
1 1

2

V ( x) x s E s

1 1

2

E s

; Area n E 2 / 3 n E n3 / 2 ; (or) If

n E n 3 / 2

area of

UV PHYSICS ACADEMY
Shivam Road, New Nallakunta, Hyderabad, Ph: 04032458147, 09885072826
www.uvphysics.com
Q44. The Lagrangian of a particle of charge e and mass m in applied electric and magnetic fields is given by

1
L mv 2 eA.v e , where A and are the vector and scalar potentials corresponding to the
2
magnetic and electric fields, respectively. Which of the following statements is correct

a) The canonically conjugate momentum of the particle is given by p mv

p2 e
A. p e
b) The Hamiltonian of the particle is given by H
2m m
c) L remains unchanged under a gauge transformation of the potentials.
d) Under a gauge transformation of the potentials, L changes by the total time derivative of a function of

r and t
Ans: d

1
L
Exp: L mv 2 eA.v e canonical momentum p
mv eA; H qi pi L
2
v
2
1
1 ( p eA)
dF
H mv 2 e
e ; L1 L
under gauge transformation of potential.
2
2
m
dt
Q45. A static, spherically symmetric charge distribution is given by (r ) A e kr where
r

A and K are positive constants. The electrostatic potential corresponding to this charge distribution
varies with r as
1
1
kr
a) re
b) e kr
c) 2 e kr
d) 1 (1 e kr )
r
r
r
Ans: d
Q46. Consider two independently diffusing non-interacting particles in 3-dimerisional space, both placed at
the origin at time t = 0. These particles have different diffusion constants D1 and D2. The quantity

R1 (t ) R2 (t )2 , where R1 (t ) and R2 (t ) are the positions of the particles at time t, behaves as:
a) 6t ( D1 D2 )
b) 6t | D1 D2 |
c) 6t D12 D22
d) 6t D1 D2
Ans: a
[common]
A resistance is measured by passing a current through it and measuring the resulting voltage drop. If the
voltmeter and the ammeter have uncertainties of 3% and 4%, respectively, then)
Q47. The uncertainty in the value of the resistance is
a) 7.0%
b) 3.5%
c) 5.0%
d) 12.0%
Ans: a
V I

Exp: we have, V = IR; R V / I


; uncertainties in the value of resistance. = 3%+4%=7%; power
V
I
V I
P=VI; uncertainity in the value of power discipated

3% 4% 7%
V
I
Q48. The uncertainty in the computed value of the power dissipated in the resistance is
a) 7%
b) 5%
c) 11%
d) 9%
Ans: a

UV PHYSICS ACADEMY
Shivam Road, New Nallakunta, Hyderabad, Ph: 04032458147, 09885072826
www.uvphysics.com
[common]
In the absence of an applied torque a rigid body with three distinct principal moments of inertia given by
I1 , I 2 and I 3 is rotating freely about a fixed point inside the body. The Euler equations for the
components of its angular velocity (1 ,2 ,3 ) are
1

I2 I3
I I
I I
23 ; 2 3 1 13 ; 3 1 2 12 ;
I1
I2
I3

Q49. The equilibrium points in (1 , 2 , 3 ) space are


a) (1,-1,0), (-1,0, 1) and (0,-1,1)
b) (1,1,0), (1,0, 1) and (0,1,1)
c) (1,0,0), (0,1,0) and (0,0,1)
d) (1,1,1), (-1,-1, -1) and (0,0,0)
Ans: c
Exp: Under torque free motion, Euler equations for the components of angular velocities
I I
1 2 3 23 (1);
I1
I I
are 2 3 1 13 ......(2);
At the equilibrium point 1 2 3 0;1 0 I 2 I 3 and
I2
I I
3 1 2 1 2 ......(3);
I3
1 a cons tan t (say 1) The equilibrium point is (1, 0, 0) 2 0 (0,1,0) 3 0 (0,0,1) using (1)
1 , (2)by 2 and (3) by 3 I111 ( I 2 I 3 )231 ; I 2 22 ( I 3 I1 )132 ; I 3 33 ( I1 I 2 )123 =
1
2
Adding i.e., I11 I 222 I 332 0; i.e., I 2 0;T I 2 ;2T 0 k.E conserved Again Xing (4) by I1
2
2
2
(5) by I2 and (6) by I3 ; I1 11 I1 ( I 2 I 3 )123 ; I 2 22 I 2 ( I 3 I1 )231 ; I 3 2 33 I 3 ( I1 I 2 )123
Adding I1212 I 2222 I 3232 0 i.e., ( I) 2 0; L I L2 0
i..e angular momentum conserved.
Q50. The constants of motion are
a) 12 22 32 and I11 I 22 I 33
b) I111 I 222 I 332 and I1212 I 2222 I 3232
c) I111 I 222 I 332 and 1 2 3
Ans: b

d) 12 22 32 and I1212 I 2222 I 3232

UV PHYSICS ACADEMY
Shivam Road, New Nallakunta, Hyderabad, Ph: 04032458147, 09885072826
www.uvphysics.com
[common]

In the system consisting of two spin particles labeled 1 and 2, let S (1) (1) and S ( 2) ( 2) denote
2
2

the corresponding spin operators. Here ( x , y , z ) and x , y , z are the three Pauli matrices.
(1)

( 2)

(1)

( 2)

Q51. In the standard basis the matrices for the operators S x S y and S y S x are respectively,
a)

2 1 0 2 1 0

4 0 1 4 0 1

2
c) 0
4 0

0 0 i

0 i 0 2
,
i 0 0 4

0 0 0

b)

0 0 0 i

0 0 i 0
0 i 0 0

i 0 0 0

2 i 0 2 i 0

4 0 i 4 0 i

d) 1
4 0

0 2
,
0 0 i 4

0 i 0

1 0
0 0

0 i

i 0
0 0

0 0

0 0

0 0
0 1

1 0

Ans: b

(1) 0 1 (1) (1) 0 i


Exp: S (1) (1) ; S x(1) x
;Sy y
2
2
2 1 0
2
2 i 0
2 0 i 0 1 2 i 0
S x(1) S y( 2)

4 i 0 1 0 4 0 i
Q52. These two operators satisfy the relation
a) S x(1) S y( 2) , S y(1) S x( 2) S z(1) S z( 2)
b) S x(1) S y( 2) , S y(1) S x( 2) 0

(1)
x

c) S S

( 2)
y

(1)
y

,S S

( 2)
x

iS

(1)
z

d) S

( 2)
z

(1)
x

S y( 2) , S y(1) S x( 2)

Ans: d
(1) ( 2 )
(1) ( 2 )
Exp: S x S y , S y S x 0 ; S x(1) | s (y2) , s (y1) s x( 2) s (y1) s (y2) , s x( 2) s x(1) , s (y1) s x( 2) s (y1) s x(1) , s x( 2) s (y2)

4 i 4



s x(1) s (y1) is z( 2) is z(1) s x( 2) s (y2) ; x y i z i z x y i
0
8
8
2
2 2
2 2
[common]
1 1 1

Consider the matrix M 1 1 1


1 1 1

Q53. The eigen values of M are


a) 0, 1, 2
b) 0, 0, 3
Ans: b
1 1 1

c) 1, 1, 1

d) -1, 1, 3

Exp: M 1 1 1 ; sum = 3; det = 0 = product since all elements same, two eigen value must be zero.
1 1 1

1 , 2 , 3 are 0,0 and 3.

UV PHYSICS ACADEMY
Shivam Road, New Nallakunta, Hyderabad, Ph: 04032458147, 09885072826
www.uvphysics.com
Q54. The exponential of M simplifies to (I is the 3 3 identify matrix)
e3 1
M2
M
a) e M I
b) e M I M
2!
3
c) e M I 33 M

d) e M (e 1)M

Ans: a
Exp: e M I M

1 1 11 1 1 3 3 3
9 9 9
M2
; M 2 1 1 11 1 1 3 3 3; M 3 9 9 9 ;

2!
1 1 11 1 1 3 3 3
9 9 9

1 0 0 1 1 1
3 3 3
9 9 9

1
1
3
32
33
e M 0 1 0 1 1 1 3 3 3 9 9 9 .... I M M M M .....
2!
3!
4!
0 0 1 1 1 1 2! 3 3 3 3! 9 9 9

M
32 33
M
32 33
M 3
I [3 .....] I 1 3 ... 1 e M I
e 1
3
2! 3!
3
2! 3!
3

CHOOSE ANY 10 OF THE REMAINING QUESTIONS


[common]
13
The radius of a 64
cm .
29 Cu nucleus is measured to be 4.8 10

27
Q55. The radius of a 12
Mg nucleus can be estimated to be
13
a) 2.86 10 cm
b) 5.2 1013 cm
c) 3.6 1013 cm
d) 8.6 10 13 cm
Ans: c
27
Exp: Radius R R0 A1/ 3 ; R0 1.2 10 15 m for 12
Mg , A 27; R 1.2 1015 (27)1/ 3 R 3.6 1013 cm
Q56. The root mean square (rms) energy of a nucleus of a atomic number A in its ground state varies as
a) A4 / 3
b) A1 / 3
c) A1 / 3
d) A2 / 3
Ans: c
[common]
The character table of C3v the group of symmetries of an equilateral triangle of an equilateral triangle is given
below

1C1
3C2
2C3

(0)

(1)

( 2)

1
1
1

1
A
1

b
c
d

In the above C1 , C2 , C3 denote the three classes of C3v containing 1, 3 and 2 elements respectively, and

(0) , (1) and ( 2) are the characters of three irreducible representations (0) , (1) and ( 2) of C3v .
Q57. The entries a, b, c and d in this table are respectively.
a) 2, 1, -1, 0
b) -1, 2, 0, -1
c) -1, 1, 0, -1
Ans: a

d) -1, 1, 1, -1

UV PHYSICS ACADEMY
Shivam Road, New Nallakunta, Hyderabad, Ph: 04032458147, 09885072826
www.uvphysics.com
Q58. The reducible representation of C3v with character (4,0,1) decomposes into its irreducible
representations ( 0) , (1) , ( 2) as
a) 2 (0) 2 (1)
b) (0) (1) ( 2)
c) (0) 3 (1)
d) 2 ( 2)
Ans: a
[common]
Light of wavelength 660 nm and power of 1mW is incident on a semiconductor photodiode with an absorbing
layer of thick ness of (ln4) m
Q59. If the absorption coefficient at this wavelength is 10 4 cm 1 and if 1% power is lost on reflection at the
surface, the power absorbed will be
a) 750W
b) 675W
c) 250W
d) 225W
Ans: c
Exp: Absorption coefficient 10 4 cm 1 The power absorbed, p p0 e k ;
4
1000
p=1000 e 10 ln 4 104 1000(e ln 4 ) 1 ; p
250W
4
Q60. The generated photo current for a quantum efficiency of unity will be
a) 400A
b) 360A
c) 133A
d) 120A
Ans: c
nhc
p 6.60 10 9 250 10 6 250
Exp: p nhv; p
n

103 ;
34
8

hc
3
6.6 10 3 10
250
I ne
1013 1.6 10 19 133A
3
[common]
The magnetic field of the TE11 mode of a rectangular waveguide of dimensions a x b as shown in the figure is
given by H z H 0 cos(0.3x) cos(0.4 y) , where x and y are in cm.
Q61. The dimensions of the waveguide are
a) a = 3.33cm, b = 2.50cm
b) a = 0.40cm, b = 0.30cm
c) a = 0.80cm, b = 0.60cm
d) a = 1.66 cm, b = 1.25cm
Ans: a
mx
ny
Exp: H z H 0 cos(0.3x) cos(0.4y) we have H z H 0 cos
cos
a
b
m
n
0.3, 0.4; TEmn TE11 m 1, n 1 i.e.,
a
b
1
1
0.3 a 3.33cm; 0.4 b 1.25cm
a
b

Q62. The entire range of frequencies / for which the TE11 mode will propagate is
a) 6.0GHz <f < 7.5GHz
b) 7.5GHz <f < 9.0GHz
c) 7.5GHz <f < 12.0GHz
d) 7.5GHz <f
Ans: d
2

c 1
1
c m n
11
2 7.5GHz. for propagation, frequency of incident wave must
Exp: f c
; f
2
2 a
b
c a b
be greater than cutoff frequency.

UV PHYSICS ACADEMY
Shivam Road, New Nallakunta, Hyderabad, Ph: 04032458147, 09885072826
www.uvphysics.com
[common]
Consider the energy level diagram (as shown in the figure below) of a typical three level ruby laser system
with 1.6 xl019 Chromium ions per cubic centimeter. All the atoms excited by the 0.4 urn radiation decay
rapidly to level E2 which has a lifetime = 3ms.

Q63. Assuming that there is no radiation of wavelength 0.7 m present in the pumping cycle and that the
pumping rate is R atoms per cm3, the population density in the level N2 builds up as
2
a) N 2 (t ) R (e t / 1) b) N 2 (t ) R (1 et / ) c) N 2 (t ) Rt (1 e t / ) d) N 2 (t ) Rt

Ans: a
Q64. The minimum pump power required (per cubic centimeter) to bring the system to transparency, i.e., zero
gain, is
a) 1.52kW
b) 2.64kW
c) 0.76kW
d) 1.32kW
Ans: d
[common]
A flux quantum (fluxoid) is approximately equal to 2xlO-7 gauss-cm2. A type II superconductor is placed in a
small magnetic field, which is then slowly increased till the field starts penetrating the superconductor.
2
The strength of the field at this point is 10 5 gauss.

Q65. The penetration depth of this superconductor is


a) 100
b) 10
c) 1000
d) 314
Ans: a
hc
2
Exp: Flux quantum
2 10 7 gauss cm 2 ; B 105 gauss. The relation connecting penetration
2e

2
2 10 7 gauss c.m 2
depth L and B is B 2 ; 105 gauss
;
L
L 2

2L 1012 cm 2 L 106 cm 100 108 cm 100 Ao


Q66. The applied field is further increased till superconductivity is completely destroyed. The strength of the
8
field is now 10 5 gauss. The correlation length of the superconductor is

a) 20
Ans: b

b) 200

c) 628

d) 2000

8
Exp: B is increased to B0 where supercondivity disappears. B B0 ; B 0 2 Given B0 105 gauss;

L0

105 gauss

L1 100 A0
10 12 2
2 10 7 guass cm 2
2
0

cm

50
A
;
;
k

2 Correlation
L0
L
4
L0 50 A0
2L0

length k L1 2 100 200 A0

UV PHYSICS ACADEMY
Shivam Road, New Nallakunta, Hyderabad, Ph: 04032458147, 09885072826
www.uvphysics.com
[common]
A beam of poins ( ) is incident on a proton target, giving rise to the process p n
Q67. Assuming that the decay proceeds through strong interactions, the total isospin I and its third component
I 3 for the decay products, are
3
3
5
5
1
1
a) I , I 3
b) I , I 3
c) I 5 , I 3 3
d) I , I 3
2
2
2
2
2
2
2
2
Ans: c
Exp: p n for strong interaction I and I3 are conserved. Conservation of I and I3
1
5
1
3
n ; I 11 ; I3
1 1 ; I 5 / 2 and I 3 3 / 2 for the decay products.
2
2
2
2
Q68. using isospin symmetry, the cross section for the above process can be related to that of the process.
a) n p
b) p n
c) n p
d) p n
Ans: b
Exp: The reaction cross section related to p n same as p n
[common]
The two dimensional lattice of grapheme is an arrangement of carbon atoms forming a honeycomb lattice of
lattice spacing a, as shown below. The carbon atoms occupy the vertices.

Q69. The Winger Seitz cell has an area of


3 2
3 3 2
a) 2a 2
b)
c) 6 3a 2
d)
a
a
2
2
Ans: d
Exp: Area of cell = 6 Area of triangle AOB ; Area of AOB=(1/2)bh;

1
1
1
3
3 2
3 2 3 3 2
( BO ) Ay a a cos 30 0 a 2

a area of W-s Cell = 6


a
a
2
2
2
2
4
4
2
Q70. The Bravais lattice for this array is a
a) rectangular lattice with basis vectors d1 and d 2
b) rectangular lattice with basis vectors c1 and c2
c) rectangular lattice with basis vectors a1 and a 2
d) rectangular lattice with basis vectors b1 and b2
Ans: d

Exp: The Bravais lattice for this array is a hexagonal lattice with basis vectors b1 and b2

UV PHYSICS ACADEMY
Shivam Road, New Nallakunta, Hyderabad, Ph: 04032458147, 09885072826
www.uvphysics.com
[common]
Consider the decay process v in the rest frame of the . The masses of the , and v are
M , M and zero respectively.

Q71. The energy of is


( M 2 M 2 )c 2
( M 2 M 2 )c 2
a)
b)
c) (M M )c 2
d) M M c 2
2M
2M
Ans: b
Exp: pbefore 0, pafter p pv ; Ebefore m c 2 , Eafter E Ev ; conservation of momentum
p pv 0 p pv (1) ; Conservation of

energy E Ev m c 2 (2); Ev pv c; E 2 p 2 c 2 m2 c 4 ; pv E 2 m2 c 4 ; Ev E2 m2 c 4

(2) E E 2 m2 c 4 m c 2 ; E m c E
2

m2 c 4 Squaring we get E2

(m m2 )c 2
2m
2

Q72. The velocity of is


M c
M c
( M 2 M 2 )c 2
( M 2 M 2 )c 2
a)
b)
c)
d)
2
2
2
2
(M M )
(M M )
M
M
Ans: a
Exp: v Law of conservation of energy E 0 Ev m c 2 ; E pv c m c 2 (| v || |)

E c m c ; E E m c m c ( E m c ) E m c E
2

fine , use E c m c ;
2

| |

2 2

m2 c 2
to
2m

E2
c 2 (m2 m2 )
c(m2 m2 )
2 2
2 2
2 m c
m c ; | |
;;
2m
c
4m2

m 2 m2
| |
c(m2 m2 )
c
; v c 2 2
2
E
2m
m m

[common]
A narrow beam of X rays with wavelength A 0 is reflected from an ionic crystal with an fcc lattice structure
with a density of 3.32 gcm 3 . The molecular weight is 108AMU
(1 AMU = 1.66 10 24 g )
Q73. The lattice constant is
a) 6.00 A 0
b) 4.56 A 0
c) 4.00 A 0
d) 2.56 A 0
Ans: a
Exp:

M n
nm 4 108 1.66 10 24
3 M
24
3

3
.
32
g
/
cm
;

108

1
.
66

10
;
;
Since
FCC
,
n

4
;
a

6 Ao
N
N a3

3.32

Q74. The sine of the angle corresponding to (111) reflection is


a) 3 / 4
b) 3 / 8
c) 1/4
d) 1/8
Ans: b
a
6 0

1.5 3
3

A Braggs condition 2d sin n; sin


Exp: d
;n 1

2
2
2
2d
2 6
8
3
h k l

UV PHYSICS ACADEMY
Shivam Road, New Nallakunta, Hyderabad, Ph: 04032458147, 09885072826
www.uvphysics.com
Q75. If an electron is in the ground state of the hydrogen atom, the probability that its distance from the
proton is more than one Bohr radius is approximately.
a) 0.68
b) 0.48
c) 0.28
d) 0.91
Ans: a
a0
4 a0
1
1 a0
Exp:
e r / a0 ; p | |2 d 3 | e 2 r / a0 r 2 dr sin dd ; 3 r 2 e 2 r / a0 dr
a0 0
a0 0
0
a03
a0

2 a0 2 r / a0
a02
a03 2 r / a0
1 2 1 2 1 2 1
2
r

2
r

2
e

4 e e e 1 5e Probability for
2
4
8
2
4
4
2

0
finding more than one bob radius is p 1 (1 5e 2 ) 5e 2 0.68
Q76. A time varying signal Vin is fed to an op amp circuit with output signal V0 as
shown in the figure below. The circuit implements a
a) high pass filter with cutoff frequency 16Hz
b) high pass filter with cutoff frequency 100Hz
c) low pass filter with cutoff frequency 16Hz
d) low pass filter with cutoff frequency 100Hz
Ans: c
1
Exp: Cutoff frequency f c
RC ; f c 15.9; so the given op-amp becomes a low pass filter with cutoff
2
frequency 16.
Q77. The Hamiltonian of a particle of unit mass moving in the xy plane is given to be:
1
1
H xp x yp y x 2 y 2 in suitable units. The initial values are given to be (x(0), y(0)) = (1, 1) and
2
2
px (0), p y (0) 1 , 1 . During the motion, the curves traced out by the particles in the xy plane and
2 2
p x p y - plane are
4
3
a0

a) both straight lines


b) a straight line and a hyperbola respectively
c) a hyperbola and an ellipse , respectively d) both hyperbolas
Ans: d
1
1
H
dx
dx
Exp: H xp x yp y x 2 y 2 ; xy and pxpy ; x
x; ; x x; x dt; x e t e
p x
2
2
dt
x

H
dy
dy
y; y;
dt y e t c from initial condition x(t)=et ,
p y
u
dt
dp
H
p x x x p x x ; p x (et ) et et dt c1 ;
y(t ) e t x. y 1( Hyperbola) p x
x
dt
dp
1
H
1
y
p x (e t ) e 2t c1 ; p y
py y
p y y; p y (e t ) e t e t dt c2 ; p y (e t ) e 2t c2
2
2
y
dt
1
from initial conditions p x p y ( Hyperbola )
4
y

UV PHYSICS ACADEMY
Shivam Road, New Nallakunta, Hyderabad, Ph: 04032458147, 09885072826
www.uvphysics.com
Q78. Consider an ideal Bose gas in three dimensions with the energy-momentum relation p with s > 0.
The range of s for which this system may undergo a Bose-Einstein condensation at a non-zero
temperature is
a) 1 < s <3
b) 0 < s <2
c) 0 < s < 3
d) 0 < s <
Ans: d
p2
Exp: p s photon pc; i.e, p for a free non relativistic particle
i.e., p 2 ; so there is no
2m
restriction to energy so s takes values from 0 to
Q79. Two gravitating bodies A and B with masses m A and mB , respectively, are moving in circular orbit.
s

Assume that mB >> m A and let the radius of the orbit of body A be RA . If the body A is losing mass
adiabatically, its orbital radius RA is proportional to
2
a) 1 / m A
b) 1 / m A2
c) m A
d) m A
Ans: d
2
2
GM A M B
GM A M B m AV A
GM A M B
mv 2 GMm m AVA2 GM A M B
2
Exp:
R

; RAM A2
2 ;

;
m
V

;
A
A A
mA
RA
r
RA
RA
p2
r
RA2

UGC CSIR NET GENERAL QUESTIONS JUNE 2009


1. Electrical charge is stored in
1) Battery
2) Capacitor

http://physicskerala.blogspot.com

3) voltmeter

4) Wire

t.c
om

2. Boiling point of water at sea level is 100 C. What would be its boiling point at top of Mount
Everest???
1) 100C
2) 104 C
3) 114 C
4) 74o C

sp
o

3. If iodine stored in a closed chamber is slowly evacuated to sublimate. What would be effect on
sublimation rate and mean free path?
1) Both will increase
2) Both will decrease
3) Sublimation rate will increase while free path decrease
4) Sublimation rate decrease and free path increase

.bl
og

4. As shown in graph solubility of CuSO4 increases as the temperature of solution is increased.


Suppose under saturated condition temperature of solution is dropped from 60o to 30o ,amount of
CuSO4 deposited will be
60

er
ala

50
40
30
Solubility
(g) 20
10

tp
://
ph
ys
ics
k

10 20 30 40 50 60 70 80 90 100
Temperature

1) 24g

2) 44g

3) 20g

4) 100g

5. Which of the following is not a major green house gas in stratosphere?


1) CO2
2) Methane
3) Ozone
4) Water vapours
6. Atmospheric pressure decline with altitudes as shown in table

Height
0km
2km
4km
6km
Pressure (mbar) 900
800
650
450
What would be atmospheric pressure at height of 5km?
1) 720
2) 550
3) 640
4) 420

8km
200

ht

7. Among the following which ocean receives maximum sediments?


1) Arabian Ocean
2) Indian Ocean
3) Bay of Bengal
4) Dead Sea
8. Mostly inner material of earth remains in solid state. Seldom has it melted and do not remain
inside and expelled to the surface of earth because
1) It is just beneath the earth crust
2) Due to buoyancy
3) Due to high pressure
4) More density of surrounding rocks
9. The amount of rainfall in summer at any place is shown in table.

Send sms JOIN PHYSICS43 to 567678 http://physicskerala.blogspot.com

UGC CSIR NET GENERAL QUESTIONS JUNE 2009


June
10

Dec
450

3) Australia

4) Srilanka

t.c
om

Jan
March
550
30
The probable place would be
1) India
2) North America

http://physicskerala.blogspot.com

10.
Half life of any radioactive material is 50 days. How many half life it will take to become
12. 5 % of the original amount?
1) 1
2) 2
3) 3
4) 4

sp
o

11.
Which of them will have minimum resistance to flow of electric current?
1) Glass
2) Saline aquifer
3) Granite
4) Lime stone

2)

1)

m
r

re

re

re

re

er
ala

4)

3)

.bl
og

12.
Assuming equal density through out different layers of the earth, if radius r of selected part
is gradually increased from centre of earth (where r<re (radius of earth)) what would be correct
graphical representation for change in mass?

13.
What is probability of getting first three female pups out of a litter of 7?
1) 1/8
2) 7/8
3) 3/27
4) 37/64

tp
://
ph
ys
ics
k

14.
If two sides of isosceles triangle are 7 cm and 16 cm respectively. What would be length of
its third side?
1) 7 cm
2) 16cm
3) 23cm
4) (162-72)-1/2
15.
A crystalline sphere (cylinder??) of radius 1 cm is broken into sphere of radius 0.01 cm
radius each. What would be change in surface area?
1) 0
2) 10
3) 100
4) 1000
Graphical representation for function e-lxl will be

16.
1

ht

17.
A committee of two members has to be selected out of 3 men and 2 women. In how many
possible ways it can be done
1) 10
2) 25
3) 100
4) 120
18.
A non-malignant tumor with radius r shrinks at constant rate with time t. It can be
represented by equation
1) r = r0 + k/t
2) r = r0 - k/t
3) r = r0 - kt
4) r = r0 - k/t

Send sms JOIN PHYSICS43 to 567678 http://physicskerala.blogspot.com

UGC CSIR NET GENERAL QUESTIONS JUNE 2009

http://physicskerala.blogspot.com

20.
Which logical gate is represented by the following truth table?
1) AND
2) OR
3) NOR
4) XOR

t.c
om

19.
If value of is 360, then as per equation r=a, shape of object would be (Where r is
distance from origin and a is constant)
1) Spiral
2) Circle
3) Sphere
4) Ellipse

.bl
og

sp
o

P
Q Output
0
0
0
0
1
0
1
0
0
1
1
1
21.
If five flowers have nectar amount 10, 20, 30, 40, 50 l respectively. If a bee consumes all
the nectar from flowers, then at the end bee is rewarded with how much mean amount of nectar
1) 10
2) 20
3) 30
4) 150

4) COBOL

23.
Consider the following computer program
Input Z
Do
A= 3.143*Z*Z
Print A
The program computes area of a
1) Circle
2) Sphere
3) Triangle

4) Square

er
ala

22.
Among the following which is a object oriented language
1) PASCAL
2) FORTRAN
3) C++

tp
://
ph
ys
ics
k

24.
If all parameters related with cockroach are doubled such as height width and length, it will
not survive because of
1) Low surface area to volume ratio
2) High surface area to volume ratio
3) Exchange of gases
4) Problem is excretion
25.
Major weight of human body is due to
1) C
2) P
3) N
4) O
26.
Starch on treatment with dilute H2SO4 yields free glucose but cellulose not because
1) Cellulose is linear
2) Cellulose is branched
3) Starch is carbohydrate
4) Starch is linear
27.
Sum of two binary numbers 101 and 011 would be
1) 1000
2) 100
3) 101
4) 1001

ht

28.
What would be effect on shape of copper metallic tube (??) carrying electric current due to
nerate magnetic field
1) No effect
2) It will swell from middle
3) It will shrink from middle
4) will be eclipse shape

Send sms JOIN PHYSICS43 to 567678 http://physicskerala.blogspot.com

UGC CSIR NET GENERAL QUESTIONS JUNE 2009


The graph represents

t.c
om

29.

http://physicskerala.blogspot.com

Rate

20
30
40
Temperature

1) Exothermic reaction
3) Endothermic reaction

50

60

70

2) Isolated reaction
4) Physiological reaction

sp
o

10

.bl
og

30.
Area required to store fats in seed as compare to carbohydrate would be
1) Equal
2) More
3) Less
4) Slightly more

er
ala

31.
Coriolis force is due to rotation of earth on moving object. The direction of Coriolis force is
1) Along the axis of rotation of the moving object
2) Against the axis of rotation of the moving object
3) Perpendiculars to the axis of rotation of the moving object
4) Tangential to the axis of rotation of the moving object

tp
://
ph
ys
ics
k

32.
If parents with genotype AABBccddeeFF and aabbCCDDEEff are crossed, the genotype of
resulting progeny will be
1) AABBccDDeeFf
2) AaBbCcDdEeFf
3) aaBBccDDeeFF
4) AaBbCCddEeFf
33.
Terminal electron acceptor for metabolic reactions in organisms is
1) CO2
2) H2
3) O2
4) H2O
34.
What would be effect on time period of pendulum one laced on equator and other on pole
1) No effect
2) Time period would be greater at poles
3) Time period would be greater at equator
4) Pendulum will stop at poles
35.
The effect of input of any fertilizer on rice yield is shown in graph. The optimum utilization
of nutrient is at point?

ht

Rice
Yield

c
a

b
Phosphate amount

1) a

2) b

3) c

4) d

Send sms JOIN PHYSICS43 to 567678 http://physicskerala.blogspot.com

UGC CSIR NET GENERAL QUESTIONS JUNE 2009

http://physicskerala.blogspot.com

36.
Angle between two vector 2i + 3j and 3i - 2j will be
1) 30 o
2) 45o
3) 60o
4) 90o

t.c
om

37.
At ground state of hydrogen atom its Bohr radius is 5.3 x 10-11 m and mean velocity is 2.1 x
106rn/s. What would be value of fundamental time unit?
2) 252x10 -5sec
1) 2.52x10 -17sec
4) 1.52 x 10 -17 sec
3) 1.2 x 10 -17sec

sp
o

38.
During combustion of carbon in presence of oxygen CO2 is formed, what will be effect on
the reaction if availability of oxygen is doubled?
1) No Change
2) Will double
3) Will half
4) Will increase four times

.bl
og

39.
Relative mean kinetic energy or Helium atom (atomic weight 4) and Argon ( atomic weight
40) would be
1) 1:10
2) 1:4
3) 1:100
4) 1:16

KEY
2) 4
14) 2
26) 1
38) 1

3) 1
15) 3
27) 1
39) 1

4) 1 5) 4 6) 2 7) 3 8) 2 9) 3 10) 3 11) 2 12) 2


16) 2 17) 1 18) 2 19) 1 20) 1 21) 4 22) 3 23) 1 24) 1
28) 2 29) 4 30) 3 31) 3 32) 2 33) 3 34) 3 35) 3 36) 4
40) 1

tp
://
ph
ys
ics
k

1) 2
13) 1
25) 1
37) 1

er
ala

40.
The main reason for release of energy from sun is
1) Fusion of hydrogen
3) Fusion of Helium
2) Fission of hydrogen
4) Fission of Helium

ht

This is constructed from memory. There may be errors or corrections. Author is not responsible
for errors. Please notify the errors and correction at physicskerala.blogspot.com contact us
link.

Send sms JOIN PHYSICS43 to 567678 http://physicskerala.blogspot.com

UGC CSIR NET PAPER2 QUESTIONS JUNE 2009

http://physicskerala.blogspot.com

2. A differential equation  

 


+u




co
m

UGC CSIR NET PAPER-2 PHYSICAL SCIENCE FROM MEMORY


PART A
1. (z) = tanh z; find the singularities, natures of singularities and residues

+ y =0; assuming u = log x ; find the solution of the

above differential eqn. Give the general solution.

3. v(x ) = x x 0 ; x <0 ;  2


=   ; find the energy values using WKB



sp

ot.

approximation.
4. O15 decays by positive beta decay into N15 find the Emax interms of their columbic attractions
Ecoul.
Maximum energy of positive beta emission is given . find the ....
(I saw similar problem in Arthur Beissers text)
5. A hexagonal & isosceles triangle is given we have to find the magnetic induction.

era
la.
blo
g

What is the magnetic field at a point 'p' which is at a perpendicular distance 'r' from a segment
of wire carrying a current 'i' making an angle theta1 and theta2 w.r.t. the two ends of the
segment diagram given.
P

ys

ics
k

A wave function is given. From Schrdinger equation we have to find the energy eigen
values.???
7. A question from Einstein A coefficient A31 , A21 , A32 are given 1x107s 7x107s

ph

6.

Send sms JOIN PHYSICS43 to 567678 http://physicskerala.blogspot.com

http://physicskerala.blogspot.com

co
m

UGC CSIR NET PAPER2 QUESTIONS JUNE 2009

ot.

1) .
2) Find the life time of the level 3
3) If N/cm3 is what is .population inversion.

era
la.
blo
g

sp

8. Two polarizer plates are placed perpendicular to each other. If unpolarised light passing
through it what will be the intensity I2 in the output. If the first plate is rotating with a
frequency 1 and the second plate is rotating with a frequency 2 , then find the equation for
the intensity. Also draw a graph for I2.
9. Find the energy levels and expectation values if a harmonic oscillator is restricted only in one
side.
10. An unit mass partical in a potential energy V(r)=-1/2q2 (a)find hamiltonian H ,(b)calculate
poisson braket of H with F(q,p)=pq/2q.2 (c)from poisson braket show that F is conserve.
PART - B
11. Reduce the logic circuit in to a single level logic circuit.

A
B
C

ph

ys

ics
k

12. 5Kg ice at 273K is connected to a sink which is at 373K


a. Find the change of entropy of ice.
b. Find the change of entropy of sink
c. Find the change of entropy of universe.
13. A particle of mass m is in a central force, r = r0e-i.
a. Is the angular momentum is conserved.
b. Find the law of force.
c. Find the total energy.
d. ?
14. Using variation principle calculate minimum energy of a system of potential energy
V(r)=exp(),consider the trial wave function exp(-ar2)
15. draw the interaction picture of some reactions. the quark structure is given. (-+-)/root(2)
Draw the diagrams of        &    + 
16. *Q1 : Two spin half particle connected in a heat reservoir at temp. T ,can stay any of three
energy levels , 0 , and + . Then find partition function for (a)MB statistics (b)BE ,(c)FD
Three particles with energies , 0 , +, find the partition function in MB,BE and FD
statisitics.
Send sms JOIN PHYSICS43 to 567678 http://physicskerala.blogspot.com

UGC CSIR NET PAPER2 QUESTIONS JUNE 2009

http://physicskerala.blogspot.com

ot.

co
m

17. (a) n2 - n1 transition and (b) n3-n2 transition of H atom find the recoil momentum in each case.
find also the recoil velocity in the first case.
18. In the following circuit {A noninverting op-amp(where feadback resistance is a diode)}
show that output voltage V0 is a logarithamic function.

Eg = .

f = 0.1 (??)

pe = .

era
la.
blo
g

For P type

sp

19. A question from three dimensional Fermi energy


20. Draw the energy lel diagram of a semi conductor junction from the following data. Label each
values. Also find the work function of the semiconductor.
Conduction
For N type
Eg = 1.2
f = 0.2 (??) ne = .
???

21. For a n type semi conductor with Nd=?*10 raise to19,Eg=?.when a pure semi condtor
formed,wat s the value of Ef in terms of tempertr.Nc=__,Nd =__.[the __ Means the values
given]
22. a)Write the frequency values of different polarizations in dielectrics.
b)A problem was given based on the equation

   1,    in this problem,

refractive index was given


23. Problem from Fermi gold transitions, where the transitions are valid or not
0+  0 +
1+  1+
3/2+  5/2+
like ( the equation given is not exactly as in the question paper)

ics
k

24. It is practically proved that the Hydrogen atom contributes to the rotational energies at
100K.Using this data find the Bond length of Hydrogen
25. Rotational partition function problem: Energy is given from that values we have to find the
bond lengths. A). j=2. B)j = 4 to j =2 etc.
26. A particle falling from a height (?) onto to a square plane sheet of charge density(?), find the
force on the particle

ys

27. Probablities : Two particles of mass m1 and m2 having the probabilities P1(m1) =
=

 !

, P2(m2)

 !

ph

Find the probabilities of the function P(n) =


,      
28. An Analog to digital converter what are the functions of IC1 AND IC2. Also give the function
of V and Rs
29. Problem on experimental technique: an experimental technique regarding emission of electrons
by heating a coil (With a high potential difference to emit electrons from the target material.
Role of different parts & their names.)
Send sms JOIN PHYSICS43 to 567678 http://physicskerala.blogspot.com

UGC CSIR NET PAPER2 QUESTIONS JUNE 2009

http://physicskerala.blogspot.com

co
m

A connection 1 vacuum pump tungsten filament constant current source


constant voltage source micro ammeter . name each components CDEFG and give ther
functions and give the theory behind it.
30. A question from WKB vx^2 (is a second one?)
31. One problem on quark theory
32. A problem on particle decay combined with quantum mechanics

ph

ys

ics
k

era
la.
blo
g

sp

ot.

The question paper contains 10 Questions in PartA and 20 Questions in PartB. Total = 30.
(Some of the questions have parts). Here the 31 and 32 only a hint to remember. Pls reply your
comments to physics43@gmail.com

Send sms JOIN PHYSICS43 to 567678 http://physicskerala.blogspot.com

htt
p:/
/ph
ys

ics
k

era
la.
blo
g

sp

ot.
co
m

Part B UGC CSIR NET question 2009 June PHYSICAL SCIENCE - Memory Based

htt
p:/
/ph
ys

ics
k

era
la.
blo
g

sp

ot.
co
m

Part B UGC CSIR NET question 2009 June PHYSICAL SCIENCE - Memory Based

2 physicskerala.blogspot.com

htt
p:/
/ph
ys

ics
k

era
la.
blo
g

sp

ot.
co
m

Part B UGC CSIR NET question 2009 June PHYSICAL SCIENCE - Memory Based

3 physicskerala.blogspot.com

htt
p:/
/ph
ys

ics
k

era
la.
blo
g

sp

ot.
co
m

Part B UGC CSIR NET question 2009 June PHYSICAL SCIENCE - Memory Based

4 physicskerala.blogspot.com

htt
p:/
/ph
ys

ics
k

era
la.
blo
g

sp

ot.
co
m

Part B UGC CSIR NET question 2009 June PHYSICAL SCIENCE - Memory Based

5 physicskerala.blogspot.com

htt
p:/
/ph
ys

ics
k

era
la.
blo
g

sp

ot.
co
m

Part B UGC CSIR NET question 2009 June PHYSICAL SCIENCE - Memory Based

6 physicskerala.blogspot.com

htt
p:/
/ph
ys

ics
k

era
la.
blo
g

sp

ot.
co
m

Part B UGC CSIR NET question 2009 June PHYSICAL SCIENCE - Memory Based

7 physicskerala.blogspot.com

htt
p:/
/ph
ys

ics
k

era
la.
blo
g

sp

ot.
co
m

Part B UGC CSIR NET question 2009 June PHYSICAL SCIENCE - Memory Based

8 physicskerala.blogspot.com

Potrebbero piacerti anche